Überlichtschnelle Neutrinos?

BLOG: RELATIV EINFACH

… aber nicht einfacher
RELATIV EINFACH

Meinem persönlichen Eindruck nach ist die fundamentale Teilchenphysik in den letzten Jahren eher weniger aufregend gewesen. Das Standardmodell der Teilchenphysik ist nach wie vor ein nicht besonders schönes Gebilde, und mir war schon vor einem Jahr als Blogger beim Lindauer Nobelpreisträgertreffen aufgefallen, dass die Biologen, z.B. Jack Szostak, dort hochspannende Vorträge über rasante Fortschritte der letzten Jahre hielten, während der Vortrag z.B. des theoretischen Teilchenphysikers David Gross im großen und ganzen genau so auch vor zehn Jahren hätte gehalten werden können. Hat sich das mit der neuesten CERN-Meldung geändert?

Erst noch einmal ein paar Schritte zurück. Die Neutrinos, in der es bei der neuen Meldung geht, waren bereits für die letzte größere Erschütterung in der Teilchenwelt verantwortlich. Das war vor etwas mehr als 10 Jahren; das Problem ließ sich aber noch deutlich weiter zurückverfolgen: In den 1960er Jahren hatten Raymond Davis (der später für seine Arbeiten den Nobelpreis bekam) und der Astrophysiker John Bahcall (der sich u.a. unermüdlich für den Bau des Weltraumteleskops Hubble einsetzte) ein Experiment durchgeführt, das von der Sonne kommende Neutrinos nachweisen sollte (genauer: Bahcall hatte die theoretische Vorarbeit geleistet, Davis das Experiment gebaut).alt

Neutrinos sind schwer nachzuweisen, da sie nur höchst selten mit normaler Materie in Wechselwirkung treten. Ursprünglich wurde ihre Existenz nur postuliert, weil in bestimmten Reaktionen ansonsten Energie, Impuls und Drehimpuls verloren gegangen wären. Sie entstehen bei Kernreaktionen, die mit der schwachen Kernkraft zu tun haben, einer der vier Grundkräfte der Physik. Das machte sie zu idealen Probeteilchen für Bahcall, der einen Großteil seiner Karriere der Entwicklung des Standardmodells der Sonne widmete und das Problem hatte, dass man nun einmal nicht ins Sonneninnere sehen kann. Dagegen entstehen im Sonneninneren bei den Kernfusionsreaktionen, die die Sonne letztlich zum Leuchten bringen, Unmengen von Neutrinos, von denen die meisten (kaum Wechselwirkung mit Materie!) weitgehend unbeschadet in den Weltraum fliegen. Diese Neutrinos wollte Bahcall nachweisen, um sein Modell der Sonne zu überprüfen. Die Detektoren dafür sind typischerweise große Tanks, umgeben von konventionellen Teilchendetektoren – man wartet darauf, dass Neutrinos mit der Tankmaterie reagieren und sich das Ergebnis nachweisen lässt.

Solche Detektoren haben eine höchst geringe Nachweisrate (kaum Wechselwirkung von Neutrinos mit Materie eben). Diese geringe Nachweisrate kann man aber in die Vorhersage mit einbauen, und dann erhält man eine Aussage darüber, wieviele Sonnenneutrinos ein gegebenes Experiment denn nun im Mittel auffangen sollte.

In Davis’ Experiment, das in 1,5 km  Tiefe in einem Bergwerd in Süd-Dakota aufgestellt war (damit andere, störende Teilchen gar nicht erst bis dorthin vordringen sollten) kamen aber bei weitem nicht soviele Neutrinos an wie von Bahcall vorhergesagt. Das “Problem der Sonnen-Neutrinos” war geboren, bei dem sich Astrophysiker/Kernphysiker und Teilchenphysiker über Jahrzehnte gegenseitig den Schwarzen Peter zuschoben, was denn nun schiefgegangen war und worin die Diskrepanz bestand (“Repariert mal euer Sonnenmodell!” – “Bringt ihr doch erstmal eure Experimente in Schuss!” – zugegeben, das ist leicht vereinfacht.)

Solch ein Konflikt ist für die Wissenschaft selbst dann von großem Nutzen, wenn keine fundamental neuen Erkenntnisse dabei herauskommen, denn er gibt vielen Leuten besondere Motivation, jedes Glied der Argumentationskette, inklusive der Argumente und Rechnungen des Standardmodells der Sonne kritisch und ganz genau unter die Lupe zu nehmen. Bei Experimenten, bei denen alles so läuft wie erwartet, wird höchstwahrscheinlich etwas weniger in die genaue Prüfung und in Gegenproben investiert. Nicht zuletzt: Förderanträge dürften eine deutlich größere Chance auf Bewilligung haben, wenn ein ungelöstes Problem im Raum steht, als wenn jemand einfach nur mal so sein bereits durchgeführtes Experiment überprüfen möchte.

In diesem Fall gab es dann aber in der Tat fundamental neue Erkenntnisse. Kurz vor der Jahrtausendwende gab es erste belastbare Hinweise darauf, dass sich die drei verschiedenen Sorten von Neutrinos, die es im Standardmodell gibt, ineinander umwandeln können. Wenn ein so genanntes Elektron-Neutrino losfliegt, eben z.B. im Sonneninneren, muss in großer Distanz nicht unbedingt auch ein Elektron-Neutrino ankommen. Stattdessen könnte auch ein Muon-Neutrino oder ein Tauon-Neutrino (das sind die beiden anderen Sorten) ankommen. Das war die Auflösung des Rätsels der fehlenden Neutrinos. Mit den neuen Experimenten wurde klar: In der Sonne waren in der Tat soviele Elektron-Neutrinos losgeflogen, wie die Astrophysiker vorhergesagt hatten. Nur hatten sich eine Menge davon auf ihrem Weg zur Erde in andere Neutrinosorten verwandelt. Und die Experimente, die ebenfalls nur Elektron-Neutrinos nachweisen konnten, meldeten dann eben eine Diskrepanz. Bei einem Experiment, das alle Neutrinosorten nachweisen konnte, stimmte die Bilanz dann wieder.

Das war noch aus einem anderen Grunde interessant, denn damit diese Umwandlungsreaktionen stattfinden können, müssen Neutrinos eine Masse besitzen. Bis dahin hatte man die Neutrinos für masselose Teilchen gehalten. Aber auch hier zeigt sich, warum das Standardmodell der Teilchenphysik nicht besonders elegant ist. Den Umstand, dass Neutrinos Masse besitzen, konnte man dort ziemlich einfach einbauen. Zugespitzt gesagt: Es gab keinen Aufschrei, kein großes Krachen, keine Verschiebungen der ganzen Struktur aufgrund dieser neuen Erkenntnis; man hat nur ein paar neue Terme eingeführt und ein paar neue Parameter. Hm.

Immerhin war das der Auftakt zu einer Reihe interessanter Experimente. Damals (wie weitgehend auch heute) bestand die moderne Form des Teilchenexperiments darin, Teilchen aufeinanderzuschießen und die Bruchstücke anzuschauen. Nun gab es verstärktes Interesse an Experimenten, die beschleunigte Teilchen auf geeignete Materie schossen, so dass dabei schön viele hochenergetische Neutrinos erzeugt wurden. Die Neutrinos konnte man dann hunderte oder mehr als tausend km weiter weg mit ganz ähnlichen oder sogar denselben Detektoren nachweisen, wie sie bereits zum Nachweis der Sonnenneutrinos verwendet worden waren. Allerdings mit dem entscheidenden Vorteil, dass man im Unterschied zur Sonne die Erzeugungsparameter direkt beeinflussen konnte.

Eines dieser Experimente ist CNGS,  ausgeschrieben: “Wir schicken Cern-Neutrinos zum Gran Sasso!” Beim CERN steht der für das Experiment eingesetzte Teilchenbeschleuniger (nicht der berühmte LHC, sondern ein Vorgängermodell namens SPS), und im Gran-Sasso-Laboratorium im gleichnamigen Tunnel unter dem gleichnamigen Gebirgsmassiv steht ein Detektor.

Und von eben dort kam gestern (oder so) die Nachricht, die Forscher hätten die mittlere Flugzeit der Neutrinos gemessen und wären auf einen Wert gekommen, der einige hundertstel Promille über der Lichtgeschwindigkeit liegt. Das wäre, wenn es stimmt, eine Revolution – laut der Speziellen Relativitätstheorie ist die Lichtgeschwindigkeit eine absolute Geschwindigkeitsobergrenze, und Teilchen, die sich schneller bewegen (“Tachyonen“) kann man zwar definieren, aber sie haben alle möglichen höchst sonderbaren Eigenschaften; bis auf wenige Ausnahmen dürfte wohl kein Teilchenphysiker erwarten, dass diese hypothetischen Teilchen in der Natur tatsächlich vorkommen.

Hinzu kommt, dass die Geschwindigkeit von Neutrinos bereits vorher schon gemessen wurde –  aus spektakulärem Anlass, nämlich beim Nachweis der ersten außerirdischen Neutrinos, die nicht von der Sonne stammen. Das waren die Neutrinos der Supernova 1987A, und die zumindest haben sich mit großer Genauigkeit als mit ziemlich genau Lichtgeschwindigkeit fliegend herausgestellt – zehntausend Mal genauer als bei den jetzigen Messungen (in Wirklichkeit dürften sich die Neutrinos, da sie ja, s.o., eine Masse besitzen, innerhalb der Genauigkeitsgrenzen ein winziges bisschen unterhalb der Lichtgeschwindigkeit bewegt haben). Freilich, das bliebe als Schlupfloch, war die Energie dieser Neutrinos deutlich geringer als bei den jetzt künstlich erzeugten Neutrinos. Das macht die Erklärung aber eher noch schwerer; die erwähnten merkwürdigen hypothetischen überlichtschnellen Teilchen bleiben immer überlichtschnell und lassen sich nicht auf unterhalb der Lichtgeschwindigkeit abbremsen.

Die Nachricht der vermeintlich überlichtschnellen Neutrinos machte, soweit ich weiss, gestern erstmals die Runde. Heute morgen jedenfalls, auf der Jahrestagung der Astronomischen Gesellschaft in Heidelberg, gab es schon eine Sprecherin, die zwei Mal, als ihre Powerpoint-Präsentation aufgrund technischer Fehler verschwunden war und sie ihren Fachvortrag nicht weiter halten konnte, nach weiterem Konversationsstoff suchend ins Auditorium fragte, ob denn jemand von den überlichtschnellen Neutrinos am CERN gehört habe. (Bevor eine Diskussion aufkommen konnte, funktionierte die Präsentation aber dann jeweils wieder.)

Jedenfalls wissen die CERN-Menschen so gut wie die anderen Physiker, dass solch ein Ergebnis eine große Sensation wäre. Schlecht vorstellbar, dass die überlichtschnellen Neutrinos, wenn sie sich denn bestätigten, nur wieder zu einer disjunkten kleinen Korrektur am Standardmodell der Teilchenphysik führen würden. Wenn, dann dürfte das ganze Gebäude ziemlich in Bewegung geraten. Das ist eine aufregende Vorstellung, denn dann wäre in der fundamentalen Teilchenphysik endlich einmal wieder richtig etwas los – am LHC, dem derzeit größten Teilchenbeschleuniger, geht es ja doch eher langsam voran; man hat das Higgs-Teilchen, das zum Vervollständigen des Standardmodells fehlt, etwas weiter eingekreist als vorher, während die exotischeren möglichen Ergebnisse dieses Teilchenbeschleunigers (Supersymmetrie! Winzige Schwarze Löcher!) erst einmal ausgeblieben sind. Da liest sich die letzte CERN-Pressemitteilung schon ziemlich kläglich, wenn der Sprecher eines Experiments aufgeregt davon spricht, die LHC-Daten hätten ihnen in den letzten Monaten “great strides in our understanding of the Standard Model” erlaubt. Sorry, Leute, aber große Schritte beim Verständnis des Standardmodells sind (a) euch zu gönnen, (b) beim LHC sicher in Zukunft noch möglich, aber (c) sicher nicht das, was ihr da in den letzten Monaten gemacht habt. 

Da wären die überlichtschnellen Neutrinos schon etwas ganz anderes. Allerdings gilt bei solchen Ergebnissen immer: erst einmal ist Vorsicht geboten. Wenn Time Magazine aufgeregt schreibt, dass Wissenschaftler aus aller Welt jetzt aus diesem Anlass Einstein und die Grundlagen der Physik überdenken, dann ist das schlicht Unfug. Ich bin ziemlich sicher, dass derzeit nicht nennenswert mehr Wissenschaftler über Alternativen zur Speziellen Relativitätstheorie nachdenken als vorher. Den Motivationsschub werden beim jetzigen Stand vor allem jene bekommen haben, die sowieso schon Gründe dafür sahen, sich mit solchen Alternativen zu beschäftigen.

Stattdessen dürften sich jetzt viele Wissenschaftler darüber Gedanken machen, wie und was die CERN-Forscher da eigentlich gemessen haben. BBC News hat da, nicht überraschend, eine realistischere Einschätzung. Die Kollegen vom CERN sind jedenfalls nicht in die Falle getappt, ihre Ergebnisse jetzt schon als physikalische Revolution zu präsentieren, sondern sie machen es genau richtig: “Wir haben etwas merkwürdiges gemessen,  wir haben alles, was uns an möglichen Fehlerquellen einfiel, so gut wir konnten überprüft; liebe Kollegen: bitte helft uns, herauszufinden, was da eigentlich los ist.” Das war der Tenor der Äußerungen der Gruppe, und den ersten Ansatzpunkt für ihre Kollegen haben sie gestern in Form eines E-Prints, also eines noch nicht in der qualitätsgeprüften Fachliteratur veröffentlichten elektronischen Artikels, öffentlich gemacht. Zugriff auf die Daten, wenn schon nicht für alle, dann zumindest für einige Wissenschaftler außerhalb der Kollaboration dürfte folgen.

Dann man los – irgendwo muss die Diskrepanz ja stecken. Beim letzten ähnlich gelagerten Fall, der Pioneer-Anomalie, war die Auflösung zumindest keine physikalische Revolution. Seit den 1980er Jahren hatte es so ausgesehen, als ob die Raumsonden Pioneer 10 und 11 (gestartet 1972 bzw. 1973, schöne Bilder und Daten v.a. von Jupiter, Saturn) etwas anders durch das Sonnensystem drifteten, als den gängigen physikalischen Theorien nach zu erwarten. Dort war dann, nachdem eine erste Runde von Prüfungen keine herkömmliche Erklärung gefunden hatte (ausströmendes Gas, thermische Abstrahlung, was eine Sonde eben so machen kann) tatsächlich der Fall eingetreten, dass eine Reihe von Wissenschaftlern begann, sich aus diesem Anlass tatsächlich Gedanken über Alternativen zur herkömmlichen Gravitationstheorie, Einsteins Allgemeiner Relativitätstheorie, zu machen. Aber 2011 fanden einige Forscher dann doch eine überzeugende Lösung, die die Grundlagen der Physik unangetastet ließ (vgl. hier und hier):  Die ursprünglichen Erklärungsversuche hatten die Wärmestrahlung (Quelle vor allem: die Radionuklidbatterie an Bord) nicht realistisch genug modelliert; ging man genauer vor, verflüchtigte sich die Anomalie (vgl. diesen Blogbeitrag von Michael Khan).

Insofern: So spannend es wäre, wenn dies der Anfang eines Neuarrangements der Grundlagen der Teilchenphysik wäre (immer mit der Hoffnung, dass das Ergebnis schöner ist als das derzeitige Standardmodell): Weit wahrscheinlicher ist im jetzigen Stadium irgendein vergleichsweise banaler technischer Fallstrick. Irgendetwas bei den Triggern und Timern, die verwendet werden, um Erzeugung und Nachweis einen Zeitpunkt zuzuordnen; bei der Auswertungssoftware; bei der Verbindung zum GPS-System; bei der Vermessung der Entfernung von Erzeugungs- und Nachweispunkt; bei der Statistik, mit der die erzeugten und gemessenen Ergebnisse einander zugeordnet werden und die Flugzeit berechnet wird. 

Entweder geht die Aufklärung vergleichsweise schnell, oder wir haben über die nächsten Jahre hinweg eine “CERN-Neutrino-Anomalie”.  Bleibt die Anomalie lange genug bestehen, wird es weitere Experimente auch anderer Forschungsgruppen geben (am Fermilab in Chicago und in Japan gäbe es entsprechendes Gerät). Spätestens falls dann noch andere Gruppen ungefähr das Gleiche messen sollten, dürften sich in der Tat viele Theoretiker der Frage zuwenden, wie man die Grundstruktur unserer teilchenphysikalischen Theorien an die neue Datenlage anpassen kann. Oder eben es läuft wie bei der Pioneer-Anomalie – Jahre an Forschung, viele theoretische Ansätze und keiner davon der Durchbruch, und 30 Jahre später kommt die Forschergruppe, die doch noch eine einfache technische Erklärung findet.

Neue Physik wäre jedenfalls die spannendste Entwicklung; eine Erweiterung dessen, was wir wissen, oder ein ganz neuer Blickwinkel, der zeigt, warum die jetzigen Teilchentheorien (samt ihrer Grundlage, der Speziellen Relativitätstheorie) von vielen hochgenauen Experimenten so gut bestätigt werden, während hochenergetische Neutrinos etwas ganz anderes machen. Meine Daumen sind gedrückt, aber ich bin ziemlich pessimistisch. Schaun mer mal, wie es weitergeht.

 

Update 23. Februar 2012:

Ein Problem bei der Kabelverbindung zum GPS und ein defekter Taktgeber sind als mögliche Fehlerquellen im Gespräch.

 

 

Avatar-Foto

Markus Pössel hatte bereits während des Physikstudiums an der Universität Hamburg gemerkt: Die Herausforderung, physikalische Themen so aufzuarbeiten und darzustellen, dass sie auch für Nichtphysiker verständlich werden, war für ihn mindestens ebenso interessant wie die eigentliche Forschungsarbeit. Nach seiner Promotion am Max-Planck-Institut für Gravitationsphysik (Albert-Einstein-Institut) in Potsdam blieb er dem Institut als "Outreach scientist" erhalten, war während des Einsteinjahres 2005 an verschiedenen Ausstellungsprojekten beteiligt und schuf das Webportal Einstein Online. Ende 2007 wechselte er für ein Jahr zum World Science Festival in New York. Seit Anfang 2009 ist er wissenschaftlicher Mitarbeiter am Max-Planck-Institut für Astronomie in Heidelberg, wo er das Haus der Astronomie leitet, ein Zentrum für astronomische Öffentlichkeits- und Bildungsarbeit, seit 2010 zudem Leiter der Öffentlichkeitsarbeit am Max-Planck-Institut für Astronomie und seit 2019 Direktor des am Haus der Astronomie ansässigen Office of Astronomy for Education der Internationalen Astronomischen Union. Jenseits seines "Day jobs" ist Pössel als Wissenschaftsautor sowie wissenschaftsjournalistisch unterwegs: hier auf den SciLogs, als Autor/Koautor mehrerer Bücher und vereinzelter Zeitungsartikel (zuletzt FAZ, Tagesspiegel) sowie mit Beiträgen für die Zeitschrift Sterne und Weltraum.

129 Kommentare

  1. Viel Rauch um Nichts.

    Mal wieder viel Rauch um nichts, nur Aktionismus und Politik.
    Ich zitiere hier mal den letzten Absatz eines Artikels eines Verfassers, der 1915 in der Zeitschrift: “Die Kultur der Gegenwart” auf Seiten 703–713 bezüglich der Relativitätstheorie erschienen ist.
    Zitat:
    “Der Verfasser dieser Zeilen ist aber der Ansicht, dass die Relativitätstheorie noch einer Verallgemeinerung bedarf, in dem Sinne, dass das Prinzip von der Konstanz der Lichtgeschwindigkeit fallen zu lassen ist. Nach dieser Meinung ist jenes Prinzip nur für Gebiete von praktisch konstantem Gravitationspotential aufrecht zu erhalten. Die Zukunft muss lehren, ob diese in der Hauptsache auf erkenntnistheoretische Gründe sich stützende Ansicht sich bewähren wird.”

  2. Ausgezeichnete Zusammenfassung

    Hallo,

    Dieser Abriss der Vorereignisse und aktuellen Geschehnisse ist hervorragend!

    Ich wuenschte, ein solches recherchetechnisches und sprachliches Niveau waere bei den Tagesmedien gegeben: Wir wuerden in einer weitaus besseren Welt leben! ;(

    Bye, Marc

  3. Mehr Neutrinos: Neutrionfabriken

    Neutrinos sind immer wieder für eine Überraschung gut. Möglicherweise sind sie ja auch ihre eigenen Antiteilchen, eine Vermutung, der das GERDA-Experiment nachgeht.

    Aber allein schon Neutrinooszillationen sind ein wichtiges Fenster in die
    Physik jenseits des Standartmodells.

    Praktisch wären hochgradig kollineare Neutrinostrahlen, wie sie von Neutrinofabriken erzeugt werden. Dabei nutzt man den Myonenzerfall auf dem geraden Teil eines Speicherrings aus. In 100 m Abstand sollen so mehr als 1 Million mehr Neutrinos zur experimentellen Verfügung stehen als mit bisherigen Quellen.

    Vielleicht kommt das seltsame Ergebnis mit den überlichtschnellen Neutrinos den Neutrinoforschern ja gelegen. Antonio Ereditato vom Opera-Experiment meint jedenfalls “Meine erste Reaktion: Neutrinos verblüffen uns noch immer mit Geheimnissen.”

    Mit dem Design von Neutrino-Fabriken beschäftigt sich jedenfalls auch das CERN (neben Forschungsgruppen in den USA und in Japan).

  4. @Euro vs. DM

    Naja, gaaaanz so einfach ist es (außerhalb der Blogkommentarosphäre) dann doch nicht. Aber, wie auch geschrieben: bevor man schaut, welche Änderungen nötig sind, um die Ergebnisse mit dem Standardmodell der Elementarteilchenphysik, zu bringen (das ja auch umfangreich erfolgreich getestet wurde; das macht es so schwierig), ist erst einmal genaue Prüfung angesagt.

  5. Eine Frage drängt sich hier bei mir auf.

    (a) waren die Neutrinos bei ihrer Erzeugung langsamer als das Licht und wurden dann auf Überlichtgeschwindigkeit beschleunigt?

    (b) waren sie schon bei ihrer Geburt schneller als das Licht?

    (c) das Ganze beruht auf einem Messfehler?

    Fall (a) geht ja nach der SRT gar nicht und zwar deswegen, weil unendlich viel Energie für die Beschleunigung notwendig wäre, das kann man anhand dieser Formel zeigen (b=wurzel(1-v²/c²))

    E=m*c²*(1/b-1)

    Und jetzt kommt eine schöne Matheaufgabe für die Mittelstufe! Weise nach dass man obige Gleichung in folgende Form überführen kann.

    E=m*v²/(b²+b)

    Da haben wir also unter dem Bruchstrich 2 Summanden, die sich bei unseren alltägliche Schneckentempi zu 2 ergänzen, führt man die Teilchen allerdings an c heran, dann haben wir unterm Strich die 0 und damit unendlich viel Energie, die natürlich kein Teilchenbeschleuniger des Universums liefern kann!

  6. Hervorragend

    Herrn Ruef kann ich nur voll zustimmen. Und Sie schaffen es einen fesselnden Bericht aus diesem für Laien eigentlich schwierigen Thema zu machen.
    Wenn ich lese in welchen Extrembereichen noch mit hoher Messgenauigkeit gearbeitet wird, und versuche mir die technischen Apparaturen dazu vorzustellen, soweit meine Phantasie die Bilder aus Netz und anderen Medien integrieren kann, verbleibe ich mit großen Augen und offenem Mund.
    Jeder weiß wie schwierig es ist, alle Fehlerquellen auch bei recht einfachen Prozessen zu berücksichtigen. Skepsis versteht sich von selbst. Aber es ist einfach phantastisch was schon geht.

  7. Überpünktliche Geschwindigkeit?

    Markus Pössel schrieb (23. September 2011, 23:50):
    > […] die Nachricht, die Forscher hätten die mittlere Flugzeit der Neutrinos gemessen

    … die mittlere Dauer von Elementen des Versuchsaufbaus (SPS, Gran-Sasso-Laboratorium) während der sie jeweils ein Neutrino untereinander austauschten;
    nicht die mittlere Dauer jeweils eines Neutrinos von Erzeugung/Abflug am SPS bis Ankunft/Wechselwirkung im Gran-Sasso-Laboratorium …

    > und wären auf einen Wert gekommen, der einige hundertstel Promille über der Lichtgeschwindigkeit liegt.

    Werte von “Flugzeit” bzw. “Dauer” und Werte von “Geschwindigkeit” sind nicht kommensurat …

    (Die Forscher haben auch den Abstand zwischen den betreffenden Elementen des Versuchsaufbaus abgeschätzt, den Quotienten aus diesem Abstands-Schätzwert und der gemessenen Austauschdauer errechnet, und sind erst so auf einen Wert gekommen, der einige hundertstel Promille über der Lichtgeschwindigkeit liegt.)

    > Stattdessen dürften sich jetzt viele Wissenschaftler darüber Gedanken machen, wie und was die CERN-Forscher da eigentlich gemessen haben.

    Hoffentlich nicht nur darüber, wie und was die CERN-Forscher da eigentlich gemessen haben;
    sondern auch darüber, was jene gemessen haben (wollen), die meinen, dass “die […] Spezielle[] Relativitätstheorie von vielen hochgenauen Experimenten so gut bestätigt” oder ggf. auch widerlegt werden könnte,
    statt sie als Grundlage insbesondere für die Beurteilung der “(Hoch-)Genauigkeit” von Experimenten zu benutzen und vorauszusetzen.

  8. Per Anhalter durch die Galaxis

    Ein sehr spannender Artikel und wie schon RD bemerkte:

    “Wenn ich lese in welchen Extrembereichen noch mit hoher Messgenauigkeit gearbeitet wird, und versuche mir die technischen Apparaturen dazu vorzustellen, soweit meine Phantasie die Bilder aus Netz und anderen Medien integrieren kann, verbleibe ich mit großen Augen und offenem Mund.”

    Ich bin gespannt, ob sich das Ganze schnell in Schall und Rauch aufloest oder wir uns noch eine Weile mit der “CERN-Neutrino-Anomalie” beschaeftigen werden.

  9. @W.G.

    wenn die da nun wirklich irgendwas revulotinärem auf der spur sind, geht es doch genau darum, dass die spezielle relativitätstheorie für die neutrinos nicht zwingend gelten muss. erscheint mir irgendwie schwer, das denn irgendwie in ne argumentation dagegen einzubringen. oder? 🙂 ich hab halt auch keine ahnung 😀

  10. Danke Dir

    Kann da Marc (u.a.) nur zustimmen.
    Auf diesem Niveau wünscht man sich das in der Süddeutschen etc auch mal.
    Vielen Dank Markus.

    Ps. Das nichts schneller sein kann als Licht, habe ich noch nie geglaubt, aber ich denke doch wohl, dass sich irgendwann ein Messfehler herausstellen wird.
    Schaun mer mal.

  11. Alternative zu Tachyonen

    Ich weise auf diesen interessanten Artikel hin:

    http://arxiv.org/abs/astro-ph/9505117v1

    Darin wird aufgezeigt, dass die Minkowski-Geometrie von Teilchenbewegungen (= relativistisches Verhalten wie Lorentz-Invarianz) sich nicht nur als Eigenschaft der Raum-Zeit interpretieren lässt, sondern auch als Bestandteil der Bewegungsgleichungen für Teilchen. Teilchen mit abweichenden Bewegungsgleichungen könnten sich im gleichen Raum-Zeit-Rahmen aufhalten, sich aber in einem anderen Minkowski-Rahmen bewegen (z.B. mit einer von c unterschiedlichen Grenzgeschwindigkeit). Diese Teilchen wären KEINE Tachyonen, hätte “normale” (positive) Masse und könnten sich auch langsamer als c bewegen (aber eben auch schneller). Einstein bleibt auf dem Sockel, aber wir hätten zwei Minkowski-Rahmen.

    Natürlich ist das alles hypothetisch. Natürlich können die OPERA-Ergebnisse auch Messfehler sein. Das muss geprüft werden. Aber ich wäre sehr vorsichtig, die OPERA-Ergebnisse allein aus theoretischen Erwägungen abzutun.

  12. @Peer Schaefer

    Das wäre, wenn sich die Neutrino-Anomalie bestätigt, vielleicht eine mögliche Richtung. Ich habe in dem Preprint jetzt nichts nachgerechnet, aber auf den ersten Blick wäre das ein Beispiel für einen neuen Blickwinkel wie im letzten Absatz angedeutet.

    Eine Korrektur meinerseits aber dann doch noch: Es geht ja nicht “nur” um theoretische Überlegungen, sondern auch um die erwähnten Supernova-Messungen an Neutrinos. Dafür, dass Neutrinos je nach Energie von einer der kritischen Geschwindigkeiten zur anderen springen könnten, finde ich auch in dem erwähnten Preprint keinen Anhaltspunkt – aber, wie gesagt, ich denke, dass es für solche Überlegungen noch deutlich zu früh ist. Schauen wir erst einmal, ob alle Bausteine der CERN-Messungen der kritischen Nachprüfung standhalten.

  13. Supernova 1987N

    Superluminale Teilchen, die keine Tachyonen sind sondern sich mit positiver Masse aber in einem abweichenden Minkowski-Rahmen mit einer anderen Grenzgeschwindigkeit (>c) bewegen, würden bei Bewegung im Vakuum Tscherenkow-Strahlung abgegeben und durch den Energieverlust abbremsen. Bei kurzen Strecken (wie z.B. die 730km vom CERN zu OPERA) mag dies keine Rolle spielen, bei langen Strecken (wie z.B. die 160.000 Lichtjahre von 1987N) dürfte dies aber erheblich sein. Vielleicht ist es daher ein Kurzstrecken-Phänomen.

    Aber hinsichtlich der Daten von OPERA haben Sie natürlich völlig Recht: Wir müssen jetzt abwarten. Wenn sich kein systematischer Fehler finden lässt, muss das Experiment andernorts von einem anderen Team nachgebaut werden. Das kann Jahre dauern. Immerhin: es tut sich etwas Spannendes!

  14. @Peer Schaefer

    Zur Tscherenkow-Strahlung: Neutrinos sind ja nun elektrisch neutral; insofern bekommt man da keine richtige elektromagnetische Tscherenkow-Strahlung.

    Zum Nachbau der Experimente: Es gibt ja sowohl beim Fermilab MINOS und T2K am KEK schon entsprechende Experimente; da müsste wohl allenfalls die Zeiterfassung hinreichend präzise gemacht werden und die Entfernung zwischen Target und Detektor genau genug bestimmt werden. Und, richtig, wahrscheinlich Monate bis Jahre Ereignisse gemessen werden.

  15. The trouble with distance

    Relativistische Distanzbestimmung ist eine heikle Angelegenheit. Speziell dann, wenn die zu messende Strecke innerhalb Erdkruste verläuft, wo keine direkte Messung möglich ist. Dass dessen ungeachtet die Baseline Länge bis auf +/-20cm exakt ermittelt worden sein soll, das ist dann doch schon irgendwie suspekt.

    Aus der OPERA Dokumentation wird aber nicht einmal klar, ob die Baseline Länge überhaupt eine relativistische Distanz ist.
    http://operaweb.lngs.infn.it/…cnotes/note132.pdf

    Das sieht erst einmal verdächtig danach aus, dass hier tatsächlich nur eine geodätische Koordinatendistanz angegeben wird. Wenn dem so ist, dann sind die +/-20cm allerdings glaubhaft. Aber es wäre eine Pseudo-Distanz, die unbedingt noch relativistisch korrigiert werden müsste — und zwar nach unten. Und dazu habe ich leider keine Angaben gefunden… habe ich da etwas übersehen?

  16. Tscherenkow-Strahlung

    Sie haben Recht. Echte Tscherenkow-Strahlung kann es nicht geben. Der o.g. Artikel beschreibt allerdings die Emission “tscherenkow-ähnlicher” Strahlung in Form von unterlichtschnellen Teilchen. Auch dadurch würde ein Energieverlust (und Geschwindigkeitsverlust) eintreten. Ich hätte Anführungszeichen setzen müssen.

    (Natürlich alles hochspekulativ. Ebenso wie Tachyonen. Ebenso wie die Daten des OPERA-Experimentes… Aber denkbar und bedenkenswert.)

  17. adenosine

    Die Abweichung ist ja nur sehr klein. Die die Lichtgeschwindigkeit ist ja nur eine empirisch ermittelte Konstante. Ist es nicht auch möglich, dass die absolute Maximalgeschwindigkeit etwas größer ist, als wir hier unter unseren Bedingungen gemessen haben und es Effekte gibt die das hier gemessene Licht verzögern? Vielleicht sind die Neutrinos nur näher dran.

  18. @Chrys

    Ich hab’s jetzt nicht durchgerechnet, aber auf den ersten Blick scheint es mir nicht so, als könnten sich da Korrekturen in der gemessenen Größenordnungen ergeben. Ich kann ja z.B. beim CERN im Moment der Neutrinoerzeugung ein frei fallendes Inertialsystem einführen; das ist in den 2 Mikrosekunden, nach denen die Neutrinos im Gran Sasso Lab ankommen, gerade einmal 30 Mikrometer oder so gefallen. Und in diesem Inertialsystem gilt laut Äquivalenzprinzip in guter Näherung die Spezielle Relativitätstheorie; dass da relativistische Korrekturen von 20 cm o.ä. dazu kommen könnten, sehe ich aus diesem Grunde nicht.

  19. Mögliche Fehler im Experiment

    In einem Artikel auf “The Reference Frame” (Blog eines tschechischen Stringtheoretikers) werden verschiedene Fehlerursachen beschrieben, die den Neutrino-Vorsprung von 60 Nanosekunden bzw. 18 Metern wegerklären könnten. Z.B. sind im GPS möglicherweise Fehler enthalten, die vielleicht bisher nur von niemandem bemerkt wurden.
    http://motls.blogspot.com/…n-opera-research.html

    In einem weiteren Artikel spekuliert er übrigens, wie die Ergebnisse (falls sie wider erwarten doch keine Messfehler sind) von der Stringtheorie erklärt werden könnten.
    http://motls.blogspot.com/…l-neutrinos-from.html
    Das dürften aber wohl nur Stringtheorie-Fachleute verstehen…

  20. @M. Schlederer

    Was Lubos Motl da zum GPS schreibt, finde ich nicht überzeugend. So wie ich Geometer kenne, werden die seit der Einführung von GPS sehr genau geprüft haben, dass GPS und herkömmliche Vermessung z.B. mit Hilfe von Triangulation übereinstimmen. Und Distanzen von 730 oder so km konnte man ja auch vor GPS schon sehr präzise vermessen! Dass da 18 Meter einfach so verlorengehen, halte ich für sehr unwahrscheinlich.

    Dass man auch die Distanzmessung nachprüfen sollte, möglichst mit Gegenproben, steht außer Frage. Mein Bauchgefühl (das in Experimentalphysik aber nicht allzu erfahren ist) geht in Richtung (a) Fehlerquellen bei den diversen elektronischen Bauteilen, (b) Behandlung der systematischen Fehler (sind die wirklich alle symmetrisch?) und (c) Zuordnungsverfahren von den erzeugten zu den nachgewiesenen Neutrinos (wieder inklusive Behandlung der Ortsunsicherheit).

  21. Was soll der Aufruhr eigentlich? …

    Chrys schrieb (25.09.2011, 11:41):

    > http://operaweb.lngs.infn.it/…cnotes/note132.pdf

    Prima Recherche!
    Besonders bequem und bemerkenswert finde ich dort die

    “Table 6: OPERA system positions of the neutrinos beam start point at CERN and the OPERA detector point at LNGS [provided for additional convenience]” —
    { x, y, z } :=
    { 3177.974 m, 729297.439 m, -42378.794 m }.

    .

    In diesem Zusammenhang:
    wie groß ist eigentlich der (nominelle) Höhenunterschied (z.B. “über NN”) zwischen “LNGS” und “CERN”?
    Wie genau ist die (ohne viel Aufwand zu erhaltende) Abschätzung
    “980 m – 422 m”? …

    > Pseudo-Distanz, die unbedingt noch relativistisch korrigiert werden müsste […]

    Da “LNGS” und “CERN” (sicherlich) gar nicht zueinander ruhten, sondern (bestenfalls) zueinander starr waren, ist eine SRT-gemäße, bilateral einvernehmliche Distanzbewertung sowie gar nicht strikt anwendbar.

    Die unmittelbaren und jedenfalls nachvollziehbar definierten (aber eben experimentell nicht aus unmittelbaren Beobachtungen zu gewinnenden) Messgrößen wären allenfalls:

    – die (kürzeste) Pingdauer des CERN (zu LNGS, und zurück),

    – die (kürzeste) Pingdauer des LNGS (zu CERN, und zurück), sowie

    – die Dauer des CERN von der Anzeige einer “extraction” bis zur Anzeige der (ersten) Wahrnehmung der entsprechenden “interaction” mit LNGS, und

    – die Dauer des LNGS von einer bestimmten Signalanzeige bis zur “interaction“-Anzeige für Neutrinos, deren “extraction” CERN koinzident mit dem Signal wahrnahm.

  22. @Markus Pössel

    Diese ca. 30 Micrometer, das wäre jetzt ein Argument dafür, warum es kein nennenswerter Fehler ist, die Baseline als einen Geradenabschnitt anzusehen. Da stimme ich zu, das habe ich aber auch nicht gemeint.

    Gemeint war tatsächlich, dass hier nicht relativistisch mit einer “ping distance” (“radar distance”, if you prefer) gerechnet wurde. Was man mit diesem ETRF2000 Web Interface als Entfernung erhält, ist jedenfalls keine “ping distance”, sondern nur ein Koordinatenabstand. Eine Abschätzung für die daraus resultierende Differenz wird nirgends präsentiert.

    Kann sein, dass diese Differenz sich als unerheblich erweist, ich kann das so nicht abschätzen. Aber es ist alles andere als selbstverständlich, dass man hier stillschweigend die Geometrie auswechseln darf, da braucht es eine solide Begründung. Eine solche sehe ich nicht, was auch mein Fehler sein kann. Andernfalls wäre die OPERA Entfernungsbestimmung aber allemal eine Schwachstelle, wo man vorzüglich die Axt anlegen kann.

    Signifikante Fehler im GPS oder im gedodätischen Referenzmodell sind eher recht unwahrscheinlich. Da hat es schlicht zu viele Anwender, um nicht anderweitig ebenfalls aufzufallen.

  23. Wer andern eine Gräbe grubt, meist selb.

    Chrys schrieb (26.09.2011, 10:25):
    > Signifikante Fehler im GPS oder im gedodätischen Referenzmodell sind eher recht unwahrscheinlich. Da hat es schlicht zu viele Anwender, um nicht anderweitig ebenfalls aufzufallen.

    Anwender mit Sensitivität “(Δ Pingdauer) / Pingdauer” im ppm Bereich ?

    Unterm Mont Blanc ??

    (Na gut: vorstellbarer Weise eben “drumherum”; unter Berücksichtigung bzw. Annahme der passenden Cayley-Menger-Determinanten …)

  24. Klasse Artikel! Respekt und Danke!

    Guten Tag,

    seit dem ich mich sehr intensiv mit der Quantenmechanik und im Besonderen mit der QFT beschäftige, hielt ich es nicht mehr für unvorstellbar, dass C überschritten werden kann. Der Meinung wäre ich auch dann noch, wenn sich die Messung des N als Fehler herausstellen würde.
    Wenn man es genau durchdenkt kommt man zu dem Schluss, dass C eigentlich keine unabhängige Eigengeschwindigkeit ist. Eine Pistolenkugelgeschwindigkeit ist z. B. vom Luftwiederstand, der Gravitation u. s. w. abhängig.
    Ein Photon ist jedoch nichts Anderes, als eine Energiewirkung, welche durch einen auslösenden Impuls seine Existent begann und im Vakuum in Abhängigkeit von den durch Vakuumfeldfluktuationen entstehenden virtuellen Elektron-Positronpaaren steht. Man könnte nun im Kehrwert der Formel herleiten, dass die Entstehungsgeschwindigkeit der Elektron-Positronpaare und der vorausgegangene Impuls zur Entstehung der Elektromagnetischen Welle die wesentlichen Faktoren für C sind. Andere Parameter wie z. B. Ladung und Impuls könnten bezüglich der Vakuumpolarisation auch maßgebend für höhere Geschwindigkeiten sein. Wenn ein anderer Impuls u. s. w. das Quantenfeld nun anders beeinflussen würde, wäre auch eine Geschwindigkeit über C vorstellbar und C wäre somit niemals die höchste mögliche Geschwindigkeit für eine Informationsübertragung gewesen. Sie wäre es nur für jene elektromagnetischen Wellen gewesen, die diesen Impuls haben und somit das Quantenfeld entsprechend zur Erzeugung der Paarbildung stimulieren.
    Es würde dann auch nicht mehr das Problem der unendlichen Masse existieren, da sich die Formel entsprechend verändern würde.
    Wir hätten in diesem Fall bislang nur elektromagnetische Wellen gefunden, welche die bis vor kurzen schnellste Geschwindigkeit C zeigten. Leider wäre die ART dann falsch, da C keine Naturkonstante mehr an für sich wäre, sondern lediglich für elektromagnetische Wellen. Licht könnte zwar weiterhin auch niemals Licht überholen, doch man müsste davon ausgehen, dass das Quantenfeld gewisse Impulsausgangsfrequenzen und die Bildung der Virtuellen – Teilchenpaare entsprechenden Einfluss auf Geschwindigkeiten haben, wenn genannte Parameter diese begünstigen. Somit könnten andere Erscheinungsformen das Quantenfeld günstiger beeinflussen, als es elektromagnetische Wellen tun. Das eröffnet für die Forschung immense Fenster! Auf der Ebene von Geschwindigkeiten bezüglich ruhemasselosen oder ruhemassebesitzenden Teichen/Wellen würde man gewiss auch einer vereinheitlichten Formel bezüglich Quantenmechanik u. s. w. und den Aussagen der ART näher kommen. Doch das sollen die richtig schlauen Leute machen, ich mach mir jetzt Pfannenkuchen.

    Netter Gruß

    Saskia-Nene

  25. @Frank Wappler

    Aber 18 Meter Abweichung bei einer simplen GPS Positionsbestimmung — das würde schwerlich sehr lange verborgen bleiben können. Auch wenn nicht alle Anwender hochsensibel sind.

    Im OPERA Normalbetrieb wird GPS offenbar zur UTC-Synchronisierung der beiden lokalen Atomuhren eingesetzt. Auch da kann eigentlich nicht viel schiefgehen, ohne dass es rasch auffällt. An GPS liegt’s höchstwahrscheinlich nicht.

  26. Da stelle mer ons erstmal janz dumm …

    Chrys schrieb (26.09.2011, 13:58):
    > Aber […]

    Zugegeben: meine Skepsis ist nicht zuletzt eine Ausgeburt meiner gepflegten Naivität.

    (Irgend jemand sollte die angemessene Naivität doch hoffentlich aufbringen — dort, wo sie sich eventuell auszeichnen kann? …)

    Haste denn schon mal den “Höhenunterschied zwischen LNGS und CERN” überschlagen, unter Verwendung der Angaben in “Table 6“? …

  27. @Frank Wappler

    Na klar, der Höhenunterschied CERN/LNGS erfordert einige Vorkehrungen zur Time Of Flight Messung — die Synchronität der Uhren muss gegeben sein. Dieses Problem soll ja durch die GPS UTC-Synchronisierung erschlagen werden, und wenn das ordentlich gemacht wird… aber keine Dummheit ist zu abwegig, um nicht irgendwann von irgend jemandem begangen zu werden.

    Beim aktuellen Stand der Dinge sehe ich jedenfalls absolut keinen Anlass, hysterisch zu werden oder gar die GR in Zweifel zu ziehen.

  28. Überlichtschnelle Neutrinos?

    Danke für diesen Artikel!
    So einen findet man eher selten:
    1. Alles verständlich
    2. Unterhaltsam
    3. Keine Marktschreierei
    4. Physikalisch korrekt
    Ich fühle mich gut informiert

  29. 20 Meter erfordern einigen Hirnschmalz

    Chrys schrieb (26.09.2011, 18:53):
    > Na klar, der Höhenunterschied CERN/LNGS erfordert einige Vorkehrungen zur Time of Flight Messung

    Das auch, na sicher.
    Mir geht’s aber (so weit sich das überhaupt zumindest gedanklich trennen lässt) ganz konkret um “die berühmten 20 Meter”.
    Und dabei scheint der Höhenunterschied schlicht wichtig und interessant zu sein, denn (z.B.):

    Sqrt[ 730^2 + 0.5^2 – 2 * Cos[ Pi/2 + Pi * 730 / 40000 ] * 730 * 0.5 ] = 730.029
    .

    Und wenn ich nun anhand von “Table 6” aus http://operaweb.lngs.infn.it/…cnotes/note132.pdf mal sehr naiv und überschlägig rechne

    6370000 – Sqrt[ (6370000 – 42378.794)^2 + 729297.439^2 + 3177.974^2 ]
    = 488.639,

    und andererseits lese
    http://it.wikipedia.org/…azionali_del_Gran_Sasso

    Entrambe ad una quota sul livello del mare di circa 1000 m, la parte sotterranea è coperta da 1.400 m di roccia del massiccio del Gran Sasso d’Italia

    sowie (z.B.)
    http://de.wikipedia.org/wiki/Meyrin

    Höhe: 435 m ü. M.

    ,

    … sollte man wohl eher

    6380000 – Sqrt[ (6380000 – 42378.794)^2 + 729297.439^2 + 3177.974^2 ]
    = 554.303

    rechnen (?) …

    > absolut keinen Anlass […] die GR in Zweifel zu ziehen.

    Eben; und gerade deshalb allen Grund, jene anzuzweifeln, die mit Koordinaten hantieren (oder gar mit Koordinaten zu argumentieren versuchen), anstatt mit Pingdauern.

  30. @Frank Wappler

    Dem stimme ich voll und ganz zu, “[…] und gerade deshalb allen Grund, jene anzuzweifeln, die mit Koordinaten hantieren (oder gar mit Koordinaten zu argumentieren versuchen), anstatt mit Pingdauern.

    Nach meinem Eindruck, leider nur wenige Autoren behandeln die Problematik der Distanzmessung so klar und akzentuiert, dass es in den Köpfen überhaupt ankommt und etwas davon hängenbleibt. Als rühmliche Ausnahmen fallen mir dabei John Synge und George Ellis ein.

    Synges Relativity Books dürften heutzutage fast ausschliesslich als Staubfänger in Bibliotheken dienen. Das halte ich für sehr bedauerlich.

  31. Complete guide to the ghostly particle

    Eine Sammlung von populärwissenschaftlichen Artikeln über Neutrinos findet sich
    hier unter dem Titel Neutrinos: Complete guide to the ghostly particle
    mit den Aritkeln:

    – Dimension-hop may allow neutrinos to cheat light speed

    – Faster-than-light neutrino claim bolstered

    – Neutrinos caught ‘shape shifting’ in new way

    – Sterile neutrino back from the dead

    – Indian neutrino lab to boast world’s biggest magnet

    – Anti-neutrino’s odd behaviour points to new physics

    – First ‘chameleon particle’ spotted after changing type

    – Neutrinos: The key to a theory of everything

    – At last, a way to test time travel

  32. Arxiv on OPERA/Superluminal Neutrinos

    120 wissenschaftliche Artikel zu Opera und speziell zu den behaupteten superluminalen Neutrinos finden sich auf dem Arxiv-Server.

    Man findet alle möglichen Erklärungsansätze, von statistischen Fehlern bis zur Physik von schwarzen Löchern und ihre Verallgemeinerung auf die normale Raum/Zeitphysik

    Neutrino Shortcuts in Spacetime

    Relativistic Superluminal Neutrinos (Erklärung mit einem Skalarfeld in der Erde)

    Supersonic Velocities in Noncommutative Acoustic Black Holes (von SchwarzLoch-Physik zu superluminalen Teilchen im allgemeinen)

    Schön wie hier Wissenschaft und Science-Fiction zusammenfinden.

  33. Hat mir gefallen

    Hallo,

    sehr sehr guter Artikel!
    Ich bin zwar physikalisch nicht ganz unbeleckt, aber wenn es so tief in die Materie geht, erfreue ich mich auch, wenn ein Mann vom Fach dieses komplexe Thema leienhaft darstellt.

    MfG
    Mario

  34. OPERA Berechnungsfehler wg. Erdrotation?

    Die Arbeit The OPERA neutrino velocity result and the synchronisation of clocks dürfte vielleicht einige Kommentatoren hier interessieren. Sie behauptet, die Neutrino-Geschwindigkeitsbestimmung im OPERA-Experiment (CNGS) habe nicht berücksichtigt, dass das Experiment – senden von Neutrinos vom CERN zum LNGS – nicht in einem Inertialsystem stattfand: Nur ein reflektiertes Signal wäre immun gegen den Einfluss der Erdrotation. Vor allem der Kommentator Chrys hat sich in eine ähnliche Richtung geäussert, wobei er aber fälschlicherweise annahm, die Varianz des Gravitationsfelds in der Verbindungslinie CERN – Gran Sasso spiele eine entscheidende Rolle.

  35. GPS-Genauigkeit

    Aber 18 Meter Abweichung bei einer simplen GPS Positionsbestimmung — das würde schwerlich sehr lange verborgen bleiben können

    Nö.
    http://www.scienceblogs.de/…e-physik-am-ende.php

    Benutzt man ein falschen Radiuswert für die Erde, ändert sich an der Präzision nichts. Man bekommt immer noch die korrekte Koordinate, nur leider zu tief.

    Frank Wappler hat ja schon bereits Kritik an der Höhenmessung geäußert. Obwohl das nicht ganz so einfach ist, würde ich tatsächlich die Höhenwerte überprüfen.

  36. Is it solved? / @Martin Holzherr

    Interessant ist schon, dass R.A.J. van Elburg in seiner Rechnung ziemlich genau auf die obskuren 60 ns kommt. Allerdings schreibt er da (http://arxiv.org/abs/1110.2685, p.3):

    The clocks in the OPERA experiment are orbiting the earth in GPS satellites.

    Nun, nach dem OPERA Preprint sind die fraglichen Uhren erdfest. Und GPS kommt nur zum Einsatz, um diese Uhren auf die gemeinsame Zeitskala UTC zu synchronisieren, die Time Of Flight wird demnach stets in UTC-Sekunden ausgedrückt.

    Eine solche UTC-Synchronisierung mittels GPS ist doch eigentlich eine Standardaufgabe. Dass dabei die Schweizer (METAS) wie die Deutschen (PTB) unabhängig voneinander praktisch denselben amateurhaften Bug produziert haben sollten, das wäre dann äusserst befremdlich.

  37. Physik ein Fake

    Ob die Neutrinos schneller sind als Licht oder nicht ist eigentlich nicht die wichtigste Frage. Viel wichtiger ist zu beweisen dass sie tatsächlich gibt.

    Ist nämlich nicht der Fall dann ist die ganze Teilchenphysik Schrott. Die so angeblich Wissen liebenden Physiker haben mit dem Experiment OPERA jetzt die Möglichkeit zu beweisen, dass ihre Thesen wirklich stimmen und zwar unabhängig von der Suche nach Higgs-Boson.
    Man sollte in Cern die Myonen im OPERA genauso erzeugen, aber die Orientierung so verändern dass die angeblichen Neutrinos nicht Richtung Gran Sasso fliegen können.
    Den Rest des Experiments soll genauso laufen wie vorher.
    Ich bin sicher dass man trotz nicht Vorhandenen Neutrinos im Detektor genauso die Ausschläge mit der gleichen Zeitunterschied messen würde.
    Ich bezweifle aber dass die Physiker dazu Mut haben. Bis jetzt haben die mit Erfolg vermieden ihre Theorien ernsthaft zu Testen.

    Würden sich die Physiker weigern dieser ultimativen Test durchzuführen, werden sie damit eindeutig zugeben, dass ihre Physik ein Fake ist.

  38. Zeitdilitation und dunkle Materie

    der Effekt der Zeitdilitation wurde schon bedacht? Die Neutrinos bewegten sich durch das Erdinnere, die Forschungslabore waren weiter vom Erdinneren entfernt. Die Zeit läuft in den unterschiedlich schnellen Systemen nicht gleich ab, spätestens beim Zusammentreffen ist ein Zeitunterschied feststellbar.

    Die Ausbreitungsgeschwindigkeit des Lichts hängt vom Medium ab. Kann es sein, daß wir das, was wir für Vakuum halten, nicht ganz frei
    von Materie ist, z.B. dunkler Materie. Neutrinos wechselwirken vielleicht mit dieser Form von Materie seltener als Photonen.

  39. c=const

    Seit 1925 (Michelson/Gale)ist bekannt, dass die Lichtgeschwindigkeit auf der Erde aufgrund des Sagnac-Effekts anisotrop ist (Michelson/Gale). Die Neutrino-Experimente fanden auf der Erde statt, wo man gar nicht erwarten konnte, dass die SRT mit dem Postulat c=const gilt. Insofern muss die Aufregung um das Neutrino-Experiment äußerst verwundern.

    Auf Anfrage hat mir Prof. Ereditato mitgeteilt, dass die ca. 170 Autoren des OPERA-Reports den Sagnac-Effekt bei der Datenauswertung nicht berücksichtigt haben. Der Effekt wirkt sich bei der Uhrensynchronisation noch viel stärker aus. Diese Synchronisation wurde von einem Dr. Feldmann im Auftrag der PTB besorgt, von dem nicht bekannt ist, ob er als Einziger an den Sagnac-Effekt gedacht hat. Leider weigert sich die PTB, die Adresse von Dr. Feldmann bekannt zugeben. Dr. Bauch von der PTB behauptet, Dr. Feldmann habe die Regeln der SRT, also c=const verwendet, der Präsident Prof. Göbel behauptet, Dr. Feldmann habe den Sagnac-Effekt berücksichtigt, also c =/= const angenommen. Einer Klärung dieser Fragen geht der Präsident aus dem Weg, indem er sich weigert, künftig e-mails von mir zu beantworten.

    Herr Pössel wäre gut beraten, den Sagnac-Effekt in seine Erklärungen zum OPERA-Experiment einzubeziehen. Vielleicht gelingt es ihm, von der PTB zu erfahren, ob man nun c=const entsprechend dem Postulat der SRT angenommen hat, oder ob man berücksichtigt hat, dass das Experiment auf der Erde stattgefunden hat, wo die Lichtgeschwindigkeit in Richtung Ost-West c+v und in Richtung West-Ost c-v ist, wie schon 1925 experimentell verifiziert.

    Die jüngste Behauptung, ein “schief” eingeführter Stecker könne zu 60 ns Verzögerung führen, kann unter Experten nur Gelächter auslösen. Sollte das Signal 60 ns in der Steckdose verbummelt haben, weil es den Ausgang nicht fand?

  40. @Dr. Wolfgang Engelhardt

    Zitat Dr. Wolfgang Engelhardt: »Diese Synchronisation wurde von einem Dr. Feldmann im Auftrag der PTB besorgt,«

    Das stimmt so nicht. Die fragliche GPS Installation war von den Schweizern (METAS) vorgenommen worden, die Deutschen (PTB) hatten lediglich die Funktionalität nachgeprüft und bestätigt.

    Bei GPS wird der Sagnac-Effekt routinemässig berücksichtigt. Das ist dokumentiert und lässt sich übrigens auch via google leicht finden.

    Haben Ihre “lachenden Experten” denn auch schon eine Erklärung dafür, warum die OPERA Sensationsmessungen bei ICARUS ganz und gar nicht bestätigt werden?
    http://press.web.cern.ch/…ases2011/PR19.11E.html

  41. Wackelkontakt

    Prof. Ereditato hat mir im Zusammenhang mit Fragen zur Uhrensynchronisation am 31.10.2011 geschrieben:

    “Dear Wolfgang,
    even if we did not elaborate on that in the paper, this is taken into account in the synchronization procedure. We will be more explicit in the journal publication.
    If you wish you could directly ask the PTB about it.
    Kind regards,
    Antonio”

    Dies habe ich mit Hinweis auf Ref. [22] (Verfasser Thorsten Feldmann, PTB) des OPERA-Reports getan und von der PTB unbefriedigende Auskünfte erhalten, wie oben beschrieben. Was Herr Feldmann genau gemacht hat, lässt sich auch “via google” nicht ermitteln.

    Vom 21. Oktober bis 6. November gab es offenbar eine neue Kampagne zur Messung der Neutrinogeschwindigkeit, wobei Einzelereignisse registriert werden konnten. OPERA machte etwa 20 Ereignisse dingfest und bestätigte die Ergebnisse, die in den Jahren 2009-2011 mit statistischen Methoden an ca. 15000 nachgewiesenen Neutrinos gewonnen worden waren. Daraufhin wurde das Manuskript an das Journal of High Energy Physics (17 November 2011) zur Veröffentlichung eingereicht.

    Im gleichen Zeitraum führte ICARUS Messungen am gleichen Neutrinostrahl durch und machte 7 Neutrino-Ereignisse dingfest. Man kam zu dem Schluss: “The result del t = 0.3 +/- 4.9(stat.) +/- 9.0(syst.) ns is compatible with the simultaneous arrival of all events with speed equal to that of light.” Das Erstaunliche ist, dass beide Gruppen die gleiche Prozedur zur Uhrensynchronisation benützten, wie von ICARUS ausdrücklich mit Bezug auf den revidierten OPERA-Report arXiv:1109.4897 betont wird. Hat etwa ein Saboteur immer dann, wenn OPERA am Messen war, am Stecker gewackelt? Und das auch schon die drei Jahre vorher, als OPERA allein am Messen war? Man erträgt nur mit einem gehörigen Maß an Humor die Kapriolen ernsthafter Wissenschaftler, die den Steuerzahler nicht unerhebliche Summen Geld kosten. Gelächter ist vielleicht doch die angemessenste Reaktion.

    Und das alles, weil man plötzlich an einer ziemlich verrückten Idee von Woldemar Voigt (http://www.wissenschaftliche-physik.com/…ion.pdf , http://www.wissenschaftliche-physik.com/…_LT.pdf ), die sich für Schall als völlig unzutreffend erwies, die aber einige Physiker (Lorentz, Poincaré, Einstein) bei Licht für denkbar hielten, Zweifel bekam. Im Jahre 1925 war diese Idee schon durch die Michelson-Gale Messungen falsifiziert worden, denn c=const galt offenbar nicht auf der Erde. Man klammerte sich an die Tatsache, dass die Erde doch rotiere, also kein Inertialsystem sei. Ja, das ist richtig. Während das Licht in entgegengesetzten Richtungen das riesige Sagnac-Interferometer umkreiste und zu unterschiedlichen Zeitpunkten am Ausgangspunkt wieder ankam, bewegte sich die Erdoberfläche mit den Interferometerspiegeln tatsächlich um 3 mm! Wegen der Rotation allerdings auf einem Kreisbogen, der um einen hundertstel Atomdurchmesser von der geraden Linie abwich. Schwupps! war der x v/c^2 Term der Lorentz-Transformation wieder verschwunden, die Galilei-Transformation war im beschleunigten Bezugssystem wieder zu ihrem Recht gekommen. Doch im Wolkenkuckucksheim der Inertialsysteme gilt weiterhin die Lorentztransformation, wird mal widerlegt, mal bestätigt. Eigentlich ist das nicht zum Lachen, Herr Chrys.

  42. @Dr. Wolfgang Engelhardt

    Was sich beispielsweise recht schnell ergoogeln lässt, ist diese Quelle:
    Neil Ashby. The Sagnac effect in the Global Positioning System. In: G. Rizzi and M. L. Ruggiero (eds.) Relativity in Rotating Frames, Kluwer, Amsterdam, 2004 [Draft].
    Da heisst es dann:

    Section 5 discusses Sagnac corrections that are necessry when comparing remote clocks on earth by observations of GPS satellites in common-view.

    Und die besagte Section 5 schliesst mit der Feststellung:

    Sagnac corrections of the form of Eq. 1.19 are routinely used in comparisons between distant time standards laboratories on earth.

    Dies deckt sich dann doch auch mit der Auskunft, die Sie von Prof. Ereditato erhalten haben, sofern sich diese auf den Sagnac-Korrekturen bezieht, wovon ich ausgehe. Die Time Of Flight Inkonsistenz zwischen den OPERA und den ICARUS Messungen wird ihre Ursache schwerlich in dem Teil der LNGS Equipments haben, den beide Experimente gemeinsam nutzen. Man sollte also schauen, wo OPERA mit den Zeitsignalen etwas anders macht als ICARUS, aber dafür kann der Sagnac-Effekt doch keine Rolle mehr spielen. Dann eher schon der ominöse Stecker.

  43. @Chrys

    Als Außenstehender kann man nicht wissen, welche Kämpfe in den Höhlen tief unter dem Gran Sasso ausgetragen werden. Allerdings empfinde ich es als Skandal, dass sich zwei riesige Teams von hochbezahlten Wissenschaftlern (das kleinere von ca. 70 Personen verfügt sogar über einen Nobel-Preisträger) nicht darauf einigen können, wann Neutrinos, die zu einem bekannten Zeitpunkt in CERN erzeugt werden, unter dem Gran Sasso ankommen. Warum können sich die Herrschaften nicht zusammensetzen, gegenseitig ihre Daten analysieren und anschließend der Öffentlichkeit ein belastbares Messergebnis präsentieren? So wie die Sache jetzt aussieht, haben diese “Forscher” jegliches Vertrauen verspielt und verdienen nicht, dass man ihren politischen Verlautbarungen, die offenbar mit Physik nichts mehr zu tun haben, noch Beachtung schenkt.

    Immerhin war für mich eine Bemerkung im ICRUS-Report aufschlussreich, nämlich dass die Standard GPS-Empfänger in CERN und LNGS eine Genauigkeit von nur etwa 100 ns hätten. Um eine Zeitdifferenz dieser Größenordnung geht es gerade! Man hat deshalb genauere Empfänger “Septentrio PolaRx2e” und dazu noch eine Cäsium-Uhr “Symmetricom Cs4000” gekauft, sicher nicht billig das Ganze. Dabei hätte ich mit meinem handlichen Wander-Garmin für ein paar 100 € aushelfen können, das auf 3 m, entsprechend 10 ns genau misst, wie ich selbst im Gebirge auf meinen Wanderungen überprüfen konnte. Man fragt sich, was diese “Experimentatoren”, die nicht mal einen Stecker fixieren können, eigentlich wissen und machen.

    Das einzig Interessante an diesen “Messungen” liegt darin, dass der verdrängte Widerspruch zwischen Sagnac-Effekt und SRT ans Tageslicht gebracht wird. Vielen Dank, Herr Chrys, für den Ashby Aufsatz, der diesen Widerspruch schön herausarbeitet. Ashby leitet die übliche Sagnac-Formel im nicht rotierenden ECI-System her (1.9) und lässt dann die gegenläufigen Strahlen in diesem System “theoretisch” interferieren, obwohl es da gar kein Interferometer gibt. Die Interferenz erfolgt im rotierenden System, z.B. an Michelson´s Spiegeln. Tatsächlich findet man dort eine Phasendifferenz wie Ashby sie ausrechnet (1.10), aber dies bedeutet, dass man die im ECI-System berechneten Zeitdifferenzen mit Hilfe der Galilei-Transformation t´ = t auf das rotierende System einfach überträgt. Hätte man korrekt mit der Lorentz-Transformation gerechnet, dann wäre natürlich c = const herausgekommen, d.h. im rotierenden System hätte es, im Gegensatz zur Messung, gar keine Zeitdifferenz gegeben.

    Bei der Satellitenbeobachtung “in common view” könnte man nun auf die Idee verfallen, die SRT anzuwenden, nach der die Lichtgeschwindigkeit isotrop, unabhängig von der Geschwindigkeit des Senders und auch des Empfängers ist. Dann würde man nach Aussendung eines Signals von einem bekannten Satellitenort aus das gleiche markierte Signal an zwei unterschiedlichen Orten auf der Erde auffangen, die bekannten Abstände zum Satelliten durch c teilen und mit Hilfe der berechneten Zeitdifferenzen die räumlich entfernten Uhren synchronisieren. So scheint es nach Aussage von Dr. Bauch sein Mitarbeiter Dr. Feldmann gemacht zu haben, denn die korrekten Regeln der SRT, also c=const, seien sicher gestellt gewesen.

    Nach Aussage des Präsidenten Prof. Göbel, scheint es aber anders gewesen zu sein. Dr. Feldmann habe nämlich den Sagnac-Effekt korrekt berücksichtigt, vermutlich also Ashby´s obige Rechnungen zugrunde gelegt, welche bei der Übertragung der berechneten Zeiten aus dem ECI-System auf das ECEF-System die Galilei- statt der Lorentz-Transformation verwenden. Dann freilich gilt für die Lichtgeschwindigkeit im ECEF-System zwischen Satellit und Bodenstation c+/-v, je nachdem, ob sich die Bodenstation auf den Satelliten zu, oder von ihm wegbewegt. Wie es wirklich gewesen ist, könnten wir nur von Herrn Feldmann selbst erfahren, aber die PTB lässt nicht zu, dass ich mit ihm kommuniziere. Mir ist nicht bekannt, was in dieser eigentlich wissenschaftlichen Frage die PTB zu verbergen hat. Ein wenig schmunzeln muss ich aber schon, wenn der Präsident keine e-mail mehr von mir beantworten will, weil er glaubt, mich mit den widersprüchlichen Aussagen aus seinem Haus hinreichend informiert zu haben. Fehlt es ihm möglicherweise selbst an hinreichender Information?

  44. @Dr. Wolfgang Engelhardt

    Hier sind noch die Folien zu einem Vortrag “CERN-LNGS distance computation for the OPERA Project”, vom 23. Nov. 2011 [PDF]

    Speziell zum Sagnac-Effekt siehe p. 31f.

    Kann allerdings nicht behaupten, dass mir damit die Distanzbestimmung in allen Punkten klar wird. Jemand, der sich damit auskennt, sollte das einmal allgemeiner verständlich aufbereiten.

  45. Distanzbestimmung

    Vielen Dank für die Folien zu diesem Fachvortrag über die Distanzbestimmung zwischen CERN und LNGS. Fast unbesehen kann ich glauben, dass man dabei keine nennenswerten Fehler gemacht hat, wenn mein billiges Wandergerät außerhalb des Tunnels schon auf 3 m genau messen kann. Innerhalb des Tunnels von 10 km Länge konnte man schon vor 100 Jahren Messungen auf wenige cm genau realisieren. Hier dürfte also keine Fehlerquelle die Messung verfälschen. Natürlich ist es bemerkenswert, dass man die Distanz im inertialen ECI-System berechnet und das Ergebnis kritiklos unter Missachtung der Lorentztransformation auf das rotierende ECEF-System mit Hilfe der Galilei-Transformation überträgt. Der Fehler von 66 cm, der auf diese Weise eingebracht werden könnte, ist aber nicht gravierend.

    Schlimmer sieht es schon bei der Uhrensynchronisation aus, wo man im rotierenden Erdsystem c=const in Übereinstimmung mit der SRT annehmen könnte, wie es wahrscheinlich Herr Feldmann nach Aussage von Dr. Bauch gemacht hat, oder c+/_v unterstellt hat, was sein Präsident suggeriert. Bisher haben wir kein Statement von Herrn Feldmann, wie es wirklich gewesen ist.

    Völlig unannehmbar ist es aber, dass die Öffentlichkeit mit unterschiedlichen Fakten über dasselbe Experiment bedient wird. Es ist auffällig, dass die 7 ICARUS Messungen sich völlig symmetrisch um den ungefähren Erwartungswert del t = 0.5 ns herumgruppieren, während der statistische Fehler zu +/- 4.9 ns und der systematische Fehler zu +/- 9.0 ns abgeschätzt wurde. Es soll ja Wunder geben, aber mir ist ein solches in langjähriger Messpraxis noch nicht untergekommen.

  46. Neutrino hin oder her

    Heute möchte ich mich einmal ungeniert in die Diskussion um das Problem der rasenden Neutrinos einmischen.
    Vielleicht kann ich auch etwas zur Ehrenrettung der teilnehmenden Forschern am Neutrino-Experiment beitragen. Die haben nämlich nicht nur großen persönlichen Einsatz am Experiment geleistet sondern auch noch den Mut besessen zu sagen, dass es etwas gibt, dass schneller ist als Licht. Sie werden sich auch bewusst gewesen sein was sie damit anrichten. Mit dieser Behauptung stießen sie die Relativitätstheorie vom Sockel. Denn während des Experiments hätte die Zeit rückwärts laufen müssen – tat es aber nicht!
    Es war sehr aufregend als mitgeteilt wurde, dass die Relativitätstheorie mit diesem Experiment hinfällig ist. Die rund 730 km lange unterirdische Strecke wurde, mir nichts dir nichts, schneller von den Neutrinos überwunden als dass es das Licht je könnte. Die Geschwindigkeit des Licht gilt als die Grenzgeschwindigkeit die kein Körper je erreichen kann. Und nun?
    Nun wurde eifrig nach Fehlern und Ausreden gesucht, nicht von Experimentatoren, die mussten erst einmal einstecken. Dann gab es eine Meldung: „Ein wackliger Stecker war Schuld“ Es gibt auch andere Meldungen in der Art wie: „Es ist alles richtig abgelaufen nur die Auswertung war Fehlerhaft“ oder „defektes Glasfaserkabel“

    Meine Gedanken. Das Experiment ist zur vollsten Zufriedenheit und ohne Fehler abgelaufen. Es wird nur Wasser mit Feuer verglichen. Wie ich darauf komme?
    Die Neutrinos durchlaufen die Erdkruste mit Überlichtgeschwindigkeit. Es ist Materie die die flinken Burschen durchdringen. Was, wenn sie „nur“ Vakuum durchlaufen müssten? Die Lichtgeschwindigkeit wird mit dem Zusatz „im Vakuum“ angegeben.

    Die Lichtgeschwindigkeit in Materie wie Glas, Wasser, Diamant, ist ein völlig andere. Wenn also die hohe Geschwindigkeit der Neutrinos mit der der Lichtgeschwindigkeit verglichen wird, dann bitte in der selben Materie. Dann wird es noch übler für die Relativitätstheorie.

    Jungs, da in Cern, die ihr am Experiment gearbeitet habt, lasst euch nicht ins Boxhorn jagen. Macht noch ein Experiment welches in Materie wie Glas, Diamant usw. stattfindet. Lasst die Kontrahenten, Neutrino und Licht, unter gleichen Bedingungen von der Leine.
    Man lässt schließlich auch keine Rennautos auf Asphalt und Sumpf gegeneinander antreten.

  47. So, inzwischen ist alles erledigt. Im Mai wurde ein neuer Neutrino-Strahl benutzt, und gleich vier Gran Sasso Experimente (OPERA, ICARUS, LVD, BOREXINO) haben diesen ausgewerted: Alle in Übereinstimmung mit der Lichtgeschwindigkeit.

    Borexino: ´t = 2.7 ± 1.2 (stat) ± 3(sys) ns
    ICARUS: ´t = 5.1 ± 1.1(stat) ± 5.5(sys) ns
    LVD: ´t = 2.9 ± 0.6(stat) ± 3(sys) ns
    OPERA: ´t = 1.6 ± 1.1(stat) [+ 6.1, -3.7](sys) ns

    Basierend auf den im Februar gefundenen Fehlerquellen (looser Kabel, fehlerhafter Oszillator) hat OPERA auch ihre 2011-Daten korrigiert und erhielten Übereinstimmung mit der Lichtgeschwindigkeit:

    ´t = (6.5 ± 7.4 (stat.)+9.2 (sys.)) ns

    Schließlich hat auch MINOS am Fermilab Messungen vorgelegt, ebenfalls in Übereinstimmung mit C:

    ´t = 11.4 ± 11.2 (stat) ± 29 (syst) ns (68% C.L)

    Siehe CERN-Pressemitteilung:
    http://press.web.cern.ch/…ases2011/PR19.11E.html

    Wikipedia:
    http://de.wikipedia.org/…Neutrinogeschwindigkeit

    Matt Strassler’s Blog:
    http://profmattstrassler.com/…f-the-opera-story/

    FAZ:
    http://www.faz.net/…-fuer-einstein-11778511.html

  48. If at first you don’t succeed …

    Titus schrieb (12.06.2012, 11:08):
    > So, inzwischen ist alles erledigt.

    Wohl kaum.
    Zu erklären bliebe der (vermeintliche) Zusammenhang der genannten (Mess-)Größe “´t” bzw. “δt” und ihrer experimentell gefundenen werte oder auch ihres gesamten Wertebereiches mit der (Mess-)Größe “Geschwindigkeit” und Werten wie z.B. “Lichtgeschwindigkeit” oder “Überlichtgeschwindigkeit”.

    Und denjenigen, die Messungen von “´t” bzw. “δt” zu “Tests der Relativitätstheorie” aufbauschen wollten, bliebe nachzuweisen, dass die RT irgendwelche bestimmte Erwartungen bzgl. der (geometrischen) Beziehungen zwischen den genannten Forschungseinrichtungen beinhaltet.

  49. @Titus

    Noch immer ist nicht geklärt, ob bei der Uhrensynchronisation die spezielle Relativitätstheorie mit dem Postulat c = const, oder ob der Sagnaceffekt mit c +/- v zur Anwendung kam. Hierüber gibt es widersprüchliche Aussagen der Physikalisch-Technischen Bundesanstalt: Dr. Bauch behauptet, dass der verantwortliche Physiker Dr. Feldmann c = const angenommen habe, während der ehemalige Präsident Professor Göbel angibt, der Sagnaceffekt, also c +/- v , sei korrekt berücksichtigt worden. Allerdings sagt er nicht, mit welcher Zeittransformation die Berechnungen im Inertialsystem auf das rotierende System Erde übertragen wurden. Es steht zu vermuten, dass die Galileitransformation t´ = t statt der Lorentztransformation verwendet wurde. Das Neutrino-Laufzeit Experiment eröffnet die Chance zu ermitteln, ob die Resultate mit dem Postulat c = const, oder mit den Messungen von Michelson und Gale übereinstimmen, die dieses Postulat nicht bestätigen.

    PS:
    Wegen Umzugs auf eine neue Adresse seien die oben zitierten links in der jetzt gültigen Form noch einmal angegeben:
    http://www.kritik-relativitaetstheorie.de/…T.pdf
    http://www.kritik-relativitaetstheorie.de/…n.pdf

  50. c +/- v

    Nach Auskunft des Schweizer Bundesamts für Metrologie (METAS) steht nun fest, dass bei der Uhrensynchronisation “in common view” nicht die SRT mit dem Postulat c = const, sondern der Sagnaceffekt mit c +/- v zugrunde gelegt wird. Damit wird die Angabe von Prof. Göbel grundsätzlich bestätigt, während die Aussage von Dr. Bauch, nämlich die Regeln der SRT seien sichergestellt gewesen, als Fehlinformation angesehen werden muss. Allerdings ist nicht auszuschließen, dass der junge Physiker Dr. Feldmann naiverweise doch die SRT als gültig angesehen hat, und unter der Voraussetzung c = const die Uhrensynchronisation vorgenommen hat. In seinem Report http://operaweb.lngs.infn.it/…cnotes/note134.pdf hat Dr. Feldmann jedenfalls den Sagnaceffekt mit keinem Wort erwähnt. Solange die PTB den Kontakt zu Dr. Feldmann nicht vermittelt, kann man hierüber nur spekulieren.

    Dank der Hartnäckigkeit von Frau Lopéz hat sich die PTB schließlich doch zum Synchronisationsproblem geäußert: http://www.kritik-relativitaetstheorie.de/…talt/ . Zwar wurden die Fragen 1) – 3), die Frau Lopéz präzise formuliert hatte, nicht wirklich beantwortet, weil die PTB offenbar auch nicht weiß, welche Annahmen der von ihr verwendeten Software zugrunde liegen, es scheint aber aus den vagen Andeutungen von R. Wynands hervorzugehen, dass man grundsätzlich den Sagnac Effekt und nicht das SRT-Postulat c = const bei der Uhrensynchronisation voraussetzt. Dies stimmt mit der Angabe von METAS in einer an mich gerichteten e-Mail (Absender: Jacques Morel) überein. Nur folgerichtig ist es dann, wenn man bei der Übertragung der im inertialen ECI-System berechneten Zeitdifferenzen in das rotierende Erdsystem nicht die Lorentztransformation, sondern die Galileitransformation verwendet. So jedenfalls hat es N. Ashby gemacht (s.oben) und es besteht kein Grund zu der Annahme, dass die PTB (bzw. die von ihr verwendete Software) anders vorgegangen ist.

    Bei aller Kontroverse um das Neutrinoexperiment hat es doch den unbestreitbaren Nutzen gehabt, die Richtigkeit der Messungen von Michelson und Gale aus dem Jahr 1925 im Prinzip zu bestätigen: Es gilt c +/- v und nicht das Postulat c = const.

  51. @Dr. Wolfgang Engelhardt

    Zum Sagnac Effekt, dessen Rolle wird, wenngleich eher beiläufig, so doch expressis verbis erwähnt in folgendem, von der PTB herausgegebenen Aufsatz:

    D. Piester, H. Schnatz. Hochgenaue Zeit- und Frequenzvergleiche über weite Strecken. PTB-Mitteilungen, 119, Heft 2 (2009) 131-143 [Link zum Heft]

    Dr. Feldmann wird dort übrigens in der Danksagung genannt.

    Erfahrungsgemäss zählen einige Physiker alles zur SR, was relativistisch ist und ohne Gravitation auskommt, also auch rotierende Bezugsysteme. Dr. Bauch mag zu diesen Leuten gehören, und dann wäre es nur konsequent, wenn er bekundet, die Regeln der SR seien sichergestellt gewesen. Diese Deutung von Dr. Bauchs Äusserung scheint mir dann doch allemal näherliegend als Dr. Feldmann so pauschal zu verdächtigen, er würde sein routinemässiges Handwerk nicht verstehen.

  52. @Chrys

    Es liegt mir völlig fern, Dr. Feldmann zu verdächtigen, er verstehe sein Handwerk nicht. Ganz im Gegenteil: Er schien nur konsequent anzuwenden, was man ihm während seiner Ausbildung beigebracht hatte, nämlich die Lichtgeschwindigkeit sei unabhängig von der Geschwindigkeit sowohl der Quelle als auch des Empfängers, also c = const entsprechend Einsteins Vermutung. Dieser Ansicht waren ja auch die 170 Autoren des OPERA-Reports, sowie Dr. Bauch, der mir an Dr. Feldmanns Statt auf meine Anfrage geantwortet hatte. Zu gerne hätte ich vom Autor selbst erfahren, warum er in seinem Report den Sagnac-Effekt nicht erwähnt, aber die PTB hat die Kontaktaufnahme nicht vermittelt.
    Nunmehr ist klar, weshalb sich die PTB so lange weder zu der von ihr angewandten Synchronisationsprozedur äußern, noch die präzise gestellten Fragen von Frau Lopez ebenso präzise beantworten konnte: Man vertraut auf die Software von Herstellern, ohne zu hinterfragen, welche physikalischen Annahmen den Algorithmen zugrunde liegen. Man sei bisher auf keine Inkonsistenzen gestoßen. Wie sollte man auch, wenn man immer dasselbe tut und die Synchronisation von weit entfernten Uhren nicht unabhängig überprüfen kann. Das geschieht erst jetzt durch das Neutrinoexperiment und prompt ist man auf Inkonsistenzen gestoßen. Immerhin weiß man nun durch ein knapp gehaltenes Statement von METAS, dass bei der Synchronisation c +/- v – in Übereinstimmung mit dem Sagnac-Effekt – und nicht c = const vorausgesetzt wird.

    Als sicher kann gelten – auch wenn es dafür keine ausdrückliche Bestätigung gibt – dass man nach Methode Ashby die im ECI-System berechneten Zeitdifferenzen mit Hilfe der Galileitransformation ins bewegte Erdsystem überträgt. Dies ist insofern konsequent, als nach Abkehr von c = const die Lorentztransformation, die auf diesem Postulat beruht, gegenstandslos wird.

    Chrys macht eine vage Andeutung, dass – anders als in einem Inertialsystem – auf der rotierenden Erde die SRT womöglich gar nicht anwendbar ist. Er wird dies aber nicht ernsthaft glauben, wenn er sich klar macht, dass sich die Erdoberfläche in unseren Breiten zwischen Aussendung und Empfang eines GPS-Signals um ca. 20 Meter auf einem Kreisbogen bewegt, dessen Abweichung von der geraden Linie nur etwa 50 Mikrometer beträgt. Es ist eine absurde Vorstellung und entbehrt jeder physikalischen Begründung, dass die damit verbundene winzige Querbeschleunigung die Lorentztransformation in die Galileitransformation überführen sollte. Viel plausibler ist es anzunehmen, dass Einsteins Vermutung auch in einem Inertialsystem nicht zutrifft.

  53. @Dr. Wolfgang Engelhardt

    Leider gibt die Literatur keine verbindliche Antwort auf die Frage, ob rotierende Bezugssysteme zur SR gehören oder nicht. Das wird recht unterschiedlich gehandhabt, sodass eine Behauptung, die Regeln der SR seien sichergestellt, durchaus missverständlich sein kann. Darauf wollte ich hinweisen, und zur Erläuterung sei das nochfolgend nochmals etwas ausführlicher dargelegt (in der Hoffnung, dass es lesbar herauskommt).

    Mit Zylinderkoordinaten (r,φ,z) im 3-Raum ist die Minkowskische Metrik gegeben durch

    g = dt² – dr² – r² dφ² – dz²,

    wobei also, der Übersichtlichkeit halber, c = 1 gelte. Sei ω > 0 als Winkelgeschwindigkeit fest gewählt, dann wird für den Bereich {(t,r,φ,z) | rω < 1} vermöge α = φ – ωt eine Transformation in rotierende Koordinaten (t,r,α,z) definiert. Setzen wir noch γ = √{1 – r&sup2ω²}, so ist bezüglich der Metrik

    g’ = (1/γ dt – r²ωγ d&alpha)² – dr² – r²γ² dα² – dz²

    dieser Koordinatenwechsel eine Lorentzsche Isometrie im Sinne der GR, aber freilich keine Lorentz Transformation der SR. Für Physiker ist g’ nichts anderes als die Minkowskische Metrik g ausgedrückt in den rotierenden Koordinaten. In der Metrik g’ erscheinen nun die Lichtkegel infolge des Coriolis Effektes geneigt, und die Lichtgeschwindigkeit ist daher richtungsabhängig — dieser Sachverhalt wird durch den ersten Ausdruck auf der rechten Seite wiedergegeben. So findet der Sagnac Effekt dann seine Erklärung im geometrischen Raumzeit-Formalismus, und genau das hat Neil Ashby wohl auch im Sinn, wo er schreibt:

    In the rotating frame of reference the [Sagnac] effect appears to arise from a Coriolis-like term in the fundamental scalar invariant.

    Die Galilei-Transformation wird dabei nirgends verwendet.

    Sie können nun sagen, die Regeln der SR seien hierbei verletzt, weil die Transformation in rotierende Koordinaten keine Lorentz Transformation ist. Das ist mathematisch gerechtfertigt.

    Dr. Bauch könnte seinerseits anführen, die Physik eines rotierenden Zylinders sei keine Frage der Koordinaten, und im nicht-rotierenden Inertialsystem seien die Regeln der SR doch schliesslich erfüllt. Das ist, nun ja, physikalisch zumindest nicht ganz ungerechtfertigt.

  54. Korrektur

    γ = √{1 – r²ω²}

    g’ = (1/γ dt – r²ωγ dα)² – dr² – r²γ² dα² – dz²

    Wann kommt hier der Preview?

  55. Korrektiur #2

    Sorry, es hätte natürlich heissen sollen

    γ = 1/√{1 – r²ω²}

    Daran war nicht die Software schuld.

  56. @Chrys

    • Dr. Wolfgang Engelhardt @Chrys
    Sehr schön! Nun wissen wenigstens Sie, wie man die Synchronisation von zwei weit entfernten Uhren bewerkstelligt, wenn es uns die PTB schon nicht sagen kann, weil sie keinen Einblick in die verwendete Software hat. Ich für mein Teil würde folgendermaßen vorgehen: Zur GPS-Zeit t, die auf das inertiale ECI-System bezogen ist, wird ein Signal vom Satelliten zu zwei Stationen auf der Erde geschickt, die zur Zeit t den Abstand x1 und x2 vom Satelliten haben. Die Laufzeit zu den Stationen beträgt x1/c + dt1 bzw. x2/c + dt2. Die Korrekturen dt1,2 lassen sich aus Kenntnis von Geometrie und Rotationsgeschwindigkeit der Erdoberfläche leicht ausrechnen. Nun würde ich die Stationsuhren bei Eintreffen der Signale auf die Zeit t + x1/c + dt1 bzw. t + x2/c + dt2 stellen. Sie sollten dann unter Berücksichtigung des Sagnac-Effekts synchronisiert sein. Aus den Andeutungen von Dr. Morel scheint hervorzugehen, dass METAS in dieser Weise verfährt. Wenn Sie wollen, können Sie ihn ja nochmals befragen (Jacques.Morel@metas.ch).
    Allerdings hat man bei dieser Vorgehensweise angenommen, dass man die Zeiten t + x1,2/c + dt1,2 einfach eins zu eins auf das rotierende Erdsystem übertragen kann, obwohl sie im ECI-System definiert sind. Das bedeutet jedoch, dass man die Galilei-Transformation der Zeit: t´ = t angewendet hat.
    Hätte man mit Einstein unterstellt, dass die Geschwindigkeit des Lichts unabhängig von der Geschwindigkeit sowohl der Quelle als auch des Empfängers c = const sei, woraus die Lorentztransformation folgt, so wären die Laufzeiten des Lichts aus der Sicht der Stationsuhren x1/c bzw. x2/c gewesen und man hätte sie auf eine Zeit t1 + x1/c, bzw. t2 + x2/c stellen müssen, wobei die Zeiten t1 und t2 aus der Lorentztransformation zu errechnen wären. Dabei entsteht die Paradoxie, dass der Sendezeitpunkt t, der per Definitionem im ECI-System für das Signal, das in unterschiedliche Richtungen abgestrahlt wird, gleichzeitig ist, von den Erduhren als ungleichzeitig „erlebt“ wird, weil in die Lorentz-Transformation der Zeit noch die Abstände x1,2 eingehen. Nur für x1 = x2 gäbe es einen gemeinsamen Startpunkt für die Aussendung des Zeitsignals.
    METAS hat kaum so etwas Abstruses angenommen, sondern, wie gesagt, die obige Sagnac-Korrektur vorgenommen, wobei die Galileitransformation t = t´ in Übereinstimmung mit Ashby unterstellt wurde. Auch wenn Ashby nicht ausdrücklich davon spricht, so errechnet er doch beim Sagnac-Interferometer von Michelson/Gale Interfernzstreifen im ECI System, die nicht dort, sondern im rotierenden Erdsystem gemessen werden. Das heißt aber, er nimmt de facto die Galileitransformation der Zeit an: t = t´. Was die PTB inhaltlich gemacht hat, können wir nicht wissen, weil sie es nach Aussage des Leiters „Präsidialer Stab PTB“ R. Wynands auch nicht weiß. Ultra posse nemo obligatur.

  57. @Dr. Wolfgang Engelhardt

    Mich irritiert noch immer, dass Sie hierbei einen Bezug zur Lorentz- bzw. Galilei-Transformation sehen wollen. Bei der Transformation zwischen inertialem (ECI) und rotierendem (ECEF) Koordinatensystem muss weder auf das eine noch auf das andere zurückgegriffen werden.

    Mit der in Hinblick auf Sagnac unwesentlichen Vereinfachung, dass ich oben Zylinderkoordinaten und nicht Kugelkoordinaten gewählt hatte, lässt sich die Sache mit der Lichtgeschw. c = const. (ECI) und c ± v (ECEF) unter Verwendung von g’ rasch auflösen. Bezüglich der Minkowski Metrik g hatten wir c = 1 (entsprechend ECI), bezüglich g’ gilt das aber nicht (entspr. ECEF). Dazu betrachte man etwa im rotierenden System lichtartige Kurven der Form (t,r,α(t),z), deren r- und z-Koordinaten also konstant seien. Die Metrik g’ liefert hierzu als Bedingung die Diff.gleichung

    (dα/dt + ω)² = 1/r².

    Deren Lösungen zur Anfangsbedingung α(0) = 0 sind gegeben durch

    (α_+)(t) = (1 − rω) t/r und (α_−)(t) = −(1 + rω) t/r,

    wobei α_+ mit und α_− entgegen der Drehrichtung läuft. Richtungsabhängig finden wir also die Lichtgeschw. als 1 ± rω. Ohne die Normalisierung c = 1 und mit v = rω ware das Ergebnis c ± v, wie gewünscht und trotz relativistischer Rechnung.

    Die Zeitkoordinate t ist bei der ECI ↔ ECEF Transformation gar nicht betroffen. Das t wird auch bei obiger Rechnung unverändert als die Koordinatenzeit des Inertialsystems durchgeschleppt, entspricht also in der Praxis der GPS-Zeit. Die Transformation zwischen inertialem und rotierendem System war schlicht (t,r,φ,z) ↔ (t,r,φ−ωt,z), da braucht es keine Umbenennung t’ = t.

    N.B. Allenfalls die Transformation t’ = t/γ ist bisweilen noch angebracht, aber Hinblick auf die angestrebte GPS-Synchronisierung müsste man das auch wieder herausrechnen, das bringt hier also nichts.

  58. Elende Koordinaten-Schubserei

    Chrys schrieb (09.08.2012, 15:17):
    > [… Dreh-]Richtungsabhängig finden wir also die Lichtgeschw. als 1 ± r ω.

    Diskutiere den Zusammenhang dieses Ergebnisses mit den Festsetzungen der SI-“Meter”-Definition; siehe z.B. http://www.bipm.org/en/si/base_units/metre.html

    (“Drehung” bzw. Werte von “r ω” werden dort nicht genannt; und folglich offenbar auch nicht eingeschränkt.

    Wäre diesbezüglich eine bestimmte Einschränkung sinnvoll bzw. erforderlich?

    Und falls so:
    Wäre der Begriff “Lichtgeschwindigkeit” bzw. deren Bewertung (z.B. wie oben als “1 ± r ω“) auch ohne eine solche Einschränkung sinnvoll? …)

  59. @Frank Wappler

    »Diskutiere den Zusammenhang dieses Ergebnisses mit den Festsetzungen der SI-“Meter”-Definition …«

    Derlei wurde schon ad nauseam diskutiert … [Physics FAQ]:

    The use of c as a conversion between units of metres and seconds, as in the SI definition of the metre, is fully justified on theoretical grounds as well as practical terms, because c is not merely the speed of light, it is a fundamental feature of the geometry of spacetime.

  60. Hellwacher Unterschied

    Chrys schrieb (10.08.2012, 09:54):
    > Derlei wurde schon ad nauseam diskutiert …

    Offenbar ist dir entgangen, warum ganz konkret “dein Ergebnis” (09.08.2012, 15:17) im Zusammenhang mit der SI-“Meter”-Definition einer Diskussion bedarf.

    Deshalb nochmals möglichst deutlich:
    Da du offenbar weißt und akzeptierst, dass

    c is […] the speed of light

    ,

    wie kommst du darauf (09.08.2012, 15:17)
    > das Ergebnis c ± v
    genauso “Lichtgeschw.” zu nennen ?!?
    (Insbesondere sofern es ungleich c ist.)

    Nenn es besser “Koordinaten-(Irgendwas)”, da die Herleitung ja offenbar mit Koordinaten-Betrachtungen zu tun hatte;
    dann wird niemand “dein Ergebnis” mit einer physikalisch relevanten (Mess-)Größe verwechseln.

  61. @Chrys 09.08.2012, 15:17

    Sie drücken sehr präzise aus, was ich kritisiere. Sie schreiben nämlich: „Die Zeitkoordinate t ist bei der ECI ” ECEF Transformation gar nicht betroffen. Das t wird auch bei obiger Rechnung unverändert als die Koordinatenzeit des Inertialsystems durchgeschleppt, entspricht also in der Praxis der GPS-Zeit.“ Aha! Stellen Sie sich mal die Erdoberfläche, auf der sich die zu synchronisierenden Uhren befinden, auch als ein Inertialsystem ECII vor, das sich gegenüber ECI mit der Geschwindigkeit v geradlinig bewegt. Der Unterschied gegenüber dem ECEF System ist minimal: Wie oben bereits beschrieben, beträgt die Abweichung von der Geradlinigkeit nur 50 Mikrometer auf 20 m Länge. Lassen Sie im ECI-System die Zeit t (GPS-Zeit) und im ECII System die Zeit t´ gelten. Nun würden Sie die Zeit nicht „durchschleppen“ (bzw. die Galileitransformation t´ = t anwenden), sondern artig die Lorentztransformation bemühen: t´ = ³ (t – x v/c^2). Wegen der winzigen Querbeschleunigung der Stationen auf ihren Kreisbahnen, die praktisch Gerade sind, lassen Sie einfach den Term x v / c^2 weg, der in der SRT gerade für c = const sorgt? Natürlich kommen Sie dann zu c +/- v, was ja der Realität entspricht, aber Sie ignorieren offenbar die Lorentztransformation und mit ihr die SRT. Lassen Sie mal in Ihrer Rechnung r-> unendlich gehen, É-> 0 und zwar so, dass r É = v. Dann kann man das rotierende ECEF-System gewiss mit obigem ECII identifizieren, denn die Abweichung von der Geradlinigkeit ginge in der Nähe der Stationen gegen Null. Würden Sie jetzt auch die Lorentztransformation ignorieren und die Zeit mit t´ = t „durchschleppen“?

    Bis jetzt haben weder Sie noch die PTB gesagt, wie die Uhren in der Praxis synchronisiert werden sollen. Oben habe ich einen Vorschlag gemacht, von dem ich glaube, dass er dem entspricht, was METAS tut. Stimmen Sie diesem Vorschlag zu?

  62. Software

    Leider stellt die hier verwendete Software meine griechischen Buchstaben nicht richtig dar. Oben muss es natürlich heißen:
    t´ = gamma (t – x v/c^2),
    und:
    omega -> 0 , omega r = v

    Eine Vorschau des Kommentars vor der Veröffentlichung könnte solche nachträgliche Korrekturen vermeiden.

  63. @Frank Wappler

    Konstant und invariant ist die Geschwindigkeit lichtartiger Kurven, die ist null. Was aber als Lichtgeschwindigkeit bezeichnet wird, entspricht der Länge von raumartigen Komponenten lichtartiger Geschwindigkeitsvektoren, und dazu muss man u.a. festlegen, auf welche raumartige Hyperfläche dies bezogen sein soll. So ist Lichtgeschwindigkeit eigentlich grundsätzlich erst einmal koordinatenabhängig.

    Die Definition des SI-Meters mit einem konstanten Konvertierungsfaktor c ist insofern gerechtfertigt, als dass sich lokal bei jedem Punkt p einer Raumzeit (M,g) normale Koordinaten wählen lassen, mit denen sich die Metrik g am Punkt p gerade als die Minkowski Metrik darstellt, also diag[c²,−1,−1,−1].

  64. @Dr. Wolfgang Engelhardt

    Die fraglichen, zu synchronisierenden Uhren bei CERN und LNGS sind aber erdfest. Also machen Sie es doch bitte nicht verwirrender als es ist, indem Sie noch geradlinig gleichförmige Bewegungen dort hineinbringen, die weder METAS noch PTB zu berücksichtigen hatten. Für erdfeste Uhren sind nur ECEF und ECI von Belang.

    Wenn Sie meinen Ausführungen zur Transformation in rotierende Koordinaten und speziell der Begründung für t’ = t misstrauen, in ausführlicherer Darstellung finden Sie das ganz entsprechend auch in einem Artikel von Neil Ashby [Relativity in the Global Positioning System, Sec. 2], den ich jetzt noch aufgestöbert habe.

    Ashby belässt c beliebig konstant, verwendet aber ebenfalls Zylinderkoordinaten und schreibt die Koordinatentransformation als Gl. (3),

    t = t2, r = r2, φ = φ2 + ωt2, z = z2.

    Insbesondere bemerkt er,

    The time transformation t = t’ in Eqs. (3) is deceivingly simple. It means that in the rotating frame the time variable t2 is really determined in the underlying inertial frame. It is an example of coordinate time. A similar concept is used in the GPS.

    Ihre Überlegungen zur GPS-Synchronisierung scheinen mir im Ansatz durchaus zutreffend, allerdings ist dabei t = t’ nicht durch eine allfällige Anwendung der Galilei-Transformation (und damit eine Verletzung der relativistischen Regeln) zu begründen, sondern ganz einfach durch den Hinweis auf die vorstehend genannten Transformationsformeln.

  65. Korrektur

    Ist wieder mal etwas schiefgegangen. Hier nochmals Ashby’s Transformationformeln:

    t = t’, r = r’, φ = φ’ + ωt’, z = z’

  66. RT ist ein Gebiet der Physik

    Chrys schrieb (11.08.2012, 08:53):
    > […] Was aber als Lichtgeschwindigkeit bezeichnet wird, entspricht der Länge von raumartigen Komponenten lichtartiger Geschwindigkeitsvektoren […]

    Gemeint sind hier offenbar Vierervektoren mit Betrag s = 0.
    (Und deren “zeitartige Komponente” ist in diesem Zusammenhang ja nicht ganz egal.)

    Na meinentwegen.
    Das ersetzt wohl kaum die zugrundeliegende (Mess-)Definition von “Geschwindigkeit” (von etwas, das zwischen einem bestimmten geeigneten Start-Beteiligten A und einem bestimmten geeigneten Ziel-Beteiligten B ausgetauscht wurde) als Quotient aus

    Distanz von A und B zueinander
    geteilt durch

    Dauer des Austausches,
    d.h. Dauer As von As Startanzeige bis zu As Anzeige gleichzeitig zu Bs Ankunftsanzeige
    bzw. Dauer Bs von Bs Anzeige gleichzeitig zu As Startanzeige bis zu Bs Ankunftsanzeige.

    Insbesondere die dabei erforderliche Bewertung von Gleichzeitigkeit setzt entsprechend Einsteins Definition natürlich voraus, dass A und B dabei zueinander ruhten; d.h. dass sie zusammen mit (mindestens drei) weiteren Beteiligten starr und flach waren.

    (Eine explizite Forderung gegenseitiger Ruhe des Start-Beteiligten bzw. Licht-Signalgebers und des Ziel-Beteiligten bzw. Licht-Signalempfängers zueinander ist in der SI-“Meter”-Definition zwar nicht gestellt …)

    > und dazu muss man u.a. festlegen, auf welche raumartige Hyperfläche dies bezogen sein soll.

    Start-Beteiligter A und Ziel-Beteiligter B gehören wohl keinesfalls zur selben “raumartigen Hyperfläche“.
    Und beide müssen festgelegt sein, so dass zu gewährleisten ist, dass sie zueinander ruhten.

    > So ist Lichtgeschwindigkeit eigentlich grundsätzlich erst einmal koordinatenabhängig.

    Quatsch — für die Nachvollziehbarkeit des oben Beschriebenen es ist völlig irrelevant, die Beteiligen (A, B sowie weitere) bzw. deren Anzeigenfolgen im Nachhinein mit irgendwelchen Zahlentupeln zu bestreuseln.

    Physik ist grundsätzlich koordinaten-unabhängig. Bedeutung gewinnen Koordinaten allenfalls durch eine bestimmte Beziehung zu den ggf. zugrundeliegenden geometrischen Beziehungen (Dauerverhältnissen, Distanzverhältnissen, Krümmungsradien); so wie z.B. “gute Koordinaten”, d.h. Zahlen, die die zugrundeliegenden geometrischen Beziehungen ggf. affin repräsentieren.

    > Die Definition des SI-Meters mit einem konstanten Konvertierungsfaktor c ist insofern gerechtfertigt, als dass sich lokal bei jedem Punkt p einer Raumzeit (M,g) normale Koordinaten wählen lassen, mit denen sich die Metrik g am Punkt p gerade als die Minkowski Metrik darstellt, also diag[c²,-1,-1,-1].

    Eine solche Argumentation verkennt, dass die grundlegenden Gegenstände der Betrachtungen in der RT nicht irgendwelche koordinaten-haltige “Metriken” oder Mannigfaltigkeiten (“M“) sind, sondern die betrachteten Beteiligten (in MTW Box 13.1 z.B. auch “principal identifiable points” genannt), mit deren gegenseitigen Beobachtungen und den daraus (in gegenseitigem Einvernehmen) zu folgernden geometrischen Beziehungen (wie z.B. gegenseitige Ruhe, und falls so: Distanzverhältnisse).

    Im Übrigen geht es in der Definition von “Geschwindigkeit” (und in der zugrundeliegenden Definition von “Distanz”, sofern sie von Null verschieden ist, sowie in der damit zusammenhängenden SI-“Meter”-Definition) um zwei verschiedene, voneinander getrennte Beteiligte, A bzw. B; die also nicht unbedingt als nur ein Punkt einer Mannigfaltigkeit “M” zu repräsentieren sind.

    p.s.
    Ich komme vermutlich erst Ende August wieder dazu, diesen Blog zu lesen.

  67. @Frank Wappler

    »Quatsch … völlig irrelevant … MTW Box 13.1 …«

    Verstehe. Ein Hinweis auf MTW Formel 13.14 hätte hier möglicherweise mehr gebracht.

    Es existiert noch eine Welt jenseits der Box 13.1 …

  68. @Dr. Wolfgang Engelhardt

    Eine Anmerkung noch zu Ihren Überlegungen zum Vergleich von Kreisbewegung vs. geradlinig gleichförmige Bewegung.

    Das läuft doch darauf hinaus, dass eine auf dem Radius r einer mit Winkelgeschw. ω rotierenden Kreisscheibe befindliche Uhr U zu vergleichen ist mit einer tangentialen Näherung U’. Mit andern Worten, die tangentiale Uhr U’ bewegt sich geradlinig und gleichförmig längs einer Kreistangente mit der Geschw. v = rω. Wenn nun für U’ eine Zeitdilatation feststellbar ist, dann sollte so etwas auch für U gelten.

    Dem ist auch so. Hierbei ist aber zu beachten, dass die Uhren der SR alle die Eigenzeit ihres Ruhesystems anzeigen. Damit dies für U’ gegeben ist, muss auch die Uhr U ihre Eigenzeit anzeigen (denn die Weltlinien von U und U’ haben im Berührungspunkt denselben Tangentialvektor).

    Für rω > 0 parametrisiert unsere Koordinatenzeit t die Bewegung von U aber nicht nach Eigenzeit, wie aus der früher angegebenen Darstellung der Metrik g’ ersichtlich wird. Die Metrik g’ zeigt aber auch, dass die Substitution τ = t/γ mit γ = 1/√{1 − r²ω²} das Gewünschte leistet. Wegen v = rω liefert dies genau den γ-Faktor, der auch gemäss der SR die Zeitdilatation für die Uhr U’ gegenüber der Koordinatenzeit t kennzeichnet.

    Es passt schon alles zusammen, wenn man darauf achtet, dass ein Vergleich der Uhren U und U’ nur bezogen auf einen gemeinsamen Zeitstandard erfolgen kann — also entweder Eigenzeit τ oder Koordinatenzeit t für beide Uhren.

  69. @Chrys 11.08.2012, 09:56

    Nun sind wir einen schönen Schritt weitergekommen. Offenbar akzeptieren Sie meinen Vorschlag zur Synchronisierungsprozedur, die höchstwahrscheinlich auch von METAS im Prinzip so durchgeführt wurde. Sie beziehen sich auf Ashby, der beim Übergang vom ECI-System ins rotierende ECEF-System folgende Transformationsformeln benützt:
    t = t´, r = r´, phi = phi´ + omega t´, z = z´
    Diese sind allerdings äquivalent zur Galileitransformation und widersprechen der Lorentztransformation.

    Um dies einzusehen, gehen wir zurück auf kartesische Koordinaten. Im ECI-System gilt:
    x = r cos phi, y = r sin phi und im ECEF-System gilt: x´ = r´ cos phi´, y´ = r´ sin phi´
    Setzt man obige Transformationsformeln ein, so erhält man
    x = x´ cos (omega t´) – y´ sin (omega t´) , y = y´ cos (omega t´) + x´ sin (omega t´)
    Entwickelt man für kleine Zeiten t

  70. @Chrys 11.08.2012, 09:56

    Ich weiß nicht, was mit dieser Software los ist. Nun versuche ich es scheibchenweise:

    Entwickelt man für kleine Zeiten t

  71. @Chrys 11.08.2012, 09:56

    Noch ein Versuch:

    Nun sind wir einen schönen Schritt weitergekommen. Offenbar akzeptieren Sie meinen Vorschlag zur Synchronisierungsprozedur, die höchstwahrscheinlich auch von METAS im Prinzip so durchgeführt wurde. Sie beziehen sich auf Ashby, der beim Übergang vom ECI-System ins rotierende ECEF-System folgende Transformationsformeln benützt:
    t = t´, r = r´, phi = phi´ + omega t´, z = z´
    Diese sind allerdings äquivalent zur Galileitransformation und widersprechen der Lorentztransformation.

    Um dies einzusehen, gehen wir zurück auf kartesische Koordinaten. Im ECI-System gilt:
    x = r cos phi, y = r sin phi und im ECEF-System gilt: x´ = r´ cos phi´, y´ = r´ sin phi´
    Setzt man obige Transformationsformeln ein, so erhält man
    x = x´ cos (omega t´) – y´ sin (omega t´) , y = y´ cos (omega t´) + x´ sin (omega t´)
    Entwickelt man für kleine Zeiten t sehr viel kleiner als 1/omega ( omega t = 5×10^(-6) für die Laufzeit eines GPS-Signals zur Erde) und berücksichtigt man nur Terme 1. Ordnung, so erhält man
    x = x´– y´ (omega t´) , y = y´ + x´ (omega t´)
    Wählt man weiterhin einen lokalen Bereich im Gebiet x´ sehr viel kleiner als r, y´ = r aus und setzt omega r = v, so ergibt sich
    x = x´ – v t , y = y´ + (x´/r) v t , z = z´ , t = t´
    Im Grenzfall r -> unendlich verschwindet die Krümmung der Kreisbahn und die Erdoberfläche nähert sich lokal beliebig nahe einem mit v bewegten Inertialsystem an. Die Transformation ins bewegte System sollte in diesem Grenzfall in die Lorentztransformation übergehen, aber stattdessen bekommt man die Formeln der Galileitransformation
    x = x´ – v t , y = y´, z = z´ , t = t´ .
    Dies widerspricht der Lorentztransformation
    x = gamma (x´ – v t´) , y = y´, z = z´ , t = gamma (t´ – x´ v / c^2) ,
    aus der c = const folgen würde, während die Galileitransformation c +/- v ergibt, durchaus im Einklang mit den Messungen.

    Wenn es also notwendig ist, bei der Synchronisation der Uhren den Sagnaceffekt nach Ashby zu berücksichtigen, wie Sie und ich es glauben, und wie METAS es wohl auch tut, so ignoriert man beim Übergang vom inertialen ECI-System ins bewegte ECEF-System die Lorentztransformation, die nicht nur durch die Messungen von Michelson/Gale, sondern auch durch die jetzigen Neutrino-Laufzeitmessungen widerlegt wird, weil ja die Experimente durch die Galileitransformation richtig beschrieben werden, während die Anwendung von c = const zu Diskrepanzen führt.

    Der junge Dr. Feldmann hat in seiner Unerfahrenheit vermutlich nicht gewusst (so wenig wie das gesamte OPERA-Team), dass die alten Hasen längst Abstand von der Lorentztransformation und damit vom Postulat c = const genommen hatten. Er dachte wohl, dass die Geschwindigkeit eines Funksignals unabhängig sowohl von der Geschwindigkeit des Senders als auch von der Geschwindigkeit des Empfängers sei, wie Einstein vermutet hatte und wie man es noch immer auf der Universität lernt. Der Sagnaceffekt gehört dort kaum noch zum Lehrstoff. So kam es dann zur „Sensation“, dass Einstein durch die Neutrino-Experimente widerlegt sei, obwohl diese Sensation stillschweigend schon 1925 stattgefunden hatte.

  72. @Dr. Wolfgang Engelhardt

    Mir sind, so glaube ich, Ihre Bedenken hinsichtlich der Lichtgeschwindigkeit durchaus begreifbar. Versuchen wir einmal, das noch zu klären.

    Wenn wir o.B.d.A. die ECI-Koordinatenzeit t als Eigenzeit einer in ECI ruhenden Uhr annehmen(*), dann lassen sich Ihre Betrachtungen auch gemäss der SR durchführen, was uns in den Formeln den besagten γ-Faktor einbringt und somit zu einer Lorentz Transformation führt. Diese lässt die Minkowski Metrik invariant, und folglich gilt im System der tangentialen Uhr U’ für die Lichtgeschw. c = const. Für die kreisende Uhr U muss dies dann jedenfalls im Berührungspunkt von Kreis und Kreistangenete auch erfüllt sein, und eine anisotrope Lichtgeschw. c ± v scheint in der Nähe dieses Punktes offensichtlich ausgeschlossen. Zudem lässt sich diese Überlegung für jeden Punkt auf der Kreisbahn ganz entsprechend anstellen — also nichts mit Sagnac!?

    Der subtile Pferdefuss bei der Sache ist, dass eine geometrische Eigenschaft, die lokal bei einem jedem Punkt gegeben ist, nicht zwangsläufig global gelten muss (in dem Sinne, wie ein Möbiusband zwar lokal bei jedem Punkt orientierbar ist. aber eben nicht global). Ihre Überlegungen demonstrieren für den Berührungspunkt von Kreis und Kreistangente recht augenfällig, was ich Herrn Wappler oben schon nahebringen wollte, nämlich dass es in der GR stets möglich ist, lokal bei einem Punkt p Koordinaten so einzuführen, dass die Metrik in p gerade als die Minkowski Metrik erscheint, womit lokal fur die Lichtgeschw. c = const. angenommen werden kann. Aber global ist die Lichtgeschw. in der GR i.a. nicht konstant.

    Voll relativistisch. also unter Verwendung der Eigenzeit τ statt der Koordinatenzeit t = t’, präsentiert sich Ashbys Transformation als

    Φ : t = γτ, r = r’, φ = φ’ + ωγτ, z = z’.

    Die mittels der Abb. Φ zurückgezogene Minkowski Metrik g ist dann

    h := Φ*g = (dτ − r²ωγ dφ’)² − (dr’)² − r²γ² (dφ’)² − (dz’)²,

    wo ich bei den Koeffizienten der besseren Lesbarkeit wegen schon r = r’ benutzt habe. Per constructionem ist h isometrisch zu g, und freilich nichts anderes als die schon bekannte Metrik g’ ausgedrückt mit der Eigenzeit τ. Für die am Radius r herumlaufenden Lichtkurven finden wir nun die Diff.gl.

    (dα/dτ + ωγ)² = γ²/r²

    mit den co- und contra-rotierenden Lösungen zur Angfangsbedingungen α(0) = 0,

    (α_+)(τ) = (1 − rω) γτ/r und (α_-)(τ) = −(1 + rω) γτ/r,

    Die Lichtgeschw. mit Bezug zur Eigenzeit ist dabei (1 ± rω) γ, bis auf den γ-Faktor also alles wie gehabt. Die globale Betrachtung liefert folglich etwas anderes, als Sie aufgrund der lokalen Betrachtung erwarten.

    Was aber ist die Ursache dafür? Nun, bei der lokalen Linearisierung wird etwas unterschlagen, nämlich die Coriolis Beschleunigung und das Zetrifugalpotential. Die sind in der Metrik h geometrisch verwurstet, doch das wird nicht mehr erkennbar in der lokalen Darstellung, die Sie bei der Linearisierung mit der Lorentz Transformation effektiv einführen. Wenn Sie die Coriolis Beschleunigung bei der relativistischen Betrachtung korrekt einkalkulieren, dann finden Sie auch hier den Sagnac Effekt.

    Ich stimme Ihnen dahingehend zu, dass Lehrbücher oft genug nur ein Beleg für die Armseligkeit der didaktischen Fähigkeiten ihrer Autoren sind. Hinsichtlich der einschlägigen Kompetenz von Leuten wie Dr. Feldmann, die an metrologischen Instituten in der Abteilung “Time and Frequency” in der Praxis mit diesem Zeug umgehen, bin ich allerdings deutlich optimistischer als Sie.

    (*) Die GPS-Zeit ist eine auf TAI basierte Zeitskala und entspricht damit nicht der Eigenzeit von in ECI ruhenden Uhren. Es müsste nochmals mit einem konstanten Faktor umskaliert werden, um dies alles auf die GPS-Zeit übertragen zu können. Aber das ist ein technischer Aspekt, der am Prinzip nichts ändert, und es sei nur der Vollständigkeit halber wenigstens einmal erwähnt.

  73. Test Software und Preview

    Nachdem in diesem Blog noch immer kein Preview vorliegt und das Symbol für „sehr klein gegen“ den Eintrag unterbricht, muss ich erst mal testen, ob man mit < < arbeiten kann.

  74. @Chrys

    Sie schreiben: „…was uns in den Formeln den besagten ³-Faktor einbringt und somit zu einer Lorentz Transformation führt.“ Sie befinden sich im Irrtum, wenn Sie glauben, der gamma-Faktor habe etwas mit Voigts Postulat c = const zu tun. Näheres finden Sie hier:
    http://www.kritik-relativitaetstheorie.de/…n.pdf oder hier: http://www.kritik-relativitaetstheorie.de/…T.pdf
    Es kommt also allein auf den in v linearen Term x v/c^2 an, den Voigt hergeleitet hatte, um c = const für jede Wellenausbreitung in elastischen Medien postulieren zu können. In der Akustik wurde dieser Term wegen seiner Unsinnigkeit ignoriert, in der SRT lebt er in der Lorentztransformation zwischen Inertialsystemen fort. (Sie allerdings setzen ihn in Ihren Formeln, die Sie „relativistisch“ nennen, zu Null. Ashby natürlich auch, denn er verwendet ja die Galileitransformation.)

    Sie können getrost gamma = 1 setzen (was im praktischen Fall ohnehin gilt), so wie Ashby das auch tut, und erhalten damit den korrekten, experimentell bestätigten Sagnaceffekt. Dagegen erhalten Sie im Grenzübergang zu verschwindender Bahnkrümmung ( r -> unendlich ), d.h. verschwindender Beschleunigung, nicht die Lorentztransformation der Zeit t = gamma (t´ – x´ v / c^2), obwohl im Gebiet x´ < < r, in dem sich z.B. das Michelson/Gale Experiment befand, ein Inertialsystem vorliegt, wie oben demonstriert. Die Erdoberfläche bewegt sich in der Gegend bei Chicago um ca. 3 Millimeter auf einer Kreisbahn, während das Licht Michelsons Interferometer in entgegengesetzten Richtungen umkreiste und zu unterschiedlichen Zeitpunkten am Ausgangsort wieder ankam. Die Abweichung der Kreisbahn von einer geraden Linie war in diesem Fall etwa ein Hundertstel Atomdurchmesser. Hier noch von einem „beschleunigten System“ zu sprechen, welches den Term x´ v / c^2 zum Verschwinden bringt, wäre grotesk. Also nichts mit der SRT, welche c = const fordert, während c +/- v gemessen wurde.

  75. @Dr. Wolfgang Engelhardt

    Ganz gewiss können Sie sogar ohne den Gebrauch von Papier und Bleistift sehen, dass im SR Formalismus die Linearisierung im Berührungspunkt herauskommt als ein “Lorentz boost” L mit der Geschwindigkeit v = rω entlang der Kreistangente. Folglich lässt L die Minkowski Metrk invariant, g(Lu,Lv) = g(u,v), und mithin gilt dabei die Konstanz der Lichtgeschwindigkeit. Klarer Fall, so kommt Ashby nicht zum Sagnac Effekt. Das behauptet er aber auch gar nicht, denn er resumiert ja in “The Sagnac Effect in the Glopbal Positioning System“,

    In the rotating frame of reference the effect appears to arise from a Coriolis-like term in the fundamental scalar invariant.

    Allfällige “Coriolis-like terms” involvieren die Beschleunigung der kreisbewegten Uhr, also auch die zweite Ableitung, die für eine Linearisierung aber gerade verschwindet. Ganz egal ob klassisch oder relativistisch, Ihre Deutung von Ashbys Berechnung kann unmöglich stimmen, wenn Sie dabei immer nur bis zur ersten Ableitung schauen.

    Sie hatten sich weiter oben daran gestört, dass mit t = t’ die “ECI-Zeit” einfach durchgeschleppt wird. Deshalb hatte ich das dann nochmals skizziert, formal ganz im Sinne der GR (und sogar ohne HTML-Formatierungsfehler) mit der “ECEF-Eigenzeit” τ (d.h., τ gibt mit Bezug auf die Metrik h eine Parametrisierung nach Bogenlänge für die τ-Koordinatenlinien). Für die am Radius r mit und entgegen dem Uhrzeigersinn herumlaufenden Lichtkurven findet sich dann die Geschwindigkeit anisotrop als (1 ± rω) γ.

    Die Eigenzeit τ interessiert Ashby allerdings nicht. Er will auf die GPS-Zeit hinaus, und die ist eine TAI-basierte Koordinatenzeit. Die Eigenzeit τ wird man wieder los durch die Substitution τ = t’/γ, was unterm Strich gerade t = t’ bedeutet. Das für seine Zwecke nutzlose rein- und rausrechnen des γ-Faktors kann sich Ashby also schlicht ersparen. Die besagte Anistropie in “ECEF” bleibt bestehen, bezogen auf die Koordinatenzeit t’ ist die Lichtgeschw. nun 1 ± rω. Und das alles ganz ohne Galilei — was verlangen Sie mehr?

  76. @Chrys

    Nein, mehr verlange ich nicht als dass die Galileitransformation in der Form
    x = x´ – v t , y = y´, z = z´ , t = t´

    beim Übergang vom ECI-System ins ECEF-System zum Zwecke der Uhrensynchronisation angewandt wird, so wie Ashby, Sie, METAS und ich es tun. Übrigens „deute“ ich nicht Ashby´s Berechnungen, sondern schreibe seine Transformation
    t = t´, r = r´, phi = phi´ + omega t´, z = z´
    nur in kartesische Koordinaten um und werte sie für den Spezialfall x´/ r unendlich, omega->0, omega r = v noch immer von einem „rotierenden System“ sprechen muss, in dem die SRT mit der Lorentztransformation

    x = gamma (x´ – v t´) , y = y´, z = z´ , t = gamma (t´ – x´ v / c^2)

    NICHT gilt, hätte ich gerne von Ihnen erfahren, aber auch hier trifft zu: Ultra posse nemo obligatur. Für mich ist es klar: Voigts Postulat c = const ist im Falle von Licht genauso unsinnig wie im Fall von Schall, was durch Experimente hinlänglich erwiesen ist.

  77. @Chrys

    Tut mir leid, dass diese unzureichende Software wieder meinen Text verhunzt hat. Leider gibt es noch immer keinen Preview obwohl mehrfach angefordert. Das Ganze noch mal:

    Nein, mehr verlange ich nicht als dass die Galileitransformation in der Form
    x = x´ – v t , y = y´, z = z´ , t = t´

    beim Übergang vom ECI-System ins ECEF-System zum Zwecke der Uhrensynchronisation angewandt wird, so wie Ashby, Sie, METAS und ich es tun. Übrigens „deute“ ich nicht Ashby´s Berechnungen, sondern schreibe seine Transformation
    t = t´, r = r´, phi = phi´ + omega t´, z = z´
    nur in kartesische Koordinaten um und werte sie für den Spezialfall x´/ r sehr klein gegen 1 aus. Wie Ashby zu seinen Gleichungen bei verschwindender Beschleunigung kommt, wo mit omega->0 auch die Corioliskraft und die Fliehkraft verschwinden, ist mir einerseits rätselhaft, andererseits aber ziemlich egal. Wichtig ist nur, dass seine Gleichungen, die der Galileitransformation äquivalent sind, durch die Messungen bestätigt werden.

    Warum man im Grenzfall r->unendlich, omega->0, omega r = v noch immer von einem „rotierenden System“ sprechen muss, in dem die SRT mit der Lorentztransformation

    x = gamma (x´ – v t´) , y = y´, z = z´ , t = gamma (t´ – x´ v / c^2)

    NICHT gilt, hätte ich gerne von Ihnen erfahren, aber auch hier trifft zu: Ultra posse nemo obligatur. Für mich ist es klar: Voigts Postulat c = const ist im Falle von Licht genauso unsinnig wie im Fall von Schall, was durch Experimente hinlänglich erwiesen ist.

  78. @Dr. Wolfgang Engelhardt

    Sagen wir mal so: Die Gleichungen

    (1) t = t´, r = r´, φ = φ´ + ω t´, z = z´

    wären eine “Galilei Transformation” für einen (r,φ,z)-Koordinatenraum, wenn dieser noch mit der Euklidischen Metrik

    dr² + dφ² + dz²

    versehen wird. Das ist aber nicht der Euklidische Raum E, der als geometrischer Hintergrund in der Newtonschen Physik dient, denn dieses E ist zu betrachten mit der Metrik

    η := dr² + r² dφ² + dz² = dx² + dy² + dz².

    Bezüglich (E,η) bedeutet (1) aber eine nichtlineare Transformation, denn φ, φ’, ωt sind hierbei Winkelvariablen, die als Argumente trigonometr. Funktionen erscheinen. Ausserdem ist für die Zylinderkoordinaten eine (r,φz)-Karte schon kein linearer Raum mehr, wegen der notwendigen Beschränkungen für die Winkelbereiche.

    Mit (1) als “Galilei Transformation” im Koordinatenraum kriegt man keine Rotation, nur gleichformige Koordinatentranslation. Insbesondere kriegt man keine “Coriolis-like terms”, woraus wir schliessen sollten, dass Ashby das so gewiss nicht gemeint haben kann.

    In der Euklidischen Ebene hat es zwar Kreise mit beliebig grossem Radius, aber trotzdem keinen mit unendlich grossem Radius, Für ihr Szenario r → ∞, ω → 0 mit rω = v = const. ist da keine Punktmenge, gegen die irgendwas konvergieren kann, Sie müssten durch Kompaktifizierung der Ebene eine solche erst einmal einführen.

    Bevor Sie das womöglich tun, sollten Sie genau überlegen, ob dabei Sinnvolleres herauskommen kann als beim dualen Szenario, wo also r → 0, ω → ∞ bei rω = v = const. Da ist nur eine punkthafte Lücke bei r = 0 zu stopfen. Und nach Ihren Erwartungen sollte das eigentlich doch etwas Vernünftiges liefern. Rein heuristisch rotiert dann dieser hinzugefügte Punkt mit unendlicher Winkelgeschw. um den Ursprung und bewegt sich dabei noch geradlinig und gleichförmig mit einer x-beliebigen Geschwindigkeit v.

    N.B. Sonderzeichen wie griechische Lettern kann man hier als HTML Entities eingeben, also z.B. &gamma; für γ.
    Bei spitzen Klammern ist &lt; statt < generell empfehlenswert, weil die Software oft argwöhnt, dass jemand hier Schadcode als HTML Tag einschleusen will.

    Eine Liste von HTML Entities findet man im Web, z.B. hier:
    http://htmlhelp.com/reference/html40/entities/

    Ach ja, die numerischen Codes der Form &#number; funktionieren meines Wissens hier auch nicht 🙁

    Textattibute lassen sich mit <i&gtitalic Text</i&gt oder <b&gtbold Text</b&gt setzen. Wenn man aber hierbei das schliessende Tag vergisst, dann ist alles Nachfolgende verunstaltet, bis es jemand rapariert.

  79. @Chrys

    Man kann auch so sagen: Natürlich sind Ashbys Formeln:
    t = t´, r = r´, &phi = &phi´ + &omega t´, z = z´
    nichts anderes als die Beschreibung einer Rotation in Euklidischer Geometrie, die man auch in kartesischen Koordinaten als Funktion der Zeit ausdrücken kann, wie oben geschehen. Was dies mit SRT oder ART zu tun haben soll, bleibt sein Geheimnis, doch werde ich mich nicht an der Exegese seiner Schriften beteiligen.
    Was den Grenzübergang &omega -> 0 bei konstanter Tangentialgeschwindigkeit v = &omega r und somit r -> ∞ angeht, so wusste schon Galilei, dass eine ebene Marmortischplatte in Wirklichkeit Teil einer Kugelkalotte sein muss, denn wenn sie strikt eben wäre, würde eine glatte Kugel zum tiefsten Punkt rollen, was sie erfahrungsgemäß nicht tut. Später hat Gauss die deutsche Erdoberfläche mit ebenen Dreiecken, deren Seiten Lichtstrahlen waren, vermessen, was eine sehr gute Näherung war. Eine Graphik auf dem alten 10-DMark-Schein erinnert daran. In diesem Sinn war meine Näherung x´ / r < < 1 gemeint, denn sie gilt im Gebiet, wo sich das Michelson/Gale-Sagnacinterferometer befand und auch noch zwischen CERN und LNGS. Die Geschwindigkeit der Erdoberfläche in Bezug auf das ECI System ist in diesem Gebiet mit hoher Genauigkeit geradlinig, so dass sie von der Bewegung in einem Inertialsystem praktisch nicht zu unterscheiden ist. Dies gilt exakt für r -> ∞ in einem sehr großen Bereich x´. Ich darf daran erinnern, dass auch Einstein in seiner ART eine ähnliche Näherung vorgeschlagen hat, denn sein Äquivalenzprinzip gilt natürlich nur lokal. Es ist unmöglich, Fliehkraft und Corioliskraft auf einer rotierenden Scheibe durch das Gravitationsfeld einer wie immer gearteten Massenverteilung zu simulieren.
    Man ist überrascht, dass aus Ashbys „relativistischen“ Formeln im Grenzfall r -> ∞ die Formel t = t´ nicht in die Formel t = &gamma (t´ – x´ v /c^2) übergeht, die nach Voigt für die Transformation der Zeit zwischen Systemen gelten sollte, die sich mit konstanter Geschwindigkeit v parallel zueinander bewegen. Andererseits beschreiben Ashbys Formeln das, was man tatsächlich misst, nämlich c +/- v und nicht c = const. Nach dieser langen Diskussion überlasse ich es Ihnen, was man dazu sagen soll, denn ich habe mich über den Unsinn der Lorentztransformation bereits hinreichend ausführlich geäußert.
    PS: Herzlichen Dank für Ihre Belehrung zu den HTML-Entities. Als partieller IT-Legastheniker hatte ich keine Ahnung, dass es dgl. gibt. Oben habe ich die neue Fertigkeit schon mal ausprobiert, hoffentlich klappt es! Vielleicht kriegen Sie auch einen Preview hin?

  80. @Chrys

    Mangels Preview wieder eine Korrektur:

    Man kann auch so sagen: Natürlich sind Ashbys Formeln:
    t = t´, r = r´, φ = φ´ + ω t´, z = z´
    nichts anderes als die Beschreibung einer Rotation in Euklidischer Geometrie, die man auch in kartesischen Koordinaten als Funktion der Zeit ausdrücken kann, wie oben geschehen. Was dies mit SRT oder ART zu tun haben soll, bleibt sein Geheimnis, doch werde ich mich nicht an der Exegese seiner Schriften beteiligen.

    Was den Grenzübergang ω -> 0 bei konstanter Tangentialgeschwindigkeit v = ω r und somit r -> ∞ angeht, so wusste schon Galilei, dass eine ebene Marmortischplatte in Wirklichkeit Teil einer Kugelkalotte sein muss, denn wenn sie strikt eben wäre, würde eine glatte Kugel zum tiefsten Punkt rollen, was sie erfahrungsgemäß nicht tut. Später hat Gauss die deutsche Erdoberfläche mit ebenen Dreiecken, deren Seiten Lichtstrahlen waren, vermessen, was eine sehr gute Näherung war. Eine Graphik auf dem alten 10-DMark-Schein erinnert daran. In diesem Sinn war meine Näherung x´ / r << 1 gemeint, denn sie gilt im Gebiet, wo sich das Michelson/Gale-Sagnacinterferometer befand und auch noch zwischen CERN und LNGS. Die Geschwindigkeit der Erdoberfläche in Bezug auf das ECI System ist in diesem Gebiet mit hoher Genauigkeit geradlinig, so dass sie von der Bewegung in einem Inertialsystem praktisch nicht zu unterscheiden ist. Dies gilt exakt für r -> ∞ in einem sehr großen Bereich x´. Ich darf daran erinnern, dass auch Einstein in seiner ART eine ähnliche Näherung vorgeschlagen hat, denn sein Äquivalenzprinzip gilt natürlich nur lokal. Es ist unmöglich, Fliehkraft und Corioliskraft auf einer rotierenden Scheibe durch das Gravitationsfeld einer wie immer gearteten Massenverteilung zu simulieren.

    Man ist überrascht, dass aus Ashbys „relativistischen“ Formeln im Grenzfall r -> ∞ die Formel t = t´ nicht in die Formel t = γ (t´ – x´ v /c^2) übergeht, die nach Voigt für die Transformation der Zeit zwischen Systemen gelten sollte, die sich mit konstanter Geschwindigkeit v parallel zueinander bewegen. Andererseits beschreiben Ashbys Formeln das, was man tatsächlich misst, nämlich c +/- v und nicht c = const. Nach dieser langen Diskussion überlasse ich es Ihnen, was man dazu sagen soll, denn ich habe mich über den Unsinn der Lorentztransformation bereits hinreichend ausführlich geäußert.

    PS: Herzlichen Dank für Ihre Belehrung zu den HTML-Entities. Als partieller IT-Legastheniker hatte ich keine Ahnung, dass es dgl. gibt. Oben habe ich die neue Fertigkeit schon mal ausprobiert, hoffentlich klappt es! Vielleicht kriegen Sie auch einen Preview hin?

  81. @Dr. Wolfgang Engelhardt

    Im wahrsten Sinne des Wortes ist allerdings gerade die Metrik massgeblich dafür, wie aus einer abstrakten Menge von Zahlentupeln ein Modellraum zur geometrischen Beschreibung physikalischer Gegebenheiten wird. Denn insbesondere liefert die Metrik ein Mass für Distanz, sodass die geometrische Einheitslänge mit einer physikalischen Längeneinheit begrifflich verknüpft werden kann. Und falsche Metrik bedeutet dann falsche Physik.

    Es würde Ihnen keine Schwierigkeiten bereiten, mit den Transformationsgleichungen

    t = t’, r = r’, Æ = Æ’ + É t’, z = z’,

    und folglich dt = dt’, dr = dr’, dÆ = dÆ’ + É dt’, dz = dz’, von der Minkowskischen Metrik

    g = dt² – dr² – r² dƲ – dz² = (dt’)² – (dr’)² – (r’ (dÆ’ + É dt’))² – (dz’)²

    nach ein paar weiteren elementaren Ersetzungen und Umformungen zu

    g’ = (1/³ dt – r²É³ dÆ’)² – dr² – r²³² (dÆ’)² – dz²

    zu gelangen. Die Bedingung für die am Radius r bei konstantem z kreisenden lichtartigen Kurven ist dann

    dr/dt = dz/dt = 0, (1/³ – r²É³ dÆ’/dt)² – r²³² (dÆ’/dt)² = 0,

    und Letzteres ergibt (dÆ’/dt + É)² = 1/r², mit den bereits bekannten anisotropen Lösungen. Einfacher kann ich es Ihnen nicht machen.

    Schätzungsweise würden Sie höchstens ein A4 Blatt opfern müssen, um sich eigenhändig davon zu überzeugen, dass es dabei nirgendwo eine Transformation zwischen Inertialsystemen braucht. (ECI ist ja auch das einzige Inertialsystem, das bei Ashby ins Geschehen verstrickt ist.) Probieren Sie doch vielleicht einfach mal aus.

  82. Korrektur

    Ein Experiment mit Formatierung is wieder misslungen. Versuche es nochmals mit dem mittleren Teil:

    Es würde Ihnen keine Schwierigkeiten bereiten, mit den Transformationsgleichungen

    t = t’, r = r’, φ = φ’ + ω t’, z = z’,

    und folglich dt = dt’, dr = dr’, dφ = dφ’ + ω dt’, dz = dz’, von der Minkowskischen Metrik

    g = dt² – dr² – r² dφ² – dz² = (dt’)² – (dr’)² – (r’ (dφ’ + ω dt’))² – (dz’)²

    nach ein paar weiteren elementaren Ersetzungen und Umformungen zu

    g’ = (1/γ dt – r²ωγ dφ’)² – dr² – r²γ² (dφ’)² – dz²

    zu gelangen. Die Bedingung für die am Radius r bei konstantem z kreisenden lichtartigen Kurven ist dann

    dr/dt = dz/dt = 0, (1/γ – r²ωγdφ’/dt)² – r²γ² (dφ’/dt)² = 0,

    und Letzteres ergibt (dφ’/dt + ω)² = 1/r², mit den bereits bekannten anisotropen Lösungen. Einfacher kann ich es Ihnen nicht machen.

  83. @Chrys

    Sie müssen mir nicht immer aufs Neue den Sagnaceffekt vorrechnen. Das konnte Sagnac schon 1913 auf der Basis der Äthertheorie, Michelson/Gale konnten es 1925 für ein auf der Erde feststehendes Interferometer, Ashby kann es für das GPS unter Voraussetzung einer universellen Zeit t = t´, die sowohl im ECI-System als auch im bewegten Erdsystem gilt, und ich kann es übrigens auch.

    Was aber weder Sie noch ich zustande bringen ist zu zeigen, wie die Zeittransformation t = t´ im Grenzfall verschwindender Beschleunigung in den Ausdruck t = γ (t´ – x´ v /c^2) übergeht, den die Lorentztransformation fordert, wenn zwei beschleunigungsfreie Systeme, die man auch Inertialsysteme nennt, sich relativ zueinander mit konstanter Geschwindigkeit v bewegen. Dieser Fall liegt in extrem guter Näherung vor, während das Licht das Michelson/Gale Interferometer umkreist. Dennoch bleiben die Umlaufzeiten rechts und links herum unterschiedlich, egal wie groß Sie den Erdradius bei fester Geschwindigkeit wählen. Damit wurde die Gültigkeit der Lorentztransformation eindrucksvoll bereits in erster Ordnung widerlegt und jede präzise GPS-Messung, die den Sagnaceffekt berücksichtigt, tut dies aufs Neue.

    Danke für Ihre Beiträge zu dieser Diskussion, bei der Sie zugegebenermaßen gar nicht erst den Versuch gemacht haben, die Lorentztransformation – insbesondere die Zeitransformation t = γ (t´ – x´ v /c^2) bei verschwindender Beschleunigung – zu retten, aus der c = const folgen würde.

  84. @Dr. Wolfgang Engelhardt

    » … und ich kann es übrigens auch.«

    Ausgezeichnet. Aber eigentlich ging es dabei auch speziell um diese Ihre Bemerkung (21.08.2012, 16:37),

    »Was dies mit SRT oder ART zu tun haben soll, bleibt sein [Ashbys] Geheimnis, …«

    Wie Ashby mit seiner ECI → ECEF Transformation und der Raumzeit-Geometrie zum Sagnac-Effekt gelangen kann, erachte ich jetzt mal als geklärt. Und dazu benötigt Ashby keine versteckten Transformationen zwischen Inertialsytemen (egal ob Galilei oder Lorentz), noch braucht er einen Aether.

    Von einem “Grenzfall verschwindender Beschleunigung” kann aber nicht die Rede sein, wenn rω = v > 0 entlang einer Folge von Punkten mit r → ∞, ω → 0 gilt. An all diesen Punkten wirkt dann dieselbe Coriolis Beschleunigung, die verschwindet also für positives v nicht. Zwar haben Sie an allen Punkten dieser Folge die gleiche Linearisierung, diese Linearisierungen konvergieren damit trivialerweise. Aber aus dem Limes von Linearisierungen lässt sich hier nicht auf die Linearisierung an einem Limes schliessen. Für r → ∞ divergiert die Punktfolge ganz einfach und entschwindet in der Unendlichkeit.

    Doch selbst dort, wo eine Limesmenge gegeben ist, wie im entsprechenden Fall mit r → 0, ω → ∞, lassen sich Limes und Linearisierung nicht bedenkenlos vertauschen. Für v > 0 ist der Limes der Linearisierungen doch recht offensichtlich ungleich der Linearisierung am Limes.

    Sie können absolut gewiss sein, dass sich mit derlei Überlegungen die Lorentz Transformation nicht “widerlegen” lässt. Da verschwenden Sie nur Ihre Zeit, wenn Sie es versuchen.

  85. @Chrys

    Natürlich benötigt Ashby eine Transformation zwischen ECI und ECEF. Sie lautet für die Zeit: t´ = t. Er benötigt tatsächlich keine „versteckte“ Lorentztransformation zwischen Inertialsytemen, denn er bedient sich genau wie die alte Äthertheorie der universellen Newtonschen Zeit t, die in jedem System gilt, egal ob und wie es sich bewegt. Weder er noch Sie bemerken, dass sich ECEF relativ zu ECI lokal wie ein Inertialsystem bewegt, jedenfalls während der 10 Mikrosekunden, auf die es beim Michelson/Gale Versuch ankommt. Auch bei der Uhrensynchronisation, wo sich die Erde um 20 m zwischen Aussendung und Empfang des Synchronisationssignals bewegt, ist die Bahn bis auf 50 Mikrometer geradlinig. Beschleunigungen spielen bei großem Radius und konstanter Geschwindigkeit keine Rolle, denn die Corioliskraft ist proportional ω v und für die Fliehkraft gilt r ω^2 = ω v . Beide Kräfte verschwinden also mit ω → 0. Diese Näherung ist im gegebenen Fall sehr gut erfüllt.

    Nicht ich widerlege die Lorentztransformation, sondern Sie bzw. Ashby tun es mit t = t´ statt t = γ (t´ – x´ v /c^2) anzuwenden, wobei Sie (und ich auch), nicht aber die SRT vom Experiment bestätigt werden. Tatsächlich rate ich Ihnen nicht, mit Hilfe des GPS das Postulat c = const bestätigen zu wollen. Sie würden Ihre Zeit verschwenden.

    Übrigens: Falls es wirklich nicht möglich ist – wie Sie zu glauben scheinen – einen lokalen Grenzübergang zwischen einer gekrümmten und einer geraden Bahn zu finden, dann bedeutet dies das praktische Aus für die SRT, denn es gibt nur gekrümmte Bahnen in der Realität. Abgesehen davon, dass sich die Erde um die eigene Achse dreht, kurvt sie ja auch um die Sonne herum und diese um das Zentrum der Milchstraße, welches wahrscheinlich auch nicht geradlinig durch das Weltall pflügt. Vergessen wir also die Lorentztransformation und verwenden wir immer die Newtonsche Zeit t, die Ashby, Ihnen und mir so gute Dienste leistet.

  86. @Dr. Wolfgang Engelhardt

    Von einem “Prinzip der Konstanz der Lichtgeschwindigkeit” zu reden erweist sich a posteriori als eine recht unglückliche Ausdrucksweise. In §22 seines Büchleins Über die spezielle und die allgemeine Relativitätstheorie schreibt Einstein ja selbst,

    […] daß nach der allgemeinen Relativitätstheorie das schon oft erwähnte Gesetz von der Konstanz der Vakuumlichtgeschwindigkeit, das eine der beiden grundlegenden Annahmen der speziellen Relativitätstheorie bildet, keine unbegrenzte Gültigkeit beanspruchen kann.

    In den drei oder vier mir nun bekannten Texten von Neil Ashby spricht er zwar auch gelegentlich von Konstanz der Lichtgeschw., er sagt aber auch ganz klar, dass dies nicht für ECEF gilt, nur für ECI. Die GPS-Zeitexperten an den nationalen metrologischen Instituten werden wohl nicht so sehr viel falsch machen können, wenn sie sich an Ashbys Instruktionen halten. Und an Ashby kommt beim Thema GPS-Zeit anscheinend niemand vorbei.

    Die GPS Synchonisierung ist freilich nicht als ein Test für SR oder GR konzipiert, denn GPS-Zeit ist eine Koordinatenzeit. Im Unterschied zu Einsteins Uhren zeigen GPS-Uhren nicht ihre eigene Zeit an, sondern nominell die Eigenzeit einer idealiserten, auf dem rotierenden Geoid fixierten Uhr. Entsprechendes gilt für alle Zeitskalen mit der TAI-Sekunde als Einheit. Es wird dabei also ein bestimmtes Bezugssystem willkürlich gesetzt, dessen Uhren den globalen Takt u.a. für die zivile Zeitmessung auf diesem Planeten vorgeben. Aber nicht, weil die Relativität total belanglos wäre (die steckt in der Def. der TAI-Sekunde ohnehin drin), sondern vielmehr aus rein pragmatischen Gründen.

    Eine echte Anwendung relativistischer Zeitmessung wäre hingegen die uhrenbasierte Geodäsie, wie sie durch die Fortschritte bei der Entwicklung optischer Uhren mittlerweile in durchaus realistische Nähe gerückt ist. Aber das ist eine andere Baustelle.

  87. Through the Looking-Box

    Chrys schrieb (12.08.2012, 14:41):
    > Es existiert noch eine Welt jenseits der Box 13.1 [MTW] …

    Die Metaphor der Box 13.1 als Fenster zwischen verschiedenen (Betrachtungs-)Welten (“jenseits” bzw. “diesseits”) ist ganz passend:

    nämlich einerseits die Welt der Distanz-Verhältnisse (bzw. insbesondere auch entsprechend Einsteins und Synges Auffassung, der Dauerverhältnisse, mit den entsprechenden Verallgemeinerungen),
    einschließlich der (für Physiker selbstverständlichen und in Einsteins Formulierung schon mehrfach zitierten) Voraussetzung einer nachvollziehbaren Methode zur Feststellung solcher verhältnisse;

    und andererseits die Welt der Koordinaten, bis hin zu irgendwelchen bestimmten Koordinaten-Transformationen (die offenbar in der Korrespondenz mit Dr. Wolfgang Engelhardt auftauchen).

    Box 13.1 illustriert (SWIV) insbesondere:
    die Welt der Distanz-Verhältnisse (und deshalb erst recht die Welt ihrer Messdefinition(en), d.h. die Welt der RT) ist vorrangig;
    während die Welt der Koordinaten (einschl. entsprechender Zahlen “g<sub>μν</sub>) lediglich ein mehr oder weniger “gut”-es Abbild darstellt.

    > Ein Hinweis auf MTW Formel 13.14 hätte hier möglicherweise mehr gebracht.

    Diese Formel lautet (im Wesentlichen):

    g<sub>Overhat[ α ] Overhat[ &beta ];</sub> a
    e<sub>Overhat[ α ]</sub> · e<sub>Overhat[ β ]</sub> =
    … (irgendwelche bestimmte reelle Zahlen)

    Die Symbole e sollen dabei offenbar Elemente einer Menge von “orthonormal basis vectors” bedeuten.
    Es stellt sich also (für Physiker selbstverständlich) die Frage:
    Wie ist zu entscheiden, ob zwei gegebene Vektoren “(zueinander) orthonormal” sind, oder nicht?

    Und sofern diese Formel als Hinweis (im Zusammenhang mit 11.08.2012, 08:53) gedacht sein soll:
    Was hätte die SI-“Meter”-Definition damit zu tun; bzw. auch der (dort auftauchende) Begriff “Geschwindigkeit” ?? …

    p.s.
    Chrys schrieb (26.08.2012, 17:21):
    > Eine echte Anwendung relativistischer Zeitmessung wäre hingegen die uhrenbasierte Geodäsie […]

    Dieser verkehrten Blickrichtung möchte ich die richtige/nachvollziehbare entgegensetzen:

    Die relativistische Messdefinition, wie “Dauer” (zwischen verschiedenen Anzeigenpaaren von Uhren) zuEine echte Anwendung relativistischer Zeitmessung wäre hingegen die uhrenbasierte Geodäsie messen/vergleichen ist,
    nämlich insbesondere unter Voraussetzung und Berücksichtigung der gemessenen geometrischen Beziehungen der betrachteten Uhren (und weiterer geeigneter Beteiligter) zueinander,
    ermöglicht die Definition des Begriffes “gute Uhr” (vgl. MTW Figure 1.9 und auch § 16.4) bzw. die Messung der “Güte” gegebener Uhren;
    d.h. nicht “uhrenbasierte Geodäsie” ist richtig, sondern geodäsiebasierte Uhren.

    Und das lässt sich übrigens schon anhand der klassischen Experimente von Pound/Rebka (1959) bzw. von Hafele/Keating (1971) diskutieren …

  88. @Frank Wappler / Blickrichtungen

    »… (irgendwelche bestimmte reelle Zahlen)«

    Sogar ganz bestimmte reelle Zahlen. Die Komponenten der Minkowskischen Metrik. Wenn jetzt kein Licht aufgeht, dann wird es dunkel bleiben.

    »Dieser verkehrten Blickrichtung möchte ich die richtige/nachvollziehbare entgegensetzen:«

    Optische Uhren und ihre Anwendungen gehören zwar zu dem Themenschwerpunkt, dem das irgendwo oben verlinkte Heft der PTB-Mitteilungen gewidmet ist, doch hat dies mit den Fragen zur GPS Synchronsierung nichts mehr zu tun. Die richtige Blickrichtung für Beschwerden wäre ergo die Richtung nach Braunschweig.

  89. Quasi-Bestimmtheit

    Chrys schrieb (28.08.2012, 07:28):
    > Sogar ganz bestimmte reelle Zahlen. Die Komponenten der Minkowskischen Metrik.

    Das meint offenbar “Metrik” im Sinne von http://de.wikipedia.org/…etrik#Metrischer_Tensor in irgendeiner bestimmten Koordinaten-Darstellung;

    nicht zu verwechseln mit “Metrik” im Sinne von http://de.wikipedia.org/…/Metrik#Metrischer_Raum bzw. Verallgemeinerungen (insbesondere zu Quasimetrik
    und/oder zu Funktionen s: X * X → R, deren Wertebereich auch negative reelle Zahlen umfasst).

    (Wenn jetzt nicht allmählich der Groschen fällt, dann wurde wohl keiner eingeworfen … &)

    > Optische Uhren und ihre Anwendungen gehören zwar zu dem Themenschwerpunkt, dem das irgendwo oben verlinkte Heft der PTB-Mitteilungen gewidmet ist […] Die richtige Blickrichtung für Beschwerden wäre ergo die Richtung nach Braunschweig.

    Leider ist in Braunschweig offenbar (noch?) keine Möglichkeit vorgesehen, Kommentare, Fragen oder gar Beschwerden zu PTB-Veröffentlichungen ebenso öffentlich und auffindbar vorzubringen.

    Da lob ich mir (zumindest vergleichsweise) den SciLog, einschl. Kommentarfunktion, den wir hier nutzen können.
    Immerhin ist die Behandlung des Themas “Optische Uhren” (bzw. “Lichtuhr) hier ja noch zu erwarten …

  90. @Frank Wappler / Beschwerden

    »Leider ist in Braunschweig offenbar (noch?) keine Möglichkeit vorgesehen, Kommentare, Fragen oder gar Beschwerden zu PTB-Veröffentlichungen ebenso öffentlich und auffindbar vorzubringen.«

    Die Jeanne d’Arc der deutschsprachigen Relativitätskritik schafft das ganz locker auch ohne ein eigens für Beschwerden implementiertes Web Interface. Und spätestens, wenn die Geodäsie sich offiziell relativistischer Methoden bedient, wird sie sich bestimmt der Angelegenheit annehmen. Wait and see 😉

  91. Tut sie’s noch nicht, dann mögen …

    … wir sie dazu bringen …

    Chrys schrieb (28.08.2012, 12:45):
    > Und spätestens, wenn die Geodäsie sich offiziell relativistischer Methoden bedient, wird […]

    Sollte sie ja wohl höchstoffiziell spätestens seit der Resolution 1 der 17-ten CPGM, 1983.

    p.s.
    > Die Jeanne d’Arc der deutschsprachigen Relativitätskritik schafft das ganz locker auch ohne ein eigens für Beschwerden implementiertes Web Interface.

    Das finde ich jedenfalls auch überhaupt nicht von da aus, wohin du mich bzw. jeden Leser verwiesen hast (per 28.08.2012, 07:28 bzw. 01.08.2012, 22:45):
    etwa durch http://www.ptb.de/cms/suche.html?Relativitätskritik

    Weniger als die Relativitätskritik interessiert mich jedoch, ob die nachvollziehbare Begründung und Darstellung der RT “das ganz locker schaffen” könnte …

    p.p.s.
    Mittlerweile ist mir aufgefallen, dass mit “Optischen Uhren” (im Sinne des o.g. PTB-Artikels) keineswegs unbedingt “Lichtuhren” im Sinne von MTW § 16.4 bzw. im (noch festzulegenden) Sinne dieses SciLogs gemeint sind. Folglich kann (und als Physiker sollte) man hinsichtlich dieser “Optischen Uhren” die selbe (Rabi-)Frage stellen, an der Essen gescheitert ist.

  92. Bad einstein-online.info link at ptb.de

    Aus den Webseiten der PTB:

    Einstein und die Zeit – das ist ein viel diskutiertes und gleichzeitig nur selten wirklich verstandenes Thema. Wer mehr wissen will, schaue einmal ins Internet-Portal Einstein-online.info.

    Die Notfallplanung der PTB sieht anscheinend vor, dass in relativitätskritischen Fällen ein Spezialist eingreift.

    @Markus Pössel
    Der Einstein-online.info Link wurde “as is” übernommen von
    http://www.ptb.de/…/wegweiser/fragenzurzeit.html
    Das führt aktuell zu “Die Seite existiert leider nicht…” Vielleicht sollte das jemand mal reparieren. (Sonst heisst es hinterher noch, Dr. Markus Pössel verweigert Kontaktaufnahme.)

  93. Good bipm.org link at scilogs.de

    Frank Wappler schrieb (28.08.2012, 16:53):
    > […] spätestens seit der Resolution 1 der 17-ten CPGM, 1983.

    Sollte richtiger heißen:
    “Die Geodäsie sollte sich ja höchstoffiziell spätestens seit der Resolution 1 der 17. CGPM, 1983
    relativistischer Methoden bedienen.”
    (Die dem Link zugrundeliegende URL war und bleibt unverändert wie beabsichtigt.)

    p.s.
    Anhand der Absendung dieses (wie auch jedes anderen) Kommentars lässt sich prüfen, ob und wann eine entsprechende Eintragung in der Liste “LETZTE KOMMENTARE” auf https://scilogs.spektrum.de/wblogs/summary.php erfolgt;
    im Vergleich mit der (eventuellen) Veröffentlichung dieses Kommentars auf dieser SciLogs-Seite selbst, und nicht zuletzt auch im Vergleich zu einer eventuellen entsprechenden Eintragung in
    der Liste “AKTUELLE KOMMENTARE” auf https://scilogs.spektrum.de/

    Falls eine oder beide diese Eintragungen nicht erfolgen sollten und demnach ein Anlass für entsprechende Reparaturen bestünde, möchte ich hiermit darum bitten.
    (Am Ende hieße es sonst noch, auch Dr. Markus Pössel unterbinde wissenschaftliche Korrespondenz …)

  94. @Frank Wappler

    Die Beschwerden zum SI Meter(1983) waren ja auch gekommen. Wenn nicht zum BIPM, so doch zu MP. Nachzulesen etwa in den Kommentaren hier: https://scilogs.spektrum.de/…11-05-30/gute-maszeinheit

    Die Def. von 1983 war wohl eher dazu gedacht, statt der unhandlichen Krypton 86 Wellenlänge diverse laser-interferometrische Methoden für die Realisierung des Meters einsetzen zu können. Vergleichsweise unspektakulär.

    Die uhrenbasierte Geodäsie ist aber etwas, was es nach Meinung der unorthodoxen Kritiker, die an eine absolute Zeit glauben, überhaupt nicht geben dürfen sollte.

  95. Drama, baby!

    Chrys schrieb (30.08.2012, 20:17):
    > Die Def. von 1983 war wohl eher dazu gedacht, statt der unhandlichen Krypton 86 Wellenlänge diverse laser-interferometrische Methoden für die Realisierung des Meters einsetzen zu können.

    Na gut, so nüchtern kann man das wohl ausdrücken.
    Sofern man den Begriff “interferometrische Methoden” weit genug auffasst, der in der Definition ja nicht ausdrücklich erwähnt wird (und seinerseits darum mit einer ausdrücklichen nachvollziehbaren Definition ausgestattet wäre).
    Methoden wie Synges “Five-point curvature detector” bzw. die Methode, die von Einstein schon 1905 impliziert wurde (nämlich schlicht die Ping-Dauer zwischen geeigneten Beteiligten “A und
    B” als deren “Distanz” zu nehmen und zur Unterscheidung von anderen Dauer-Feststellungen noch mit einem formalen Vorfaktor “1/2 c” zu versehen) sind darin sicherlich eingeschlossen.

    Mit “interferometrische Methoden” verbindet sich wohl insbesondere die Forderung (die in der SI-“Meter”-Definition eben leider nicht erwänt wird), dass die beteiligten Enden (“A
    und B”) und weitere wesentliche am Versuch Beteiligte dabei zueinander “starr” sein und bleiben sollten;
    und wegen der Unidirektionalität der SI-“Meter”-Definition (d.h. dass sie sich nicht auf Ping-Dauern bezieht, sondern verlangt, der Anzeige der Signal-Darstellung des einen Endes und der Anzeige der Signal-Wahrnehmung des anderen Endes zusammen mit einer Dauer zu bewerten) sogar die Forderung, dass die beiden Enden während des Versuchs zueinander ruhten.

    > Vergleichsweise unspektakulär.

    MBMN ganz im Gegenteil. (Ich erinnere mich auch, dass Asher Peres in einem “Leserbrief” in Nature (1983, oder vielleicht 1984?) die Bedeutung dieser “Def. von 1983” hervorgehoben hat; aber weiß im Moment nicht (mehr), ob er die selben Argumente wie die folgenden benutzt oder noch weitere gegeben hat.)

    Denn damit wurde eine Festlegung, die bis dahin von irgendwelchen Artefakten (“Krypton 86“) abhing, zu einer nachvollziehbaren Definition (d.h. zumindest insbesondere sofern
    nachvollziehbar ist bzw. wäre, welche Dauer mit “Sekunde” gemeint ist).

    Die Definition legt nicht nur fest, welche Distanz als “1 Meter” zu bewerten ist, sondern ebenso für jede positive reelle Zahl “r”, welche Distanzen als “r Meter” zu bewerten sind (d.h. sofern nachvollziehbar ist, welche Dauer mit “1 Sekunde” gemeint sei, was die Nachvollziehbarkeit von “r Sekunden” Dauern garantieren würde — dafür hat man ja schließlich Einheiten und reelle Zahlen).

    Und ich wüßte auch nicht, wie ohne Zugrundelegung einer solchen nachvollziehbaren Definition
    zur Ermittlung geometrischer Beziehungen “Krypton 86” als Begriff sonst überhaupt nachvollziehbar gewesen sein sollte, d.h. z.B. wie entschieden worden sein sollte, ob irgend ein “Häufchen” aus n*36 Protonen und n*50 Neutronen und n*36 Elektronen (für positive ganze Zahlen “n”) gerade “n Krypton 86” waren, oder nicht.
    Das ist eben etwas grundsätzlich und spektakulär anderes, als “Krypton 86” als Artefakt zu fordern und einzusetzen (also z.B. einfach ‘ne Gasflasche aufzudrehen, auf die “Krypton 86” gemalt wurde. Wer weiß, was man noch darauf malen könnte.)

    Besonders wichtig ist natürlich auch, das der neben dem Begriff “Sekunde” in der Definition gebrauchte Begriff “Licht” nachvollziehbar ist.

    Mir scheint jedenfalls, dass Leute, die die Bedeutung dieser (Um-)Definition unterschätzen, auch die RT insgesamt nicht richtig einschätzen und vermitteln würden.

    > Die uhrenbasierte Geodäsie ist aber etwas, was es nach Meinung der unorthodoxen Kritiker, die an eine absolute Zeit glauben, überhaupt nicht geben dürfen sollte.

    Das ist erstens ein Anlass, nochmals auf den axiomatischen (selbstverständlich
    nachvollziehbaren) Zeitbegriff der RT hinzuweisen (Ann. Phys. 17, S. 893):

    […] daß ich an Stelle der „Zeit“ die „Stellung des kleinen Zeigers meiner Uhr“ setze

    dass “Zeit” also jedem einzelnen Beteiligten zu eigen ist;
    und dass weitere Definitionen (“Gleichzeitigkeit”, “Dauer”, “Distanz”, …) eine Sache von darauf aufbauenden Festlegungen sind.

    Und zur Phrase “uhrenbasierte Geodäsie” nochmals (und genauer als 27.08.2012, 01:51):
    geodätisch-interferometrische Methoden sind insbesondere entsprechend § 16.4 MTW eine Voraussetzung für die Unterscheidung von “guten Uhren” von “irgendwelchen Uhren” (nicht anders
    herum);
    aber natürlich ist andererseits die (axiomatische) Fähigkeit jedes Beteiligten, seine Anzeigen zu ordnen, die Voraussetzung für Geodäsie.

    > Die Beschwerden zum SI Meter(1983) waren ja auch gekommen […]:
    https://scilogs.spektrum.de/…11-05-30/gute-maszeinheit

    Man mag ja denen, an die Beschwerden bzw. Fragen offentlich gerichtet werden, nicht unbedingt vorgreifen. Aber man muss ja auch nicht (im Sinne von Planck) erst Gras über sie wachsen lassen …
    Im Übrigen sind meine Beschwerden bzw. Fragen hinsichtlich des SI-“Meters” eher damit beschäftigt, ob und in wie weit die Definition seit 1983 in verschiedenen Experimenten (wie CNGS) überhaupt angewendet wurde.

  96. @Frank Wappler

    Der SI Meter von 1983 ist prinzipiell nicht relativistischer als der von 1960. Beide sind gleichermassen elektrodynamisch, und es war Maxwell, dessen Gleichungen die Lichtgeschw. in vacuo, c, als charakteristisch für elektromag. Wellen kennzeichnen. In der Beziehung c = λν bestimmen je zwei gegebene Grössen die dritte, und davon macht man halt Gebrauch. Die Maxwellschen Gleichungen waren von Beginn an Lorentz-covariant formuliert worden, und in diesem Sinne sind sie naturgemäss relativistisch.

    Die uhrenbasierte Geodäsie wird nicht interferometrisch. Im Prinzip ist es wohl so, dass man ein Paar Zwillingsuhren nimmt, die erste auf das Geoid und die zweite irgendwo anders hinsetzt. Dann wird nach einem bestimmten Modus zum Uhrenvergleich bei beiden der kleine Zeiger auf Null gestellt. Nach einer Weile werden die Zeigerstellungen nach demselben Modus wieder miteinander verglichen, und aus dem Unterschied lässt sich der Abstand der zweiten Uhr vom Geoid bestimmen. Wenn die Genauigkeit der Uhren bei 10^{−18} liegt, dann kriegt man diesen Abstand auf 1 cm genau. Technische Probleme bereiten dabei meines Wissens inzwischen weniger die Uhren als die Qualität der Signalübertragung über grosse Distanzen.

  97. @Chrys 26.08.2012, 17:21

    Beim “Prinzip der Konstanz der Lichtgeschwindigkeit” geht es nicht um „eine recht unglückliche Ausdrucksweise“, sondern darum, ob dieses Prinzip physikalisch zutreffend ist oder nicht. Voigt hat es 1887 irrtümlich als gültig für jede Art von Wellenausbreitung angenommen. Die Akustiker haben es ignoriert, Einstein nahm es 1905 für den Spezialfall „Licht“ ernst, um genau wie Voigt daraus die Lorentztransformation abzuleiten. Später nahm er hiervon wieder Abstand, wie Sie oben zitieren, allerdings hat er nicht dazu gesagt, dass damit die Grundlage für die Lorentztransformation entfällt. Folgerichtig gibt es dann keine „Relativität der Zeit“ mehr, weshalb Sie und Ashby, sowie das GPS ungeniert eine universale Zeit t sowohl im inertialen als auch im bewegten System verwenden. Die Versuche von Michelson und Gale geben Ihnen Recht. Vor Jahren habe ich mich ebenfalls auf Einsteins variable Lichtgeschwindigkeit bezogen, als ich versuchte zu verstehen, weshalb Pound und Rebka nur den halben Effekt gemessen haben: http://www.ekkehard-friebe.de/lightvelocity1.pdf

    GPS-Uhren ticken mit einer bestimmten Frequenz, die für alle Satelliten dieselbe ist, falls man sie vom Erdmittelpunkt aus beobachtet, denn sie haben alle dieselbe Relativgeschwindigkeit zu diesem Punkt und auch in etwa denselben Abstand. Anders ist es allerdings für Satelliten untereinander, glaubt man der SRT, die die Uhrenfrequenz von der Relativgeschwindigkeit abhängig macht. Betrachtet man zwei antipodische Satelliten A und B, so bewegt sich B mit fast 8 km/s relativ zu A, d.h. seine Uhr geht gegenüber A um ca. 30 Mikrosekunden pro Tag nach. Freilich geht auch A gegenüber B um dieselbe Zeitdifferenz nach, was den ganzen Unsinn der „relativen“ Zeit offenbart, die letztlich auf Voigts Irrtum c = const beruht.

    Die gleiche Überlegung gilt natürlich auch für bodenfeste Uhren, weil sie eine Relativgeschwindigkeit zueinander haben, wenn sie sich auf unterschiedlichen Längengraden befinden. So geht nach SRT die Cäsiumuhr in Boulder gegenüber jener in Braunschweig langsamer, und umgekehrt geht die Uhr in Braunschweig langsamer gegenüber jener in Boulder (auch nach Berücksichtigung der Höhendifferenz). Also doch beide gleich schnell, wie der imaginäre Beobachter im Erdmittelpunkt konstatieren würde? Aus diesem Dilemma rettet uns nur die universelle Newtonsche „Weltzeit“ UTC und die praktische Ignorierung des inzwischen ideologisierten Postulats c = const. Es freut mich, dass Sie dies inzwischen eingesehen haben, indem Sie sich der Ashby´schen Universalzeit t = t´ bedienen.

  98. @Dr. Wolfgang Engelhardt

    Statt vom “Prinzip der Konstanz der Lichtgeschwindigkeit” hätte Einstein womöglich besser vom “Prinzip der Invarianz von Lichtkegeln” reden sollen. Das gilt dann uneingeschränkt auch in der GR. Und überhaupt ist das Wort “Konstanz” in diesem Zusammenhang deplaziert, denn es geht dabei eigentlich um Invarianten der Lorentz Gruppe, nicht um Konstanten. Aus meiner Erinnerung muss ich zudem sagen, dass es nicht zuletzt diese schier unbegreifliche “Konstanz der Lichtgeschwindigkeit” war, an der meine eigenen frühen Bemühungen, als ca. 14jähriger Schüler etwas von der RT zu begreifen, kläglich gescheitert waren.

    Erdfeste Uhren in Boulder und Braunschweig haben ja feste raumartige ECEF Koordinaten. Alle rotationsbedingten Wirkungen auf die Eigenzeit der Uhren resultieren dann aus dem Zentrifugalpotential, das wiederum geometrisch in der co-rotierenden Metrik aufscheint. In unserer rotierenden Zylinderwelt treten auch keine SR Effekte auf für die Anzeigen zweier Uhren, die sich bei unterschiedlichem “Längengrad” φ’, aber gleichem Radius r’ befinden. Entsprechendes lässt für zwei erdfeste Uhren auf dem rotierenden Geoid feststellen.

    Interessieren würde mich aber noch, inwieweit Sie sich beeindrucken lassen von den in den zurückliegenden Jahren erzielten Fortschritten bei der Genauigkeit von Uhren. Wenn Sie tatsächlich so von der Newtonschen Weltzeit überzeugt sind, dann sollten Ihnen — als Physiker — diese neuen Uhren doch allmählich zu denken geben.

  99. @Chrys

    Es ehrt Sie, dass Ihnen der Unsinn der SRT, der auf einem Gedankenfehler von Voigt beruht, mit vierzehn Jahren, wo die meisten Menschen noch rational zu denken vermögen, nicht eingeleuchtet hat. Auch nach massiver Indoktrination auf der Universität haben Sie sich so viel gesunden Menschenverstand bewahrt, dass Sie mit Ashby t = t´ verwenden und nicht t = γ (t´ – x v/c^2). Für Einstein hat es übrigens keine Rolle gespielt, ob die Geschwindigkeit v geradlinig oder gekrümmt, also beschleunigt ist. Er hat die Formel der SRT kritiklos auch auf rotierende Systeme angewandt, wie Sie sowohl in seiner Arbeit von 1905 als auch in dem von Ihnen oben zitierten Büchlein nachlesen können. Insofern wäre es durchaus berechtigt, von einem wechselseitigen Nachgehen ortsfester Uhren auf unterschiedlichem Längengrad zu sprechen, denn eine Relativgeschwindigkeit besteht ja in diesem Fall unbestreitbar. Freilich demonstriert diese Überlegung nur die logische Aporie des Zwillingsparadoxons, das allein schon die SRT ad absurdum führt.

    Natürlich beeindruckt mich die Ganggenauigkeit moderner Uhren, die die relative Genauigkeit des Mössbauereffekts noch einmal um 3 Größenordnungen übersteigen soll (oder dies schon tut. So sehr genau bin ich doch nicht informiert). Auf diese Weise werden Unstimmigkeiten, die aus der Festsetzung c = 299 792 458 m/s folgen, messtechnisch verifizierbar, was letztlich in irgendeiner Form zu einer Wiedereinführung des zu Unrecht verbannten Urmeters führen könnte. Lassen Sie mich in diesem Zusammenhang über einen Diskurs mit dem ehemaligen Präsidenten der PTB, Professor Göbel berichten.

    Ich hatte angefragt, wie man aufgrund der Neudefinition des Meters über neun Zahlen und die Cäsiumsekunde die Höhe eines Turmes messen solle, da doch die Uhrfrequenz vom Erdpotential abhängt. Prof. Göbel gab zu, dass es unbefriedigend sei, unterschiedliche Höhen zu erhalten, je nachdem, ob man von unten nach oben oder von oben nach unten misst. Allerdings sei der Effekt auch für die größten Wolkenkratzer winzig klein, so dass man sich beim gegenwärtigen Stand der Uhrengenauigkeit keine Gedanken darüber machen müsse. Das könnte sich nun ändern. Ich hatte noch wissen wollen, wie warm das Wasser wird, wenn man von der Höhe des Turmes einen Stein in ein Kalorimeter wirft, denn die Energie des Steines hängt ja von der Fallhöhe ab. Diese Frage hat Prof. Göbel nicht mehr beantwortet…

  100. Rückfall

    Chrys schrieb (01.09.2012, 13:41):
    > […] Wenn die Genauigkeit der Uhren bei 10^{18} liegt, dann […]

    Derlei Argumentation wirft uns zurück zur schon mehrfach angerissenen Frage, wie denn “Genauigkeit” einer bestimmten Uhr (bzw. einer bestimmten reell-wertig und monoton Parametrisierung “t” einer betrachteten Anzeigenfolge) überhaupt bewertet werden sollte und könnte; hinsichtlich eines bestimmten Versuches, an dem diese Uhr (mit Anzeigenfolge “A := { A_ }”) teilnahm.

    So weit ich verstehe, nennt man eine Parametrisierung
    “t: A → R” einer gegebenen Anzeigenfolge “A” “genau”,
    falls für je drei Anzeigen “A_j”, “A_p”, “A_u” gilt:

    As_Dauer[ von A_j bis A_p ] * (t[ A_u ] – t[ A_p ]) ==
    As_Dauer[ von A_p bis A_u ] * (t[ A_p ] – t[ A_j ]).

    (Zwischen zwei “genauen” Parametrisierungen zweier Anzeigefolgen, z.B.”t: A → R” und “θ: B → R”, ergäben sich folglich ebenfalls bestimmte Beziehungen.)

    Jedenfalls wäre zur Ermittlung der “Genauigkeit” einer gegebenen Uhr bzw. Parametrisierung ihrer Anzeigenfolge zunächst die Feststellung der “Dauer”-Verhältnisse zwischen ihren Anzeigenpaaren erforderlich; und das erfordert nicht zuletzt zunächst die
    Feststellung der “Krümmung” der Region mit der betreffenden Uhr, während des betreffenden Versuches.

    Die Methode für Feststellung und Vergleich von “Krümmung” ist sicherlich “geodätisch” zu nennen; und insbesondere für den geschilderten Zweck der Bewertung von “Genauigkeit” muss sie definiert und durchführbar sein, ohne irgendwelche solchen
    Genauigkeits“-Bewertungen schon vorauszusetzen.

    Feststellungen zur “Genauigkeit” sind das letztendliche Ergebnis der Auswertung von Beobachtungsdaten eines bestimmten Versuchs; kein “Zwischenergebniss” oder gar “von vornherein bekannt”.

    (Dasselbe gilt ja beim bekannten Zwillingsexperiment, wo erst durch Vergleich und Berücksichtigung der i.A. ungleichen Dauern der Trennung voneinander letztlich feststellbar
    ist, ob die beiden während ihres Getrenntseins mit gleichen “eigentlichen Raten alterten”, wie man es von Zwillingen vielleicht erwarten mochte, oder in wie fern nicht.)

    > Der SI Meter von 1983 ist prinzipiell nicht relativistischer als der von 1960. Beide sind gleichermassen elektrodynamisch

    Die 1960-er Definition bezieht sich offenbar auf Phasenbeziehungen eines
    elektromagnetischen Feldes” (und ist schon insofern nicht nachvollziehbar, sondern artefaktbasiert; ganz abgesehen von der noch darüber hinausgehenden Forderung nach “Krypton 86“).

    Das fordert die 1983-er Definition dagegen offenbar nicht.”Elektro-magnetismus” spielt dabei höchstens insofern eine Rolle, als viele Beteiligte, deren geometrische Beziehungen zueinander in “Metern” zu bewerten wären, elektro-magnetische Ladungen (oder daraus bestehende
    Systeme) sind. Das Wort “Licht” in der 1983-er Definition versteht sich letztlich im nachvollziehbaren Sinne von “Signal, an sich” (und insbesondere unabhängig davon, ob es mit dem Austausch von Neutrinos assoziiert wäre, oder mit dem Auftreten irgendwelcher “Felder”).

    > […] es war Maxwell, dessen Gleichungen die Lichtgeschw. in vacuo, c, als charakteristisch für elektromag. Wellen kennzeichnen

    Man muss Einstein ja nicht unbedingt das alleinige Verdienst an der RT zuschreiben.
    Welche Messdefinition hat Maxwell wohl seinem Begriff von “Distanz” bzw. dem eng damit verbundenen Operator “&nabla:” zugrundegelegt? Oder seinem Begriff “Brechungsindex”? …

    p.s.
    Frank Wappler schrieb (31.08.2012, 11:51):
    > […] dass Asher Peres in einem “Leserbrief” in Nature (1983, oder vielleicht 1984?) die Bedeutung dieser “Def. von 1983” hervorgehoben hat

    “Defining Length”, Nature 312, 1. Nov. 1984, S. 10

  101. Päng!.

    Dr. Wolfgang Engelhardt schrieb (03.09.2012, 00:41):
    > wie man aufgrund der Neudefinition des Meters über neun Zahlen und die Cäsiumsekunde die Höhe eines Turmes messen solle

    Die geometrische Beziehung zweier Enden, die zwar zueinander starr sind, aber nicht zueinander ruhen, sind streng genommen eben nicht durch einen einzigen gegenseitige Distanz-Wert (“die Höhe“) zu beschreiben, sondern durch zwei (ungleiche) Quasidistanz-Werte; entsprechend den beiden zwar konstanten aber ungleichen gegenseitigen Ping-Dauern.

    Der Versuch einer Bewertung dieser Beziehung als einem Vielfachen von “Metern” entsprechend der “Neudefinition” würde ohne Resultat bleiben, weil die dafür erforderlichen Feststellungen von “Gleichzeitigkeit” zwischen den beiden Enden fehlschlagen (da insbesondere niemand als “Mitte zwischen” den beiden Enden identifizierbar wäre).

    Man gibt dann eben beide (ungleiche) Pingdauern an (meinentwegen auch zusammen mit dem alpha-numerischen Suffix “1/2 q”, zur Unterscheidung von irgendwelchen anderen Dauern, die
    nicht bestimmte Ping-Dauern sind; wobei “q” dann allerdings nur als Wert von “Licht-Quasigeschwindigkeit” erhalten würde, und nicht als Wert von “Licht-Geschwindigkeit”).

    > Allerdings sei der Effekt auch für die größten Wolkenkratzer winzig klein […]

    Allerdings fordert man von Messgrößen Nachvollziehbarkeit, ungeachtet irgendwelcher “Umstände”; z.B. egal ob es sich um die Bewertung eines Wolkenkratzers mit dem unteren Ende
    auf der Erdoberfläche oder auf der Oberfläche eines Neutronensterns handelte.

    > Ich hatte noch wissen wollen, wie warm das Wasser wird, wenn man von der Höhe des Turmes einen Stein in ein Kalorimeter wirft, denn die Energie des Steines hängt ja von der Fallhöhe ab.

    Diese Frage benutzt die (streng genommen) indefinite Phrase “die (Fall-)Höhe“.
    Man könnte/sollte stattdessen in Bezug auf zwei ungleiche Ping-Dauern fragen. (Das Ende mit der größeren Ping-Dauer gilt als “über” dem anderen.)

    > […] da doch die Uhrfrequenz vom Erdpotential abhängt.

    Na ja — bei Pendeluhren (mit gleichen Pendeln) hängt die Pendel-Frequenz sehr deutlich vom (Erd-)Potential ab; bei Cäsium-Atomen (mit gleichen Neutron-Anzahlen usw.) mag die eventuelle Abhängigkeit gewisser Anregungen vom (Erd-)Potential weniger drastisch sein.
    Entscheidend ist: “Frequenz” (als Kehrwert einer bestimmten konstanten Perioden-Dauer) ist doch ebenso wie “Dauer” selbst eine Eigenschaft der Uhr an sich.

    Um die jeweils eigenen “Frequenzen” verschiedener Uhren miteinander zu vergleichen, muss man zunächst die “Potential“-Differenz zwischen ihnen ermitteln und berücksichtigen, bzw. die “Krümmung” der Region, in der beide enthalten waren.

  102. Päng!.

    Dr. Wolfgang Engelhardt schrieb (03.09.2012, 00:41):
    > […] wie man aufgrund der Neudefinition des Meters über neun Zahlen und die Cäsiumsekunde die Höhe eines Turmes messen solle

    Die geometrische Beziehung zweier Enden, die zwar zueinander starr sind, aber nicht zueinander ruhen, sind streng genommen eben nicht durch einen einzigen gegenseitige Distanz-Wert (“die
    Höhe
    “) zu beschreiben, sondern durch zwei (ungleiche) Quasidistanz-Werte; entsprechend den beiden zwar konstanten aber ungleichen gegenseitigen Ping-Dauern.

    Der Versuch einer Bewertung dieser Beziehung als einem Vielfachen von “Metern” entsprechend
    der “Neudefinition” würde ohne Resultat bleiben, weil die dafür erforderlichen Feststellungen von “Gleichzeitigkeit” zwischen den beiden Enden fehlschlagen (da insbesondere niemand als “Mitte zwischen” den beiden Enden identifizierbar wäre).

    Man gibt dann eben beide (ungleiche) Pingdauern an (meinentwegen auch zusammen mit dem alpha-numerischen Suffix “1/2 q”, zur Unterscheidung von irgendwelchen anderen Dauern, die nicht bestimmte Ping-Dauern sind; wobei “q” dann allerdings nur als Wert von
    “Licht-Quasigeschwindigkeit” erhalten würde, und nicht als Wert von “Licht-Geschwindigkeit”).

    > Allerdings sei der Effekt auch für die größten Wolkenkratzer winzig klein […]

    Allerdings fordert man von Messgrößen Nachvollziehbarkeit, ungeachtet irgendwelcher “Umstände”; z.B. egal ob es sich um die Bewertung eines Wolkenkratzers mit dem unteren Ende
    auf der Erdoberfläche oder auf der Oberfläche eines Neutronensterns handelte.

    > Ich hatte noch wissen wollen, wie warm das Wasser wird, wenn man von der Höhe des Turmes einen Stein in ein Kalorimeter wirft, denn die Energie des Steines hängt ja von der Fallhöhe ab.

    Diese Frage benutzt die (streng genommen) indefinite Phrase “die (Fall-)Höhe“.
    Man könnte/sollte stattdessen in Bezug auf zwei ungleiche Ping-Dauern fragen. (Das Ende mit der größeren Ping-Dauer gilt als “über” dem anderen.)

    > […] da doch die Uhrfrequenz vom Erdpotential abhängt.

    Na ja — bei Pendeluhren (mit gleichen Pendeln) hängt die Pendel-Frequenz sehr deutlich vom (Erd-)Potential ab; bei Cäsium-Atomen (mit gleichen Neutron-Anzahlen usw.) mag die eventuelle
    Abhängigkeit gewisser Anregungen vom (Erd-)Potential weniger drastisch sein.
    Entscheidend ist: “Frequenz” (als Kehrwert einer bestimmten konstanten Perioden-Dauer) ist doch ebenso wie “Dauer” selbst eine Eigenschaft der Uhr an sich.

    Um die jeweils eigenen “Frequenzen” verschiedener Uhren miteinander zu vergleichen, muss man zunächst die “Potential“-Differenz zwischen ihnen ermitteln und
    berücksichtigen, bzw. die “Krümmung” der Region, in der beide enthalten waren.

  103. Duplikat bitte ignorieren bzw. entfernen

    > Frank Wappler schrieb (03.09.2012, 13:44)
    > [… Duplikat des Kommentars “03.09.2012, 13:39” ]

    … denn ca. 03.09.2012, 13:42 war der mit “03.09.2012, 13:39” benannte Kommentar noch nicht erkennbar veröffentlicht worden; was er inzwischen allerdings ist. Deshalb das Duplikat bitte ignorieren, oder löschen. Vielen Dank, FW.

  104. @Dr. Wolfgang Engelhardt

    Es scheint noch weitgehend unbekannt zu sein, dass das, was heute Lorentzsche Geometrie genannt wird, bereits 1873 von Felix Klein eingeführt wurde. Konsequenterweise kamen damit auch die Lorentz Transformationen auf, meines Wissens erstmals bei Wilhelm Killing (seinerzeit natürlich nicht unter diesem Namen und noch ohne jeden Bezug zur Physik). Auch ohne Voigt und die anderen Physiker hätte man so oder so bemerkt, dass der d’Alembert Operator auf Lorentzschen Räumen eine Rolle spielt, die mit der des Laplace Operators auf Riemannschen Mannigfaltigkeiten vergleichbar ist. Und in 4 Dimensionen führt das dann unausweichlich auf die Maxwellschen Gleichungen. Wenn die Physiker also nicht auf die RT gekommen wären, die Mathematiker wären jedenfalls darüber gestolpert.

    Aber egal, entscheidend für die Physik ist letztlich die experimentelle Prüfung der aus einer Theorie erhaltenen Aussagen. Gerade was die befremdlich wirkenden Konsequenzen der RT für den Gang von Uhren betrifft, da können die neuen Uhren sicherlich Klärung bringen, hinsichtlich “Uhrenparadoxon” und dergleichen. Und wie’s nun mal aussieht, verhalten sich diese Uhren in Übereinstimmung mit der RT [Pair of Aluminum Atomic Clocks Reveal Einstein’s Relativity at a Personal Scale]. Das ist mittlerweile zwei Jahre her, die Genauigkeit der Uhren lässt wohl noch deutlich verbessern [Single-Ion Nuclear Clock for Metrology at the 19th Decimal Place]. Man darf da durchaus gespannt sein.

  105. @Frank Wappler

    Es lässt sich ja z.B. physikalisch überhaupt keine Länge messen ohne zuvor eine Vereinbarung getroffen zu haben, wie geometrische Einheitslänge und physikalische Längeneinheit einander zugeordnet sein sollen. Und darum geht’s beim BIPM, eine solche Zordnung fällt nicht vom Himmel. Welche Erwägungen bei der Festlegung einer Masseinheit bedeutsam sind, hat MP anderweitig auch schon nachvollziehbar dargelegt. Wie zu messen ist, und was “Genauigkeit” in diesem Zusammenhang bedeutet, ist dann auch durch metrologische Regeln vorgegeben.

    Wie wird denn im Wapplerschen Unversum “Dauer” gemessen? Das liegt einstweilen noch verborgen im dunkeln der MTW Boxen …

  106. @Chrys

    Die Maxwellschen Gleichungen waren von Maxwell für den ruhenden Äther konzipiert worden, wie er am Schluss seines „Treatise“ ausdrücklich betont hat. Natürlich gelten sie nicht auf der rotierenden Erde, wie die Messungen von Michelson/Gale beweisen. Mit einem konstanten „c“ müssten nämlich die Umlaufzeiten rechts und links herum um das Sagnac-Interferometer von 1925 gleich sein, aber das ist eben nicht der Fall. Maxwells Wellengleichung wird erst „Lorentz-invariant“, wenn man die Zeit nach Voigt transformiert (http://www.kritik-relativitaetstheorie.de/…n.pdf , http://www.kritik-relativitaetstheorie.de/…T.pdf ). Doch dann liefert sie das falsche Resultat für das Michelson/Gale-Experiment. Transformiert dagegen nach Galilei t = t´, wie Sie es inzwischen tun, dann kommt man zum richtigen Ergebnis. Das Wandeln in Lorentzschen Räumen mag ein interessantes mathematisches Divertimento sein, eine physikalische Bedeutung hat es nicht.

    Nach all den gelehrten Erörterungen wüsste ich nun doch gerne, wie warm das Wasser wird, wenn man ein Pariser Kilogramm von einem Turm in ein Kalorimeter hinab wirft. Den Turm wollen wir so hoch aufrichten, wie 1000 altmodische, aneinander gelegte Pariser Urmeter ergeben. Wie können wir messtechnisch die Höhe dieses Turms an die moderne Lichtsekunde anschließen? Der Präsident der PTB wusste darauf keine Antwort. Vielleicht wissen Sie eine?

  107. Advances in Metrology

    Dr. Wolfgang Engelhardt schrieb (04.09.2012, 00:07):
    > […] Den Turm wollen wir so hoch aufrichten, wie 1000 altmodische, aneinander gelegte Pariser Urmeter ergeben.

    1. Vermutlich ist ein “altmodisches Pariser Urmeter” nicht per Definition “hoch[-wärts] aufgerichtet“.

    Wie sollte folglich entschieden werden, ob jeweils ein “altmodisches Pariser Urmeter” bei einer bestimmten Legung “hoch[-wärts] aufgerichtet” war ?

    2. Es gibt offenbar gar keine “1000 altmodische Pariser Urmeter“, sondern schon aus Prinzip nur ein einziges so bezeichnetes Artefakt: http://de.wikipedia.org/wiki/Urmeter

    Sofern nicht alle (1001) “Anlegestellen” des zur Diskussion stehenden Turmes koinzident mit dem einen oder anderen Ende des “altmodischen Pariser Urmeter“-Artefaktes sein sollen,
    gibt es irgendwelche Bedingungen an die erforderlichen Bestandteile des Turmes, die (“mal eben”) nicht koinzident mit dem einen oder anderen Ende des “altmodischen Pariser Urmeter“-Artefaktes sind?

    (Sollten z.B. alle Bestandteile des Turmes paarweise zueinander “hoch[-wärts] aufgerichtet” bleiben ? …)

    > Wie können wir messtechnisch die Höhe dieses Turms an die moderne Lichtsekunde anschließen?

    Vermutlich ist die Geometrie des vorgesehenen Turms gar nicht durch nur einen Messwert (“die Höhe“) zu fassen.

    In Anschluss an die SI-“Sekunden”-Definition wäre die geometrische Beziehung zweier bestimmter Turm-Enden stattdessen z.B. dadurch gekennzeichnet, dass die Ping-Dauer des einen (“unteren”) Endes (d.h. dessen Dauer von seiner Signalanzeige, bis zu seiner Anzeige der Wahrnehmung, dass das andere Ende die Signalanzeige wahrgenommen hatte)
    2000 / (299 792 458) Sekunden betrug,

    während die Ping-Dauer des anderen (“oberen”) Endes
    (1 + 9,8 * 1000 / (299 792 458)^2) * 2000 / (299 792 458) Sekunden betrug.

    Der Wert “Lichtsekunde” bezieht sich dagegen (üblicher Weise) auf einen Distanzwert; nicht auf einen Wert von Quasidistanz.

  108. @Dr. Wolfgang Engelhardt

    Nun, Maxwell hat das Ampèresche Gesetz hinsichtlich Verträglichkeit mit der Kontinuitätsgleichung modifiziert und ist dabei auf eine Wellengleichung gestossen. Ein Aether wurde dafür nicht gebraucht, der wurde nachträglich postuliert als ein zeitgenössischer Versuch der mechanistischen Deutung. Ludvig Lorenz, der in den 1860er Jahren unabhängig von Maxwell ebenfalls eine Theorie des Lichts formulierte, kritisierte den Aether damals schon als unwissenschaftliche Hypothese.

    Mir ist schon irgendwie klar, was Sie am SI Meter von 1983 stört. Was soll die SI-Höhe eines Turms sein, wenn man Lichtlaufzeit bei nicht-konstanter Lichtgeschw. misst, wo die Uhren unten und oben am Turm unterschiedlich gehen? Aber es existiert ein Bezugssystem, wo sich Zeit mittels der SI-Konstanten c in Länge konvertieren lässt, nämlich das mit dem Stein frei fallende Bezugssystem, parametrisiert durch Eigenzeit.

    Bevor Sie die RT jetzt dafür schelten, dass sie simple Fallgesetze nur unnötig verkompliziert, bedenken Sie, dass das klassische Pendant zur relativistischen Bewegung durch Jacobis Formulierung des Prinzips der kleinsten Wirkung gegeben wird. Die Darstellung durch Geodäten ist halt nicht unbedingt handlich, ihre Nützlichkeit liegt dabei woanders.

  109. @Chrys

    Und wie warm wird das Wasser?

    Nachdem Sie nicht selber nachlesen, schreibe ich Ihnen mal hier hin, wie Maxwell seinen Treatise beendet:
    „…Hence all these theories lead to the conception of a medium in which the propagation takes place, and if we admit this medium as an hypothesis, I think it ought to occupy a prominent place in our investigations, and that we ought to endeavour to construct a mental representation of all the details of its action, and this has been my constant aim in this treatise.”
    Und bei Einstein heißt es 1920: „Gemäß der allgemeinen Relativitätstheorie ist ein Raum ohne Äther undenkbar; denn in einem solchen gäbe es nicht nur keine Lichtfortpflanzung, sondern auch keine Existenzmöglichkeit von Maßstäben und Uhren, also auch keine räumlich-zeitlichen Entfernungen im Sinne der Physik.“

    Glauben Sie wirklich, dass diese Physiker ohne den Äther ausgekommen sind, weil sie ihn für eine „unwissenschaftliche Hypothese“ hielten? Maxwell war sich übrigens darüber im Klaren, dass seine Theorie nur im ruhenden Äther gilt. Er hat deshalb einen Vorschlag gemacht, wie man den auf der Erde zu erwartenden Ätherwind messen könnte. Nach seinem Tod hat Michelson die Idee aufgegriffen und versucht, mit seinem Interferometer die Geschwindigkeit der Erde zu ermitteln. Leider war dieses Instrument ungeeignet, wie damals schon Voigt, Cohn und auch Einstein selber wussten. Erst der COBE-Satellit hat die Absolutgeschwindigkeit des Sonnensystems zweifelsfrei gemessen: 370 km/s Richtung τ leonis.

  110. Plitsch – Platsch

    Chrys schrieb (03.09.2012, 17:05):
    > […] zuvor eine Vereinbarung [treffen]

    Sicher. Aber Aussagen “Vereinbarungen” zu nennen, die Worte bzw. Phrasen benutzen, die erst noch als Begriffe vereinbart werden müssten, entspricht der Art von Illusion, vor der Einstein warnte (“Relativität”, § 8).

    > Wie wird denn im Wapplerschen Unversum “Dauer” gemessen? Das liegt einstweilen noch verborgen im dunkeln der MTW Boxen …

    Na konkret insbesondere Box 16.4 (“Ideal Rods and Clocks built from […]”), wo das Symbol “τ0” mehrfach auftaucht, wodurch sich die Gleichheit der geometrischen Beziehungen bestimmter Ereignispaare ausgedrückt;
    zusammen mit Box 10.2 (“From Geodesics to [… Schild’s Ladder]”).

    Die “dunklen Stellen” darin sind (insbesondere) die dort auftauchenden Phrasen “freely falling” sowie “affine parametrization“, die einerseits natürlich Vereinbarungen erfordern, sich andererseits dafür aber nicht des (erst nachfolgend definierten) Begriffes “Dauer” bedienen können (sondern eben nur noch selbstverständlicher Begriffe wie “Reihenfolge” bzw. “Koinzidenz”).

    Chrys schrieb (04.09.2012, 15:06):
    > […] wo die Uhren unten und oben am Turm unterschiedlich gehen
    Mit welchem “Genauigkeits“-Begriff soll diese Aussage denn vereinbar sein??

    > Aber es existiert ein Bezugssystem, wo sich [Dauer] mittels der SI-Konstanten c […] konvertieren lässt, nämlich das mit dem Stein frei fallende Bezugssystem

    Die Dauer des (eines bestimmten) “frei fallenden Steins” zwischen welchen beiden seiner Anzeigen soll denn (durch Multiplikation mit “c”) konvertiert werden, um die geometrische Beziehung der beiden Turm-Enden zueinander zu charakterisieren ?

  111. @Dr. Wolfgang Engelhardt

    Beim freien radialen Fall in der Schwarzschild Metrik

    A² dt² − 1/A² dr² − r² h, wo A² = A(r)² = 1 − a/r,

    kann man zumindest Fallgeschwindigkeit und Fallhöhe in SI-Einheiten ermitteln. Aus den Komponenten eines Tangentialvektors (dt/ds,dr/ds,0,0) einer nach Eigenzeit s parametrisierten, radial frei fallenden Bewegung, (t(s),r(s),θ,φ), lässt sich unter Verwendung von standard Minkowski Koordinaten [wo also c = 1 = const.] und einem Lorentz boost auf einem Tangentialraum die Beziehung

    dr/ds = −v A(r(s))² dt/ds

    für die Geschw. v = v(s) erhalten. Wird s in Sekunden gemessen, dann wäre der Wert von v noch mit dem SI Faktor 299,792,458 zu versehen, um auf Meter/Sek. zu kommen. Fallhöhe folgt dann durch Integration über die Dauer des Sturzes.

    Maxwell war ein Kind seines mechanistischen Zeitalters, wo sich noch kaum jemand eine Welle ohne ein stoffliches Medium vorstellen konnte. Es gab nie eine widerspruchsfreie Vorstellung von den einem stofflichen Aether unterstellten Eigenschaften. Mal wird er mitgeschleppt, mal ruht er, mal bläst ein Aetherwind — ganz abhängig vom Experiment, zu dessen Deutung seine Apologeten ihn gerade heraufbeschwörten. Das ist in der Tat dann nicht sehr wissenschaftlich.

    Hermann Weyl hatte einmal bemerkt, dass man den metrischen Tensor als Aether betrachten kann. In diesem Sinne ist das in Ordnung, dieser Aether hat dann aber nur geometrische Eigenschaften.

    Dass die CMB Anistropie mit all dem nichts zu tun hat, dass ist doch schon x-mal irgendwo dokumentiert worden. Der CMB ist ein kosmologisches Ding und ohne Belang für die Frage nach einem lichttragenden Aether.

  112. @Frank Wappler

    MTW vermittelt eine für mich machmal befremdliche Vorstellung von den Dingen. Da wird im Anfang mal irgendwo gesagt, Zeit soll in Sekunden und Länge in geometrodynamischen Centimetern gemessen werden, und diese Masseinheiten sind dann immer mindestens im Hinterkopf mit dabei. Die Identifizierung von Einheitsmass mit Masseinheit ist für mich die klare Schnittstelle zwischen theoretischem und experimentellem Bereich, zwischen Geometrie und Metrologie. Bei MTW soll wohl diese Grenze möglichst verschwinden und die Physik als ein einheitliches Ganzes erscheinen. Was MTW mit den Boxen erklären wollen, erschliesst sich mir nicht immer so recht. Das bringt vielleicht auch hier mancherlei Missverständnisse auf.

  113. Turmbau zu Braunschweig ?

    Chrys schrieb (05.09.2012, 15:27):
    > […] Was MTW mit den Boxen erklären wollen, erschliesst sich mir nicht immer so recht.

    Ich schätze die genannten MTW Boxen (13.1, 10.2, 16.4), weil darin Zugänge zur ART dargestellt sind, die nicht mit irgendwie dahingestreuselten (Koordinaten-)Zahlentupeln aufmachen, sondern mit “principal identifiables […] buoys, ships, icebergs, lighthouses” “wie du und ich” und wie “A”, “B”, “M” usw. aus Einsteins grundlegenden Darstellungen der SRT; bzw. mit deren (einzelnen) geordneten Mengen von Anzeigen.

    Chrys schrieb (05.09.2012, 14:46):
    > Beim freien radialen Fall in der Schwarzschild Metrik […]

    Dr. Wolfgang Engelhardt hat zwar noch nicht nachvollziehbar gemacht, was in seinem Kommentar (04.09.2012, 00:07) unter “Turm” zu verstehen sein soll;
    aber wir sollten (zumindest ersatzweise bis auf Weiteres) wohl ein Paar Enden entsprechend meinem Kommentar (04.09.2012, 10:47) betrachten, also deren gegenseitige Pingdauern ungleich (aber einzeln konstant) sind;
    wobei die Differenz (insbesondere im Verhältnis zur Summe) signifikant (größer als Null) sein soll.
    Die geometrische Beziehung zwischen diesen beiden wesentlichen Beteiligten (und auch weiteren geeigneten Beteiligten) ist demnach offenbar nicht durch eine Metrik dargestellt, sondern zunächst durch eine Quasimetrik.

    Wie soll dieser Quasimetrik denn ein bestimmter metrischer Raum oder ein sogar metrischer Tensor zugeordnet werden ??

  114. @Chrys

    • Dr. Wolfgang Engelhardt @Chrys

    Ausgezeichnet! Nun bauen wir aber erst mal den Turm (wie das geht können Sie von einem 3jährigen Kind erfahren, Herr Wappler) und messen seine Höhe von Fuß bis Spitze in Lichtsekunden aus, bevor wir den Stein fallen lassen. Dazu können wir einen Lichtpuls von unten nach oben schicken, aber wir wissen nicht, wie lange er braucht, denn wir haben keine synchronisierten Uhren oben und unten. Die gibt es auch nicht, denn die Frequenz der Cäsiumuhren hängt von der Höhe des Aufstellungsortes ab, so dass sie schlicht nicht synchronisierbar sind. Behelfen wir uns also mit einem Spiegel und schicken wir das Licht hinauf und hinunter. Jetzt bekommen wir ein messbares Zeitintervall, nur leider ist es nicht dasselbe, wenn wir den Puls von oben nach unten schicken und wieder hinaufspiegeln. Zudem ist die Lichtgeschwindigkeit eine Funktion des Graviationspotentials und wir wissen nicht, auf welches Potential sich die berühmten 9 Zahlen beziehen. So geht es also nicht.

    Chrys´ Methode, erst den Stein hinunterzuwerfen und aus der Fallzeit die Turmhöhe zu bestimmen, erinnert an die berühmte Prüfungsfrage: Wie messen Sie mit einem Thermometer und einer Stoppuhr die Höhe eines Turmes? Antwort: Ich werfe das Thermometer hinunter und stoppe die Zeit bis zum Aufschlag. Auch hier wieder das Problem: Wie messe ich denn das Zeitintervall zwischen Abwurf oben und Ankunft am Boden? Wieder bräuchte ich oben und unten synchronisierte Uhren, die es nicht geben kann.

    Professor Göbel wusste schon, warum er meine Frage nicht mehr beantworten wollte. Es handelt sich hier nicht um eine haarspalterische, scholastische Spitzfindigkeit, sondern es geht darum, wie man den Energiesatz im neuen Einheitensystem ohne Urmeter formulieren muss und kann, bzw. wie man experimentell verifizieren kann, dass er stimmt. Wir wollen ja verhindern, dass ein cleverer Tüftler mit einem Vorschlag zum Bau eines perpetuum mobile zum Patentamt läuft, wir ihn aber nicht widerlegen können, weil das neue Einheitensystem das nicht hergibt. Zwei unterschiedliche Höhen für denselben Turm stimmen jedenfalls sehr bedenklich. Ich erspare Ihnen hier, daraus ein perpetuum mobile zu entwickeln.

    Der CMB ist nicht nur „ein kosmologisches Ding“, sondern auch ein elektromagnetischer Wellensalat, der sich im absoluten Raum nach allen Seiten isotrop mit Lichtgeschwindigkeit fortpflanzt. Einstein hielt es für undenkbar, dass dies ohne einen „Lichttragenden Aether“ geschehen könne. Aber Sie wissen das offenbar viel besser. Ich möchte Ihnen nicht widersprechen, doch wäre ich Ihnen dankbar, wenn Sie mir mitteilen könnten, mit Hilfe welcher kosmologischer Grundsätze man genau jene 370 km/s Richtung τ leonis herleiten kann.

  115. @Dr. Wolfgang Engelhardt

    Bei dem zuvor angedachten freien Fall ist allerdings bedeutsam, dass eine Uhr mit hinunterfällt, und nach deren Eigenzeit ist die Dauer des Falls zu bestimmen. Das Tangentialbündel längs einer Geodäten verhält sich quasi wie ein einzelner Minkowski Raum, indem sich seine einzelnen Räume via Parallelverschiebung miteinander identifizieren lassen, ohne dass dabei die Regeln der SR (“Konstanz der Lichtgeschw.”) verletzt werden. Somit ist auf diese Weise die SI Konvertierung Sekunde → Meter bei konstanter Proportionalität möglich.

    In die Deutungen der Kosmologie habe ich kein nennenswertes Vertrauen, und es braucht keinen Propheten um vorherzusagen, dass das derzeit verkündete kosmologische Weltbild gewiss nicht von dauerhaftem Bestand sein wird. Die CMB Anisotropie könnte ja auch ein Hinweis darauf sein, dass mit dem kosmologischen Prinzip etwas nicht stimmt, weiss der Kuckuck. Aber eine Aetherdrift von ca. 370 km/s, das sollte sich auch anderswo bemerkbar machen. Moderne Versionen des Michelson-Morley Experimentes sollten eine Aetherdrift entdecken können, sofern diese nicht kleiner als 15 m/s ist. Wie soll das zusammenpassen?

  116. Weit aus dem Fenster

    Dr. Wolfgang Engelhardt schrieb (06.09.2012, 00:08):
    > Nun bauen wir aber erst mal den Turm (wie das geht können Sie von einem 3jährigen Kind erfahren

    Es dürfte manche offenbar überraschen, wie viele ungleiche “Türme” 3-jährige Kinder als “1000 altmodische, aneinander gelegte Pariser Urmeter” in einem Sandkasten unterbringen könnten; geschweige denn darüber hinaus.
    (D.h. ungleich hinsichtlich der einen und/oder der anderen Ping-Dauer der beiden wesentlichen “Enden des Turmes“.)
    Jedenfalls sofern nicht von maßgeblicher Seite öfter mal “So geht es [aber] nicht!” dazwischengequakt würde.

    > […] Behelfen wir uns also mit einem Spiegel und schicken wir das Licht hinauf und hinunter.

    Na also.
    Aber wenn schon, dann zwei Spiegel. Es sollen doch wohl beide Enden einander beobachten; und auch hinreichend “unablässig”.

    > Jetzt bekommen wir ein messbares Zeitintervall,

    Wir” bekommen bzw. identifizieren dadurch jede Menge (verschiedene) messbare Zeitintervalle des einen Endes (jeweils von einer bestimmten seiner Anzeigen als Signal, bis zu seiner Anzeige der Wahrnhemung, dass das andere Ende dieses Signal wahrgenommen hatte), und auch jede Menge (verschiedene) messbare Zeitintervalle des anderen Endes.
    Und die (Mess-)Größe, mit der Zeitintervalle quantifiziert werden, heißt “Dauer”, nicht wahr?

    > nur leider ist es nicht dasselbe […]

    Die Dauern all der verschiedenen genannten Zeitintervalle des einen Endes sollten einander gleich sein;
    und die Dauern all der verschiedenen genannten Zeitintervalle des anderen Endes sollten einander gleich sein;
    und die Dauern all der verschiedenen genannten Zeitintervalle des einen Endes sollten ungleich den Dauern all der verschiedenen genannten Zeitintervalle des anderen Endes sein — stimmt’s?.

    > Zudem ist die Lichtgeschwindigkeit eine Funktion des Graviationspotentials

    ???
    Was genau wäre “eine Funktion des Gravitationspotentials“:
    das Wort “Lichtgeschwindigkeit“,
    oder der Buchstabe “c”,
    oder die Phrase “299 792 458 m/s”,
    oder das Wort “Licht” einzeln,
    oder die Definition “(Durchschnitts-)Geschwindigkeit := Distanz von Start und Ziel zueinander, geteilt durch die Dauer …”,
    oder der Dauerwert “1 Sekunde”
    ?

    > Chrys´ Methode, erst den Stein hinunterzuwerfen und aus der Fallzeit die Turmhöhe zu bestimmen

    Soweit hat’s Chris ja offenbar (noch) nicht getrieben.
    D.h., wir sind uns doch hoffentlich einig, dass der Wert
    “299 792 458 m/s * Fall-Dauer des Steins”
    nicht “Turmhöhe” oder auch nur “Distanzwert” genannt werden sollte.

    (Die Betrachtung des Falles, anstatt eines “Pariser Kilogramm“s ein Photon zu werfen, können wir uns hier wohl sparen …)

    Trotzdem …
    > Wie messe ich denn das Zeitintervall [des Steins] zwischen Abwurf oben und Ankunft am Boden?
    … ist eine ausgezeichnete Frage.

    > Wieder bräuchte ich oben und unten synchronisierte Uhren, die es nicht geben kann.

    Zunächst mal: Dass sich insbesondere Uhren (“oben und unten“) nicht synchronisieren lassen, die dabei beide mit Perioden-Dauern von “1 Sekunde” tickten bzw. parametrisiert waren, hängt eng damit zusammen, dass diese Dauer eben gleich sein soll; ganz unabhängig vom “oben” oder “unten” oder “Gravitationspotential” usw.

    Zweitens: Was wäre denn in dem einfacheren Fall, dass die beiden Enden/Uhren im Flachen wären, oder zumindest … ähm … gleich “hoch”; also mit gleichen (und sowieso konstanten) gegenseitigen Pingdauern, so dass die Synchronisation gelingt, falls beide mit gleichen Perioden-Dauer “tickten”.

    Wie wäre denn in diesem Fall die Dauer des Zeitintervals des Steins (von seiner Anzeige des Aufbruchs vom einen Ende, bis zu seiner Anzeige der Ankunft am anderen Ende) zu messen? …
    (Die anschließende Betrachtung weiterer Steine, die sich gegenseitig beobachten und ihre Pingdauern untereinander vergleichen usw. mag man sich ausmalen.)

    Im Übrigen: wer (vielleicht irgendwann mal) ausreichend Geometrie bzw. Kinematik verstehen sollte, um damit guten Gewissens Variationsrechnung (d.h. Dynamik) zu betreiben, sollte sich doch nicht zuletzt darum kümmern, ob und wie man “Brechungsindex” messen kann.

    p.s.
    Ich versteh ja noch nicht mal, ob die Fall-Dauer (des Zeitintervalls beim Heruntersausen) des Steins unbedingt gleich der Steig-Dauer (des Zeitintervalls beim Hinauffliegen) des Steins wäre …

  117. @Chrys 06.09.2012, 12:47

    Eine tolle Idee! Ich bewundere Ihren Einfallsreichtum. Wenn ich also den Abstand zur Sonne wissen will, weil ich ihr Graviationspotential – G M/r auf der Erde benötige, besorge ich mir eine feuerfeste Version dieser Cäsiumuhr: http://www.watchtime.net/…006/CH_2006_02_016.pdf , schieße sie auf die Sonne, lese die benötigte Zeit bis zum Aufschlag ab, berechne daraus die Geschwindigkeit, finde damit die kinetische Energie, welche der potentiellen Energie auf der Erde, abzüglich der Anfangsenergie gleich sein muss. Daraus kann ich letztendlich den Abstand zur Sonne berechnen, wenn ich noch einige Korrekturen wegen des vorhandenen Erdpotentials berücksichtigt habe. Nicht ganz leicht durchzuführen, aber wenigstens exakt und im Einklang mit dem neuen Einheitensystem, welches ohne Urmeter auskommt.

    Wie Sie schon wissen, hat auch Einstein das Michelson-Morley Experiment nicht als Bestätigung seiner Theorie akzeptiert, weil es ungeeignet ist, eine Ätherdrift festzustellen. Schließlich kannte er das Nullresultat und hat trotzdem 1920 ein Plädoyer für die Existenz des Äthers gehalten. Wenn Sie Genaueres wissen wollen, können Sie mal hier nachlesen: http://www.worldsci.org/…acts/abstracts_6815.pdf

  118. @Dr. Wolfgang Engelhardt

    Den Abstand zur Sonne mit einem Urmeter zu messen, wäre auch nicht ganz leicht durchzuführen. Ein Ingenieur, dem nichts zu schwör ist, muss sich da jedenfalls schon einiges einfallen lassen.

    So manches aus der Experimentalphysik übersteigt meinen Horizont bei weitem. Was und warum mit welchem Experiment gezeigt oder widerlegt wird, das sollten zuvörderst die Physiker untereinander klären. Irgendwann stiess ich mal auf einen Italiener, Maurizio Consoli, von INFN, der auch für den Aether streitet und eine Aetherdrift von ca. 200 km/s festgestellt zu haben meint (einiges von ihm ist hier zu finden: http://www.nu.to.infn.it/SuperLuminal_Neutrino/). Aber im Stil von Michelson-Morley, glaube ich. Tja, da bleibt unsereinem letztlich wohl die Qual der Wahl und darf sich aussuchen, was am ehesten plausibel erscheint.

  119. Chrys schrieb (07.09.2012, 11:33):
    > Ein Ingenieur, dem nichts zu schwör ist, muss sich da jedenfalls schon einiges einfallen lassen.

    Der Ingeniör,
    der hat es leicht:
    Genauigkeit wird ihm gereicht
    vom Physikör.

  120. @Frank Wappler / It ain’t easy

    Technische Probleme:

    Der Ingeniör baut Türme
    bis heisse Sonnenstürme
    und die Protuberanzen
    erzeugen Resonanzen.
    Und dann noch — ach du Shrek!
    — schmilzt ihm sein Meter weg.

  121. Verachte doch die Ing.ster nicht

    Chrys schrieb (07.09.2012, 19:51):
    > Der Ingeniör baut Türme
    > bis heisse Sonnenstürme […]
    > […] — schmilzt ihm sein Meter weg.

    Mal nicht die Ingenieure unterschätzen!

    Seit sie mit der SI-“Meter”-Definition die Einstein-Syngesche Distanzdefinition praktisch einsetzen (sollen), und seit sie (spätestens seit 1997) wesentliche Stücke ihres Sammelsuriums “auf 0 K beziehen” (sollen; vgl. http://www.bipm.org/…re/chapter2/2-1/second.html), verfügen sie wohl über ein ziemlich nachvollziehbares Maß zur Bewertung des eventuellen “Wegschmelzens” (von Stücken ihres Sammelsuriums).

    Damit ausgestattet können sie doch zielstrebig lostüfteln (oder zumindest zielführend experimentieren bzw. botanisieren).
    Und wer weiß: vielleicht projektieren sie ja tatsächlich mal einen Turm, von dem zu erwarten ist, dass er (im Rahmen einer bestimmten, reellen Genauigkeit) eben _nicht_ “wegschmelzen würde”;
    und vielleicht könnte ein derartiger Turm (im Rahmen einer bestimmten, reellen Genauigkeit) dann sogar hergestellt und/oder vorgefunden werden, der (mit etwas Glück, im Rahmen einer bestimmten, reellen Genauigkeit) eben nicht “wegschmilzt“.

    Schaun wir mal, was Physiker noch anbieten, worauf doch im Prinzip eigentlich jeder (Ingenieur) auch selber hätte kommen können …

  122. @Frank Wappler

    »Mal nicht die Ingenieure unterschätzen!«

    Bestimmt nicht, aber selbst den ingeniösen Ingenieur aus Entenhausen könnte man in Schwierigkeiten bringen mit der Aufgabe, gewisse Abstandsmessungen durch unmittelbare Anwendung einer Urmeter-Definition durchzuführen. (Wobei ich gestehen muss, dass es sich meiner Kenntnis entzieht, welche Meter-Definition in Entenhausen gültig ist.)

    Dass die Definition des SI Meters (1983) im Kontext der allg. Relativität bisweilen nicht unmittelbar zur Abstandsmessung angewendet werden kann, ist in [Recommendation 1 (CI-2002)] des CIPM dokumentiert.

  123. Selber Essen treibt Düse

    Chrys schrieb (09.09.2012, 09:25):
    > […] in Schwierigkeiten bringen mit der Aufgabe, gewisse Abstandsmessungen durch unmittelbare Anwendung einer Urmeter-Definition durchzuführen.

    Vergleichbar mit den Schwierigkeiten, die man sich einhandelt, wenn man Dauer- bzw. Frequenzmessungen auf irgendwelche “Normale” zurückführen wollte (seien es nun Solarsysteme, oder Quartz-Kristalle, oder Cs133-Atome, oder welche Sau auch immer gerade durch’s Dorf rennt), anstatt der nachvollziehbaren Mess-Definition(en), durch deren Anwendung man auch Genauigkeit jedes einzelnen Artefakts bewerten würde.

    > Dass die Definition des SI Meters (1983) im Kontext der allg. Relativität bisweilen nicht unmittelbar zur Abstandsmessung angewendet werden kann, ist in [Recommendation 1 (CI-2002)] des CIPM dokumentiert.

    Sicher empfiehlt (und nicht zuletzt: beachtet) das Kommittee auch, wie zumindest im Prinzip zu entscheiden wäre, ob gegebene Beobachtungsdaten “im Kontext der allg. Relativität” auszuwerten sind, oder nicht ?
    (Mal sehn, wie die geometrischen Beziehungen bzw. deren Unsicherheiten ermittelt wurden, die “Metrologia 34 (3), 261, 1997” in Betracht zog …)

Schreibe einen Kommentar